You are on page 1of 148

Preface

Dear readers, we have started edristi English edition as well since August, 2015. We are hopeful that it

will help us to connect to the broader audience and amplify our personal bonding with each other.

While presenting Day-to-day current affairs, we are very cautious on choosing the right topics to make

sure only those get the place which are useful for competitive exams perspective, not to increase

unnecessary burden on the readers by putting useless materials. Secondly, we have also provided the

reference links to ensure its credibility which is our foremost priority. You can always refer the links to

validate its authenticity.

We will try to present the current affairs topics as quickly as possible but its authenticity is given higher

priority over its turnaround time. Therefore it could happen that we publish the incident one or two

days later in the website.

Our plan will be to publish our monthly PDF on very first day of every month with making appropriate

modifications of day-to-day events. In general, the events happened till 30th day will be given place in

the PDFs. The necessity of this is to ensure the contents factual authenticity.

Reader’s satisfaction is our utmost priority so requesting you to provide your valuable feedback to us.

We will warmly welcome your appreciation/criticism given to us. It will surely show us the right

direction to improve the content quality. Hopefully the current affairs PDF (from 1st January to 31st

January) will benefit our beloved readers.

Current affairs data will be useless if it couldn’t originate any competitive exam questions. E-Dristi has

been very successful in that direction. Almost all the questions from UPPCS and other examinations have

been asked from our materials. You can verify that by matching the question papers and e-Dristi

contents from yourselves.

1
http://www.edristi.in/
National
Interim Budget 2019-20
Question: Consider the following statements regarding Budget-2019:
(1) For 2019-20 budget size is 27.84 lakh crore
(2) PM-KISAN Yojana is a direct income support scheme targeted at small and marginal
farmers
(3) Fiscal deficit has achieved the FRBM target of 3.2 percent this year
(4) Government has proposed to establish a new department for fisheries
Of the above correct statements are:
(a) 1, 2 and 3 only
(b) 2, 3 and 4 only
(c) 1, 2 and 4 only
(d) All of the above
Answer: (c)
Related facts:

 With an aim to realize a ‘New India’ by 2022, Interim budget 2019-20 was presented by
the Union Minister for Finance, Corporate Affairs, Railways & Coal Piyush Goyal on
February 1st, 2019.
 For 2019-20 the budget size is 2784200 crore Rs which is 13.3 percent greater than the
2018-19 budget of 2457235 crore Rs (Revised Estimates).
 As of now, India became 6th largest economy of the world from 11th largest economy in
the world in 2013-14.
 Budget deficits:

 The fiscal deficit has been reported to 3.4%(634398 crore Rs) in 2018-19 (Revised
Estimates) from the high of almost 6% seven years ago.For year 2019-20 it is estimated
to be 3.4 percent(703999 crore Rs).
 Target of 3 per cent to be achieved by 2020-21.
 Fiscal deficit is the difference between the Revenue Receipts plus Non-debt Capital
receipts (NCDR) and the total expenditure. Fiscal Deficit is reflective of the total
borrowing requirements of government.
 Revenue Deficit has been reported to 2.2 per cent of GDP in 2018-19(Revised
Estimates) and is estimated to be same for 2019-20. Revenue Deficit refers to the
excess of revenue expenditure over revenue receipts.
 Effective Revenue Deficit is estimated to be 1.3 per cent of GDP(2, 69, 474 crore Rs) in
2019-20.However,it was 1.1 percent in the Revised Estimates of 2018-19. Effective
Revenue Deficit is the difference between revenue deficit and grants for creation of
capital assets.
 Primary Deficit is estimated to be 0.2 per cent of GDP(38,938 crore Rs)in 2019-20 and
was same for 2018-19(Revised Estimates). Primary Deficit is measured as fiscal deficit
less interest payments.
 Current Account Deficit, against a high of 5.6 per cent six years ago, is likely to be only
2.5 per cent of GDP this year.
 Major schemes:

 “Pradhan Mantri- KIsan SAmman Nidhi (PM-KISAN) “to extend direct income support at
the rate of Rs. 6,000 per year to farmer families, having cultivable land upto 2 hectares;
with a total outlay of Rs. 75,000 crore for financial year 2019-20 and Rs. 20,000 crore in
the revised estimates of financial year 2018-19.

2
http://www.edristi.in/
 It will benefit approximately 12 crore Small & Marginal farmer families. Farmers will get
first installment (1st out of 3) of Rs. 2,000 at the end of March, 2019 as the scheme is
implemented retrospectively form December, 1st 2018.
 To provide sustained and focused attention towards development of fisheries, the
government has decided to create a separate Department Of Fisheries. In the future this
will ensure growth over 7% to promote livelihood of about 1.45 crore people dependent
on the sector.
 To provide pensionary benefits to at least 10 crore labourers and workers in the un-
oragnised sector a new scheme called ‘Pradhan Mantri Shram- Yogi Maandhan’ is
announced. A sum of Rs.500 crore has been allocated for the Scheme.
 Rs. 3000 per month after 60 years of age with an affordable contribution of only Rs
100/55 per month.
 Allocation increased to Rs. 750 crore for Rashtriya Gokul Mission has been announced
for the current year itself. Setting up of “Rashtriya Kamdhenu Aayog” to upscale
sustainable genetic upgradation of cow resources and to enhance production and
productivity of cows has also been announced.
 The Aayog will also look after effective implementation of laws and welfare schemes for
cow.
 Jan Dhan-Aadhaar-Mobile (JAM) and Direct Benefit Transfer:In the last five years,
nearly 34 crore Jan Dhan bank accounts opened with Aadhaar now near universally
implemented so that that the poor and middle class receive the benefits of Government
schemes directly in their bank accounts by eliminating middlemen.
 As per the provisions of Fiscal Responsibility and budget Management Act, 2003, it is
mandatory to present following two statements:
 Macro- Economic Framework Statement
ii. Medium-Term Fiscal Policy cum Fiscal Policy Strategy Document

 Rupee comes from: (2019-20 Budget Estimates) – 2784200 crore

 Corporation tax (21 paise)


 Income tax (17 paise)
 Customs (4 paise)
 Union excise duties (7 paise)
 Goods and Service Tax (21 paise)
 Non-Tax Revenue (8 paise)
 Non-Debt Capital Receipts (3 paise)
 Borrowings & other liabilities (19 paise)
 Rupee goes to: (2019-20 Budget Estimates)

 Central Sector scheme (12 paise)


 Interest payments (18 paise)
 Defence (8 paise)
 Subsidies (9 paise)
 Finance Commission & Other Transfers (8 paise)
 State’s share of taxes & Duties (23 paise)
 Pensions (5 paise)
 Centrally sponsored scheme (9 paise)
 Other expenditure (8 paise)
 Facts about Interim budget:

 The government of the day presents an interim budget if it does not have the time to
present a full budget or because National Elections may be near.
 When the National Elections are near, propriety demands that the task of framing the full
budget be left to the incoming government.
3
http://www.edristi.in/
 Tax benefits:

 Individual tax payers having taxable annual income up to Rs. 5 lakh will not be required
to pay any income tax.
 Persons having income up to Rs. 6.50 lakh are not required to pay any income tax if
they make investment in provident fund, specific savings and insurance etc.
 Standard deduction limit has also been raised to Rs. 50,000 from Rs. 40,000 for salaried
persons. This will ensure additional tax benefit of ` 4,700 crore to more than 3 crore
salary earners and pensioners.
 Thus tax benefit of Rs. 18, 500 crore is proposed to be provided to an estimated 3 crore
middle class and small taxpayers comprising self employed, small business, small
trades, salary earners, pensioners, and senior citizens.
 To benefit small depositors and nonworking spouses, TDS threshold on interest earned
on bank/post office deposits is being raised from Rs 10,000 to Rs 40,000.TDS threshold
for deduction of tax on rent is proposed to be increased from 1, 80,000 to 2, 40, 000 for
providing relief to small taxpayers.
 Tax collection nearly doubled in five years- form Rs. 6.38 lakh crore in 2013-14 top
almost Rs. 12 lakh crore this year.
 80% growth in tax base- form 3.79 crore to 6.85 crore in five years.
 GST (GOODS & SERVICES TAX):

 GST made India a common market.


 GST led to increased tax base, higher collections and ease of trade.
 Inter-state movements now faster, more efficient, and hassle free.
 Responsive and sensitive reduction of tax rates – Most items of daily use now in the 0%
or 5% tax slab.
 Relieving the businesses and service providers-
 Exemptions from GST for small businesses doubled from Rs. 20 lakh to Rs. 40 lakh.
 Small businesses having turnover up to Rs. 1.5 crore pay only 1% flat rate and file one
annual return only.
 Small service providers with turnover upto Rs.50 lakhs can opt for composition scheme
and pay GST at 6% instead of 18%.
 Soon, businesses comprising over 90% of GST payers to be allowed to file quarterly
return.
 Encouraging GST revenue trends – The average monthly tax collection in the current
year is Rs. 97,100 crore per month as compared to Rs. 89,700 crore per month in the
first year.
 Inflation:

 Government has been successful in bringing down average inflation to 4.6 per cent over
last five years which is lower than the inflation during the tenure of any other
government.
 If this rate had not been controlled, our families would have spending around 35-40 per
cent more today on basic necessities such as food, travel, consumer durables, housing
etc.
 The average rate of inflation during previous years 2009-14 was as high as 10.1 per
cent.
 Healthcare:

 World’s largest healthcare programme Ayushman Bharat launched to benefit nearly 50


crore with more than 10 lakhs patients already received benefits from the Scheme.
 Medicines at affordable prices have been made available through Pradhan Mantri Jan
Aushadhi Kendras.

4
http://www.edristi.in/
 Prices of essential medicines, cardiac stents and knee implants are reduced in order to
benefit poor & middle class of the society.
 21 AIIMS operating /already established- 14 announced since 2014; 22nd AIIMS to be
set up in Haryana.
 Allocation for Integrated Child Development Scheme (ICDS) is being increased to Rs.
27,584 crore in BE 2019-20.
 Deprived sections: (SC, ST, NOMADS, WOMAN AND CHILD)

 The allocation amount is proposed to be enhanced to Rs. 76,801 crore has been made
for scheduled caste in BE 2019-20.
 A Welfare Development Board to frame special strategies for the benefit of the hard-to
reach de-notified, nomadic and semi-nomadic communities will be set up under the
Ministry of Social Justice and Empowerment.
 A committee under NITI Aayog will be set up to complete the task of identifying de-
notified, nomadic and semi-nomadic communities not yet formally classified.
 The Ujjwala Yojna delivery of 8 crore free LPG connections, more than 6 crore
connections have already been given and the remaining will get free gas connections by
next year.
 More than 70% of beneficiaries of Pradhan Mantri MUDRA Yojana are women.
 Benefits of maternity leave of 26 weeks and Pradhan Mantri Matru Vandana Yojna for
pregnant women, financial empowerment of women by increased participation in work.
 Rs. 60,000 crore has been allocated for MGNREGA in BE 2019-20.
 25% additional seats in educational institutes to meet the 10% reservation for the poor
(apart from SC/ST and OBC).
 Aspirational district programme for development in 115 most backward districts of India.
 Free electricity connection to every household under Saubhagya Yojna.
 Infrastructure:

 Under UDAN Scheme Number of operational airports crossed 100.


 Domestic passenger traffic has doubled during the last five years.
 India has become fastest highway developer in the world – 27 kms built each day.
 Arunachal Pradesh came on the aviation map of India recently.
 Meghalaya, Tripura and Mizoram came on the India’s rail map for the first time.
 Container Cargo Movement is introduced by improving the navigation capacity of the
river Brahmaputra.
 Projects stuck for decades have been completed.
 Container freight movement has started on inland waterways from Kolkata to Varanasi.
 National program on ‘Artificial Intelligence’ has been envisaged by the government.
 National Centre on Artificial Intelligence as a hub along with centers of excellence will be
established.
 More than 3 lakh Common Services Centres (CSCs) exist employing nearly 12 Lakh
people, are digitally delivering numerous services to the citizens.
 98% rural sanitation has been covered with 5.45 lakh villages have been declared as
“open defecation free”.
 Energy:

 India’s installed solar generation capacity has grown over ten times in last five years.
 This sector is now generating lakhs of new age jobs.
 Defence:

 Defence budget to cross Rs. 3, 00,000 crore for the first time in 2019-20.
 More than Rs 35,000 crore have been disbursed for OROP (One Rank One Pension).
 Railways budget:

5
http://www.edristi.in/
 Total capital support from budget proposed at Rs. 64, 587 crore in 2019-2020 (BE).
 Overall capital expenditure programme to be of Rs. 1, 58, 658 crore.
 Operating Ratio is expected to improve from 98.4% in 2017-18 to 96.2% in 2018-19 (RE)
and further to 95% in 2019-20 (BE).
 Safest year in history of railways since independence.
 All unmanned level crossings on broad gauge network eliminated.
 First Semi High Speed Vande Bahart Express launched which will provide Indian
Passengers a world class experience with speed, service and safety.
 Entertainment industry:

 Indian filmmakers to get access to Single Window Clearance for ease of shooting films.
 Regulatory provisions to rely more on self- declaration.
 To introduce anti-camcording provisioning in the Cinematographic Act.
 Vision for Next Decade:

 Foundation for India’s growth and development laid in the past 5 years.
 Poised to become a five trillion dollar economy in the next five years.
 Aspire to become a ten trillion dollar economy in the next 8 years thereafter.
 Ten dimensions of Vision for India of 2030

 India would be a modern, technology driven, high growth, equitable and transparent
society:

1. To build physical as well as social infrastructure and to provide ease of living.


2. To create a Digital India, digitize government processes with leaders from youth.
3. Making India pollution free by leading transport revolution with Electric Vehicles and
focus on Renewables.
4. Expanding rural industrialisation using modern digital technologies to generate massive
employment.
5. Clean Rivers, safe drinking water to all Indians and efficient use of water through micro-
irrigation.
6. Besides scaling up of Sagarmala, Coastline and Ocean waters powering India’s
development and growth.
7. Aim at our space programme – Gaganyaan, India becoming the launch-pad of satellites
for the World and placing an Indian astronaut into space by 2022.
8. Making India self-sufficient in food, exporting to the world to meet their food needs and
producing food in the most organic way.
9. A healthy India via Ayushman Bharat with women having equal rights and concern for
their safety and empowerment.
10. Transforming India into a Minimum Government Maximum Governance nation with pro-
active and responsible bureaucracy.

Links:
http://pib.nic.in/newsite/PrintRelease.aspx?relid=187915
http://pib.nic.in/newsite/PrintRelease.aspx?relid=187934
https://www.indiabudget.gov.in/ub2019-20/bs/bs.pdf
https://www.indiabudget.gov.in/ub2019-20/bag/bag1.pdf
https://www.indiabudget.gov.in/ub2019-20/bag/bag1.pdf
https://www.indiabudget.gov.in/ub2019-20/bag/bag5.pdf
https://economictimes.indiatimes.com/budget-faqs/what-is-an-interim-
budget/articleshow/67189292.cms
https://www.indiabudget.gov.in/ub2019-20/bag/bag2.pdf

6
http://www.edristi.in/
U.P. Budget 2019-20
Question: Consider the following statements regarding UP budget:
(1) Budget size for financial year 2019-20 is Rs 4, 79,701.10 Crore
(2) Under Uttar Pradesh Milk Policy, 5 crore have been allocated for various programmes in
financial year 2019- 20
(3) Total debt for state is approximately 30 percent of Gross Sate Domestic Product
(4) Total receipts for year 2019-20 is approximately Rs 4, 70,684 Crore
Of the above correct statements are:
(a) 1 and 2 only
(b) 2 and 3 only
(c) 1, 3 and 4 only
(d) All of the above
Answer: (d)
Related facts:

 Mr. Rajesh Agrawal, Finance minster of Uttar Pradesh presented the budget
on February 7, 2019 for financial year 2019-20.
 Budget size is approximately 4 lakh 79 thousand 701 Crore and 10 lakh rupees (Rs 4,
79,701.10 Crore), which is 12 per cent higher than the previous year.
 New schemes of worth Rs 21,212.95 Crore have been included.
 Per capita income has been increased to Rs. 5,500 in 2017-18 as compared
with 2016-17.
 Receipts:

 In financial year 2019-20 total estimated receipts is 4 lakh 70 thousand 684 crore and
48 lakh rupees (Rs 4, 70,684.48 Crore).
 Revenue receipts is 3 lakh 91 thousand 734 crore and 40 lakh rupees (Rs 3,
91,734.40 Crore).
 Capital receipts is 78 thousands 950 crore and 8 lakh rupees (Rs 78,950.08 Crore).
 Total tax revenue share in revenue receipts is 2 lakh 93 thousands 39 crore and 17
lakh rupees (Rs 2, 93,039.17 Crore).
 State’s own tax is Rs 1 lakh 40 thousand 176 Crore (Rs 1, 40,176 Crore).
 State’s share in central taxes is 1 lakh 52 thousand 863 crore and 17 lakh rupees
(Rs 1, 52,863.17 Crore).
 Expenditure:

 In financial year 2019-20 total estimated expenditure is 4 lakh 79 thousand 701


Croreand 10 lakh rupees (Rs 4, 79,701.10 Crore).

 Total revenue expenditure is 3 lakh 63 thousand 957 Crore and 04 lakh rupees (Rs
3, 63,957.04 Crore).
 Total capital expenditure is 1 lakh 15 thousand 744 Crore and 06 lakh rupees (Rs 1,
15,744.06 Crore).
 Revenue savings is estimated at Rs.27 thousand 777 Crore and 36 lakh rupees (Rs
27,777.36 Crore) in financial year 2019-20.
 Fiscal deficit:

 In financial year 2019-20, fiscal deficit is estimated at Rs. 46 thousand 910 Crore and
62 lakh rupees (Rs 46,910.62 Crore), which is 2.97 per cent gross state domestic
product.
 State’s total debt is approximately 30 percent of Gross Sate Domestic Product.
7
http://www.edristi.in/
 After subtracting total expenditure form receipts of consolidated fund of state, estimated
deficit is 9 thousand 16 crore and 62 lakh rupees (Rs 9,016.62 Crore).
 Net receipts estimated from public account is 9 thousand 500 Crore.
 Schemes:

 Agricultural and Rural development:

 Uttar Pradesh became the first state in the country to pay agricultural- grant through
DBT (Direct Benefit Transfer).
 In Mandi from now on only farmers will be president or vice-president.
 Unified license fee has been reduced to 10,000 from 1, 00,000.
 First branch of international rice research institute, Philippines, has been opened
in Varanasi.
 Soil Health Card is distributed to more than 4 Crore farmers.
 With the aim of doubling the farmers income “the million farmer’s school
programme” and International Agriculture Kumbh was successfully organized
in Lucknow.
 100 Mandis of Uttar Pradesh has been linked to portal e-NAM.
 State’s total sown areas area of sugar cane has been increased by 5 lakh hectares &
is approximately 28 lakh hectares.
 Rs 204 Crore is proposed for Modernization of Police Service System.
 Rs 25 crore is proposed for reoperation of shut sugar mills through PPP (Public
Private Partnership)model.
 Rs 892 crore has been proposed for Rashtriya Krishi Vikas Yojana.
 Rs 450 crore has been proposed for National Crop Insurance Programme.
 Rs 150 crore has been proposed for pre- storage of fertilizers.
 Rs 36 crore has been proposed for development of 13 Krishi Vigyan
Kendra underNational Agriculture Development scheme.
 Rs 247 crore and 60 lakhs has been proposed for construction of
Gaushalas and care for cows.
 Rs 200 crore has been proposed for Kanha Gaushala Evam Besahara Pashu
Aashrya Yojana in Urban areas.
 For Foundation of Agriculture University (Campus) construction is under
progressin Gonda.
 Rs 56 crore is proposed for dairy development in Mathura.
 Rs 64 crore has been proposed for Pt. Deen Dayal Upadhyaya laghu dairy
Yojanaunder which 10 thousands units are proposed to be constructed.
 In 2019-20, Rs 5 crore has been proposed for various programmes under Uttar
Pradesh milk policy, 2018.
 Rs 25 Crore is proposed for Pisciculture.
 Three new schemes under MGNREGA funded plantation have been started:

 Mukhyamantri Samudayik Vaniki Yojana


 Mukhyamantri Falodyan Yojana
 Mukhyamantri Krishak Vriksha Dhan Yojana
 Education:

 Uttar Pradesh open school (fee regulation) act, 2018 has been enacted
for regulation of private schools fees.
 There are 2 lakh 65 thousand schools in operation under basic education.
 149 Pt. deen dayal government model Inter-College is in operation since 2018-
19session.

8
http://www.edristi.in/
 94 Pt. deen dayal government Ashram schools are being operated for scheduled
caste and scheduled tribe.
 In the memory of Atal Bihari Vajpayee a provisional amount of Rs 5 Crore has been
allocated for foundation of Centre of Excellence in D. A. V. College, Kanpur.
 Under Samgra Shiksha Scheme Rs 18,485 Crore has been provisioned in the budget.
 For Mid day Meal Programme Rs 2,275 Crore has been provisioned in the budget.
 For Establishment of Sainik Schools Rs 26 Crore and 57 Lakh has been allocated.
 Rs 160 crore has been proposed for Rashtriya Ucchtar Siksha Abhiyan (RUSA).
 Rs 50 crore has been proposed for Wi-Fi services in all colleges and universities.
 A provision has been made to allocate Rs 63 lakh under infrastructural head for Gurushri
Gorakshanath Shodh Peeth at Deen Dayal Upadhyaya Gorakhpur University.
 Indian Institute of Information Technology (PPP Model) under construction.
 Engineering college is proposed in Mirzapur and Pratapgarh.
 For foundation of Atal Sushasan Peeth in Lucknow University Rs 2 Crore has been
provisioned in the budget.
 Housing:

 Under PM Awas Yojana –Grameen total 10 lakh and 9 thousand houses has been
built and provided to the beneficiaries.
 There is target of building 71 lakh Pakka houses in rural areas till the end of March,
2019.
 UP is the first place in making houses under the above scheme in financial year 2017-
18 (7 lakh 71 thousand).
 Infrastructure:

 Foundation stone of (Green field) Purvanchal Express-Way has been laid


from Lucknow to Ghazipur via Azamgarh.
 91 km Gorakhpur Link Expressway is proposed to be built in orderto connect
this Expressway with Gorakhpur.
 A 600 Km Express-Way connecting Meerut and Prayagraj has been planned.
 Electrical transmission has been raised form 92,556 MVA to 1, 14,157 MVA.
 Under Saubhagya scheme connection to 74 lakh needy families have been provided.
 Under UJALA scheme 3 core 40 lakh LED Bulb has
been distributed which save around 900 MW of power. While doing so Uttar
Pradesh secured second rank all over India.
 Vishwakarma Shram Samman Yojna has been proposed for Carpenter, tailor basket
weaver, goldsmith, blacksmith, potter, confectioner, barber, cobbler, masonry etc.
 Rs 100 crore has been proposed for Mukhyamantri Swa-Rojgar Yojana.
 Rs 175 crore has been proposed for both the Metro Rail Projects in Agra and Kanpur.
 Rs 150 crore each has been proposed for initial work of Varanasi, Meerut, Gorakhpur,
Prayagraj and Jhansi Metro Rail Project.
 Rs 1000 core is proposed for construction, expansion & consolidation of airstrip
development in the state.
 Rs 200 crore has been proposed for Uttar Pradesh Pratikaratmak Vanropan Nidhi
Prabandh Yojana.
 Bundelkhand development board has been established to address the inclusive
development of the area and address the issue of drought and regular natural
disaster.
 Purvanchal development board has been established to prepare an integrated
planning for socio-economic development of the region while notifying the obstacles
in achieving so.
 Noida international green field airport is under construction in Jewar, Gautam
Buddha Nagar.

9
http://www.edristi.in/
 Rs 3000 crore is proposed for pipe potable water in rural areas of Bundelkhand and
Vindhya region.
 Rs 224 crore has been proposed for Shyama Prasad Mukherjee Rurban Mission.
 Rs 250 crore has been proposed for “One District One Product”.
 Industry:

 MoU worth Rs. 4 lakh 68 thousand Crore have been invested by various world’s top
investors, industrialist in Uttar Pradesh.
 Business Reform Action Plan has been implemented through ease of doing
business.
 Health:

 For the first time 100 advance life support ambulance will be operated.
 2,329 sub centre and 750 PHCs (Primary Healthcare Centres) have been developed
as wellness centre.
 Under Ayushman Bharat National Health Protection Mission 6 core people of 1 core
18 lakh families has been given free health aid upto Rs 5 Lakhs annually through
government and listed private hospitals.
 Approximately 10 lakh 10 thousand beneficiaries have been given aid
through “Mukhyamantri Jan Aarogya Abhiyan”. For this scheme Rs 111 Crore have
been allocated.
 Atal Bihari Vajpayee Medical College is under construction in Lucknow.
 Rs 10 crore has been proposed for an Ayush University in the state.
 Governance:

 Average Response time for UP-100 has been reduced to 14 minutes.


 Home guards duty stipend has been raised to Rs. 500 per day from Rs. 375 per day and
total allocation for Homegaurds Welfare Fund has been increased to Rs 10 Crore.
 Vahan-4 and Sarthi-4 software has been installed in every transport offices for online
fee payment.
 Under PMJDY 4 core 87 lakh accounts have been opened for which 3 crore 85 lakh
accounts Rupay debit card have been issued.
 Under Mudra scheme 1 crore 18 lakh beneficiaries has been given credit worth Rs. 51
thousand 650 crore.
 UP became the second state to start online permit in every division.
 Bus service between Ayodhya and Janakpur (Nepal) has been started.
 Under GST (Goods and Services Tax) 7 lakh 75 thousand new registration has been
done till date.
 Annual turnover limit for GST registration has been raised to Rs 40 lakh from Rs 20
lakh and filing limit has been raised to three months for Rs 5 Crore.
 Under Smart City scheme Rs. 1,758 Crore have been approved for 10 cities.
 On Independence Day record 9 crore 66 lakh saplings have been planted.
 In 2019-20 there is a target of 22 crore plant saplings.
 Ganga Haritima Abhiyan has been run for integrated development of Ganga Gram.
 Women and child care:

 Under PM Ujjawala Yojana 1 crore 9 lakh free LPG connection has been given to
women of families lying below poverty line.
 Rs 156 crore has been proposed for Kishori Balika Yojana.
 Rs 335 crore for National Nutrition Mission and Rs 200 crore for Shabari Sankalp
Abhiyan has been proposed.
 Naya Savera Yojana has been started to curb the menace of child labour, under
which 18,376 child labors has been added.

10
http://www.edristi.in/
 Kanya Sumangla Yojna is proposed for changing a positive approach towards women.
For this scheme Rs 1200 Crore has been provisioned in the budget.
 Culture:

 During Kumbh Mela 2019 (1, 22,500) bio-digester toilet has been established.
 Kumbh has been declared as intangible cultural heritage by UNESCO.
 For the first time Akshay Vat and Saraswati Koop visit has been allowed in
the Kumbh.
 For the first time Pravasi Bhartiya Diwas has been organized in Varanasi in
between 21-23 January, 2019.
 Rs 125 crore has been proposed for infrastructure facility in Brij Tirth.
 Rs 101 crore has been proposed for integrated development of Ayodhya Tourism.
 Rs 27 crore has been proposed for development of Garh Mukteswar Tourism.
 Rs 70 crore for the implementation of tourism policy and 50 crore for pro-poor
tourism has been proposed.
 Rishi Bharadwaj Ashram and Shringwerpur dham development is proposed in
Prayagraj.
 Shakambhari Devi and Shukra taal development is proposed.
 Tulsi Peeth development is proposed in Rajapur chitrakot.
 In Prayagraj Saint Bharadwaj Ashram & Shringaverpur Dham has been proposed for
development.
 Maharaj Suheldev Sthal and Chitaura lake development is proposed in Bahraich.
 Lahartara Lake, kabir sthal and Seer Goverdhanpur, birth place of guru
Ravidas in Varanasi is proposed.
 Under buddha circuit development of Sarnath, Shravasti, Kushinagar, Kapilvastu,
Kaushambi and Sankisa is proposed.
 For foundation of Vaidik Vigyan Kendra in Banaras Hindu University Rs 16 Crore is
provisioned in the budget.
 Rejuvenation of Bijli Pasi Fort in Lucknow.
 Development of Vindhaychal and Naimisharanya is proposed.
 Rs 5 crore has been proposed for development of fencing of all Ram lila grounds.
 A provision of Rs 1 crore has been made to strengthen Vrindavan Shodh Sansthan.
 An auditorium is proposed between Mathura-Vrindavan.
 With the help of World Bank a pro-poor tourism Yojana is in operation in Agra and
Mathura region.

Links:

http://budget.up.nic.in/budgetbhashan/budgetbhashan2019_2020.pdf

Young Scientist Programme for School Students


Question: Which of the following Institute recently launched Young Scientist Programme
for School Students?
(a) ISRO (b) DRDO (c) TCS (d) Birla Group of Institute
Answer: (a)
Related facts:

 On 18th January, 2019, ISRO launched Young Scientist Programme.


 The aim of the Programme is to encourage the scientific talent and to enhance science
education in the country.
 Under this programme, ISRO will select 3 students from each State and Union Territory of
the country.

11
http://www.edristi.in/
 Mostly 8th standard passed out students will be selected for the one-month programme.
 During the month long programme students will visit ISRO centers, interact with senior
scientists and will have access to Research and Development facilities, ISRO will bear all
the expenses of travelling & boarding of the selected students.

Links:
https://indianexpress.com/article/education/isro-to-conduct-young-scientist-programme-
for-school-students-5544993/

Prime Minister visit of Kochi


Question: In which of the following states Prime Minister Narendra Modi has inaugurated
LPG bottling plant?
(a) Gujarat (b) Kerala (c) Maharashtra (d) Karnataka
Answer: (b)
Related facts:

 Prime Minister Narendra Modi visited Kochi on January 21, 2019 where he laid the
foundation of a petrochemical complex at the refinery and a skill development institute at
Ettumanoor.
 He also inaugurated a mounded storage vessel at the LPG bottling plant of the Indian Oil
Corporation Limited. It has a total storage capacity of 4350 MT.
 He also dedicated an integrated refinery expansion complex of the public sector Bharat
Petroleum Corporation limited at the Kochi Refinery to the nation.
 Equipped for production of cleaner fuels, the integrated refinery is a modern expansion
complex and would transform the Kochi Refinery as the largest PSU refinery with world
class standards in the country.
 It will double the production of LPG and diesel and commence production of feedstock for
petrochemical projects in the plant.
 LPG receipt through pipeline will bring down movement of LPG tankers on roads.
 Petrochemical complex, BPCL Kochi refinery are developed under Make in India initiative
aiming to reduce the dependence on imports.
 The skill development institute inaugurated at Ettumanoor will provide vocational training
and enhance employability and entrepreneurship for youth both in oil & gas and other
industries. It is setup in eight acre campus with the capacity to skill around 1,000 youths
annually in 20 different set of skills.

Links:
http://pib.nic.in/newsite/PrintRelease.aspx?relid=187784

Bharat Parv 2019


Question: Consider the following statements regarding Bharat Parv:
1) It’s a cultural event held annually, organized on the eve of republic day since 2016.
2) Ministry of tourism is the ministry responsible for the event.
Choose the correct option:
(a) 1 only (b) 2 only (c) Both 1 and 2 (d) Neither 1 nor 2
Answer: (c)
Related facts:

 The Bharat Parv event was organized by the Ministry of Tourism at the Red Fort, from
26th to 31st January, 2019 as a part of the Republic Day 2019 celebrations.

12
http://www.edristi.in/
 Bharat Parv has been organized annually since 2016, when India completed its 70 years
of independence.The prime objective of organizing the event to generate a patriotic mood,
promote the rich cultural diversity of the country and to ensure wider participation of the
general public.
 The major attractions this year included the replica of the statue of unity created by
sculptor Ram Vanji Suttar and also a Gandhi Gram in which 10 painting artists created
painting on the theme ‘Ideology of Mahatma Gandhi’.
 Another attraction this year was the stall on the ‘Adopt a Heritage’ project of the Tourism
Ministry, where the Monument Mitras are arranging Video walkthroughs/Project
awareness videos/Segway experience on the projects they have taken up under Adopt a
Heritage.
 The daily highlights of events were Display of Republic Day Parade Tableaux,
Performances by the Armed Forces Bands (Static as well as dynamic), a Multi-Cuisine
Food Court, Crafts Mela, Cultural Performances from different regions of the country,
exhibition-cum-sale of crafts items, Yoga demonstrations and live Kitchen
demonstrations.
 27 States participated by setting up of Stalls, Showcasing their food, handicrafts and
tourism products. Ministry of Culture, Ministry of AYUSH, Ministry of Information &
Broadcasting etc. also participated in the event.
 At the concluding ceremony, Shi K J Alphons said that everyone should travel and
promote tourism in the country.

Links:
http://pib.nic.in/newsite/PrintRelease.aspx?relid=187911

Shehri Samridhi Utsav focusing on Urban Livelihoods


Question: Shehri Samridhi Utsav focusing on Urban Livelihoods was organized by which
of the following Ministry?
(a)Ministry of Housing and Urban Poverty Alleviation
(b) Ministry of Commerce and Industry
(c) Ministry of Consumer Affairs, Food and Public
Distribution
(d) Ministry of Civil Aviation
Answer: (a)
Related facts:

 Shehri Samridhi Utsav, focusing on urban livelihoods was organised all over the country
by Ministry of Housing and Urban Poverty Alleviation, from 1st -15th February 2019.
 The event aims at extending the outreach of National Urban Livelihoods Mission (DAY-
NULM) to the poorest of the poor and the most vulnerable, showcase its initiatives and
facilitate access of SHG members to the other government schemes.
 Deen Dayal Antyodaya Yojana (DAY) aims to uplift the poor folks by enhancing
sustainable livelihood opportunities through skill development.
 Deen Dayal Antyodaya Yojana integrates National Urban Livelihoods Mission (NULM)
(urban component of the mission implemented by the Ministry of Housing and Urban
Poverty alleviation) and National Rural Livelihoods Mission (NRLM) (rural component of
the mission implemented by the Ministry of Rural Development).

Links:
http://pib.nic.in/newsite/PrintRelease.aspx?relid=187582

13
http://www.edristi.in/
Defence Minister dedicates Diffo Bridge to the nation
Question: Union Defence Minister Nirmala Sitharaman inaugurated Diffo bridge in which
of the following state?
(a) Arunachal Pradesh (b) Manipur (c) Andhra Pradesh (d) Manipur
Answer: (a)
Related facts:

 Union Defence Minister Nirmala Sitharaman inaugurated the


426.60 m long Pre Stressed Concrete Box Girder type Diffo
Bridge over Diffo River at km 654.950 on Roing – Koronu – Paya
road in Arunachal Pradesh.
 Dedicating the bridge to the nation, the Raksha Mantri praised the commendable work
being done by Project Udayak, Border Roads Organization (BRO) in this region.
 Defence Minister appreciated completion of this prestigious bridge which would provide
uninterrupted access between Dibang valley and Lohit valley region of Eastern Arunachal
Pradesh and an all-weather Road to the troops deployed on the China Border.

Links:
http://arunachal24.in/arunachal-nirmala-sitaraman-dedicates-diffo-bridge-to-nation/

Pakistan shares list of nuclear installation with India


Question: Recently which country shares list of nuclear installation with India?
(a) Pakistan (b) Nepal (c) Bhutan (d) Sri lanka
Answer: (a)
Related facts:

 On 1st Jan 2019, Pakistan shared with India a list of its


nuclear installations and facilities in conformity with
Article-II of the Agreement on Prohibition of Attacks
against Nuclear Installations and Facilities between India & Pakistan.
 The list of nuclear installations and facilities in Pakistan was officially submitted to a
representative of the Indian High Commission at the Ministry of Foreign Affairs.
 Whereas Ministry of External Affairs in New Delhi submitted the list of Indian Nuclear
Installations and Facilities to a representative of the High Commission of Pakistan.
 The agreement was signed on December 31, 1988 and it came into force on January
27th, 1997.
 According to the agreement two countries must inform each other of nuclear installations
and facilities to be covered as per agreement on 1st January every year and it is in
regular practice since 1992 despite of many low bilateral ties.

Links:
https://www.indiatoday.in/world/story/pakistan-shares-with-india-list-of-nuclear-
installations-1421103-2019-01-01

National Declamation Contest on Patriotism and Nation Building


Question: Who inaugurated National Declamation Contest on Patriotism and Nation
Building?
(a) Col. Rajyavardhan Rathore (b) Dr. Harshvardhan Bajpai
(c) Nirmala Sitharaman (d) Nitin Gadkari

14
http://www.edristi.in/
Answer: (a)
Related facts:

 Minister of State for Youth Affairs & Sports (Independent Charge) Col. Rajyavardhan
Rathore delivered the inaugural address at two days National Declamation Contest
(24th& 25th January) organized by Nehru Yuva Kendra Sangathan (NYKS) with the
support of Department of Youth Affairs. NYKS has been organizing National Level
Declamation Contest since 2015-16.
 The aim of the National Declamation Contest is to strengthen the spirit of Nationalism and
Patriotic feeling amongst youth and masses for wholehearted participation in nation
building.
 The contest will also identify youth with leadership qualities and good communication
skills for their holistic development and empowerment. It will enable them to take lead in
understanding the functions and policies of the government towards nation building.
 He notified that India is a country of diversities, and therefore symbols like National Flag
and National Anthem are necessary to inculcate feeling of belongingness and patriotism,
our Constitution entitles us to certain rights, but there are duties also which are to be
followed by all Indians.

Links:
http://pib.nic.in/newsite/PrintRelease.aspx?relid=187722

One-City-One-Operator
Question: Union Minister Shri Nitin Gadkari laid foundation stone for Namami Gange
project in which cities?
(a) Mathura & Agra (b) Varanasi & Prayagraj (c) Prayagraj & Kanpur (d) Kanpur & Agra
Answer: (a)
Related facts:

 On 23 January 2019, Union Minister for Water Resources, River


Development and Ganga Rejuvenation, Road Transport &
Highways and Shipping Shri Nitin Gadkari laid the foundation
stones of six Namami Gange projects in Agra and Mathura.
 The four projects in Mathura have a sanctioned cost of Rs 511.74
crore.These include two sewerage projects.
 The first is an integrated sewerage infrastructure for the city on Hybrid Annuity Mode with
‘One-City-One-Operator’ approach and reuse of treated sewage water in Mathura refinery
at a sanctioned cost of Rs 460.45 crore.
 The second, rehabilitation of sewerage infrastructure and up-gradation of STP at
Vrindavan at a sanctioned cost of Rs 33.82 crore.
 There is also a project for cleaning of 27 Ghats at a sanctioned cost of Rs 3.60 crore.
 For abatement of Industrial Pollution, there will be a project for up-gradation of
infrastructure of existing CETP for Textile Printing units at the Mathura Industrial Area at a
sanctioned cost of Rs 13.87 Crore.
 The projects in Agra include comprehensive Sewerage Scheme for the city with ‘One-
City-One-Operator’ approach on Hybrid Annuity Mode for creating new STPs of 176 MLD,
up-gradation, rehabilitation of old plants and O&M for 15 years at a sanctioned cost of Rs
857.26 Crore.
 Construction of Sewerage Network in Western Zone, for laying of 251 Km sewer lines,
47,827 house connections, rising main of 3.80 Km under AMRUT Scheme at a
sanctioned cost of Rs 353.57 Crore.

15
http://www.edristi.in/
Links:
http://pib.nic.in/newsite/PrintRelease.aspx?relid=187674

Government launched Technology Mission Centre on Solar Energy &


Water Treatment
Question: Government launched Technology Mission Centre on Solar Energy & Water
Treatment in which city?
(a) Delhi (b) Mumbai (c) Kolkata (d) Chennai
Answer: (d)
Related facts:

 Union Minister Dr. Harsh Vardhan launched three important


centre’s set up by Department of Science and Technology
(DST), nucleated at Indian Institute of Technology, Madras (IITM) on 25th January 2019
in Chennai.
 The first of these is the establishment of DST –IITM Solar Energy Harnessing Centre.
 The center will focus on a wide range of research and technology development activities
such as silicon solar cells that promise high efficiency and are suited for Indian conditions.
 The intention is to generate a platform,that can be enlarged readily to reinforce the
knowledge ecosystem.
 Second in line is the DST-IITM Water – IC for Sutram of Easy Water which has been
organized with an objective to commence synchronized research and training programs
on various issues related to wastewater management, water treatment, storm water
management, distribution and collection systems.
 The third one would be the Test bed on Solar thermal desalination solutions which are
being established by IIT Madras and Empereal KGDS as solution providers in
Naripaaiyur, Ramanathapuram District, Tamil Nadu with the objective to deliver
customized technological solutions to deal with common water challenges in the arid
coastal village located on the shores of the Bay of Bengal.

LINKS:
http://pib.nic.in/newsite/PrintRelease.aspx?relid=187717

National Swachh Iconic Places


Question: 5th National Swachh Iconic Places review happened in which city?
(a) Delhi (b) Mumbai (c) Kolkata (d) Chennai
Answer: (b)
Related facts:

 The Ministry of Drinking Water and Sanitation and State Government agencies, local
administrations and the Trusts/ Boards of the Swachh Iconic Places (SIPs) came together
for a two-day consultation in Mumbai, represented by over 100 delegates from across the
country.
 SIP is an initiative of Ministry of Drinking Water and Sanitation under Swachh Bharat
Mission and was launched in year 2016.
 The Swachh Iconic Places initiative aims to achieve a
distinctly higher level of sanitation at these places, especially
in the peripheries and approach areas.
 During the consultation, a review was also done of the work
already taken up at the 20 Phase-I and Phase-II SIPs.

16
http://www.edristi.in/
 Phase I iconic places are: Ajmer Sharif Dargah, CST Mumbai, Kamakhya Temple, Golden
Temple, Maikarnika Ghat, Meenakshi Temple, Shri Mata Vaishno Devi, Shree Jagannath
Temple, Taj Mahal and Tirumala Tirupati Devasthanam Temple.
 Phase II of Swachh Iconic Places: Gangotri Temple, Yamunotri Temple, Charminar,
Shree Mahakaleshwar Temple, Convent and Church of St. Francis of Assissi, Kalady,
Gommateswara Statue, BaidyanathTemple, Gaya Teerth and Somnath Temple.
 Phase III of Swachh Iconic Places: Sri Raghvendra Swamy Mutt Mantralayam,
Hazarduari Palace,Shri Sharveshwar Mahadev Temple, Vidur Kuti Temple, Mana Village,
Pangong Tso, Shri Nag Vasuki Temple, Ema Keithel, Shree Dharma Sastha Temple,
Baidyanath Temple and Kanvashram.

Links:
http://pib.nic.in/newsite/PrintRelease.aspx?relid=187553

2nd World Orange Festival


Question: 2nd World Orange Festival organized in?
(a) Nagpur (b) Kanpur (c) Mumbai (d) Delhi
Answer: (a)
Related facts:

 The second World Orange Festival was organised in Nagpur


from 18th January to 21th January 2019.
 Strong business angle focusing on exhibitions, seminars and
the participation of orange farmers from 10 other states in the
country was the key focus of festival.
 The festival, while offering its famed oranges to the world,
showcased and branded the exquisite fruit in order to boost
its export.
 Agricultural experts and scientists from various countries participated in the workshops
and guided farmers.
 The main objective of World Orange Festival is to encourage and empower the hard work
done by farmers to produce oranges of the best quality and to buoy the identity of the
‘Orange City’ to the world.

Links:
https://www.worldorangefestival.com/

The Future of Rail Report


Question: The Future of Rail Report which was launched by Piyush Goyal is prepared by
which organisation?
(a) Indian Railways (b) Ministry of Railway (c) Delhi Metro Rail Corporation
(d) International Energy Agency
Answer: (d)
Related facts:

 Minister of Railways & Coal Mr. Piyush Goyal launched “The Future of
Rail” report of International Energy Agency (IEA) at an event on 30 January, 2019.
 The IEA prepared the report titled as “The Future of Rail” in along with the International
Union of Railways (UIC).
 The indigenously produced Vande Bharat Express in India is an engineering marvel that
will change the approach to train manufacturing as well as revolutionise train travel in
India.
17
http://www.edristi.in/
 Indian Railway’s successful effort to convert diesel locomotives to electric locomotives is
truly innovative. The Railway is moving towards 100% electrification for the Indian
Railways.
 It was also informed that 5 years ago, Railways had electrified about 600 km of tracks
across the country. Last year alone, we electrified over 4,000 km, and in the coming year
we aim to electrify over 6,000 Km.

The report:

 The “Future of Rail” the first-of-a-kind report analyses the current and future importance of
rail around the world through the perspective of its energy and environmental implications.
The report reviews the impact of existing plans and regulations on the future of rail, and
explores the key policies that could help to realise an enhanced future rail.
 By 2050, Investments in Indian urban rail infrastructure will reach nearly $190bn. Along
with the development of high-speed rail, fuel expenditures are estimated to reduce by
nearly $450bn. India can save up to $64bn on fuel expenses by that time in result.
 The report had a special focus on India. The rail passenger traffic increased nearly 200%
and freight service by 150% since 2000 in India. The future prospects of rail services in
the country continue to remain bright.
 The report stated that the rail sector carries 8% of passengers across the world and 7% of
global freight movement. However, it utilises 2% of the total transport energy demand in
the world, signifying its efficiency.
 Global activity in 2050 will be 2.7 times higher than current levels.

International Energy Agency:

 The IEA is a Paris based inter-governmental organisation founded in 1974 that works to
ensure reliable, affordable and clean energy for its 30 member countries and 8
association countries.
 Its mission is guided by four main areas of focus: energy security, economic development,
environmental awareness and engagement worldwide.

Link:
https://www.iea.org/futureofrail/#

20th Bharat rang Mahotsav


Question: Where will the 20th Bharat Rang Mahotsav be organised?
(a) Pune (b) Bengaluru (c) New Delhi (d) Vadodara
Answer: (c)
Related facts:

 The 20th Bharat Rang Mahotsav (BRM) will be organised in New Delhi from1-21
February. This 21 day long Mahotsav is the Asia’s largest theatre festival which is
organised by National School of Drama (NSD).
 This festival will have 111 shows and various allied events such
as Director`s Meet, Living Legend and Mater Class.
 The festival will be inaugurated in Kamani auditorium after which
play Karanth ke Rang will be performed which is directed by
Amod Bhatt.
 This festival will also have four plays dedicated the Mahatma
Gandhi in the row of celebrating his 150th birth anniversary.
 The festival will host 69 Indian and 15 foreign plays. This includes plays of Hindi, English
and other regional languages.
18
http://www.edristi.in/
 A Theatre Bazaar will also be established where the stall will be selling a range of
products and culinary dishes.

Links:
http://www.uniindia.com/-20th-bharat-rang-mahotsav-to-commence-from-feb-
1/india/news/1482507.html

Yuva Swabhiman Yojana


Question: Which of the following state recently launched Yuva Swabhiman Yojna?
(a) Madhya Pradesh (b) Bihar (c) Uttar Pradesh (d) Rajasthan
Answer: (a)
Related facts:

 On the Republic day, Kamal Nath, Chief Minister (CM) of


Madhya Pradesh, announced the launch of the Yuva
Swabhiman Yojana for the youths from the economically
weaker sections in urban areas in Chhindwara.
 Under this scheme, 100 days of employment will be provided to youths from economically
weaker sections in urban areas.
 Mr Kamal Nath informed that from April 1st, social security pension will be raised to Rs
600, from Rs 300 to Rs 600 and every year the amount will be raised and by the end of
5th year, it will reach Rs 1000.
 The Yuva Swabhiman Yojana has been designed as a public welfare enterprise by linking
temporary employment and skill development. The state’s future lies in the power of
youthfulness there is no dearth of talents in youngsters; there is a need to enrich their
abilities.
 The venture will be executed in the near future following cabinet approval.

Links:
https://www.freepressjournal.in/bhopal/cm-announces-yuva-swabhiman-yojana-for-poor-
urban-youth/1445873

Pravasi dividend pension scheme launched


Question: In which of the following state Pravasi dividend pension scheme will be
launched?
(a) Maharashtra (b) Karnataka (c) Kerala (d) Telangana
Answer: (c)
Related facts:

 On 25 January 2019, Governor P. Sathasivam announced that the Kerala government will
soon launch a ‘Pravasi dividend pension scheme’ so as to provide regular pension to
Nonresident Keralites on a one-time payment of Rs.5 lakh.
 The onetime payment of Rs 5 lakh will be deposited with KIIFB
(Kerala infrastructure investment fund board) for financing
infrastructure projects.
 According to a study, there are 2.1 million migrants from Kerala
across the world whose estimated total remittances to the state has
been found to be Rs. 85, 092 crore. Out Of these, close to 90 % are in the Middle East.
 Governor P. Sathasivan mentioned that the state would set up a call centre with an
international Toll free line so as to provide information, register complaints and details
about action taken on various issues faced by the NRIs.

19
http://www.edristi.in/
 A comprehensive interactive portal will also be launched to provide information on job
opportunities in various countries.
 Lok Kerala Sabha (LKS) was also established with the aim of using the expertise, talent
and resources of NRIs form Kerala for the overall growth and development of the state.
 The LKS was supposed to offer a meaningful say to NRI community in implementing
schemes for building a new Kerala.

Links:
https://www.business-standard.com/article/news-ians/kerala-to-launch-nri-pension-
scheme-119012500716_1.html

Museum of Indian Cinema


Question: In which of the following cities Museum of Indian cinema has been
inaugurated?
(a) Mumbai (b) New Delhi (c) Kolkata (d) Chennai
Answer: (a)
Related facts:

 Prime Minister Narendra Modi has inaugurated the national museum of Indian cinema
(NMIC) in Mumbai on 19th January 2019.
 The state of art Museum, built at a cost of Rs. 140.61 crore, aims to take its visitors
through an absorbing journey of over a century of Indian Cinema in a story telling mode
with the help of visuals, graphics, artifacts, interactive exhibits and multimedia
expositions.
 The creation of the museum has been guided by the Museum Advisory committee
headed by Shyam Benegal.
 Prasoon Joshi headed an innovation committee in order to provide an upgrade to NMIC.
 The museum is housed in two buildings – the New Museum Building and the 19th century
historic Palace Gulshan Mahal – in the films division campus in Mumbai.
 Gulshan Mahal is an ASI grade-2 heritage structure which has been restored as a part of
the NMIC Project.
 It is divided into 9 sections namely The origin of Cinema, Cinema comes to India, India
Silent Film, Advent of Sound , the studio Era, the impact of second world war, creative
resonance, new wave and beyond and regional cinema.

Links:
http://ssmb.in/national-museum-of-indian/

ASER 2018

Question: Which of the following non-profit organization has been facilitating the ASER
2018 Report?
(a) PRAYAS (b) PRATHAM (c) PAHAL (d) NONE OF THE
ABOVE
Answer: (b)
Related facts:

 ASER 2018 was launched by government of India with the cooperation of non-profit
organization PRATHAM on January 15, 2019.
 ASER 2018 has covered 596 districts in rural India, a total of 354,944 households and
546,527 children in the age group 3 to 16 were surveyed.

20
http://www.edristi.in/
 There are 573 partner institutions, 820 master trainers and approximately 30,000
volunteers have taken part in the survey.
 Survey has been done on the basis of HOW, WHAT and WHO parameters.

ASER findings:

 Overall enrollment of children in the age group (6-14) has been above 95%.
 The proportion of children (age 6-14) who are not enrolled in school has fallen below 3%
for the first time and stands at 2.8% in 2018.
 In 2018, the overall proportion of girls in the 11 to 14 age group out of school has fallen to
4.1%. This figure is more than 5% in only 4 states.
 The percentage of out of school girls aged 11-14 has fallen significantly over time.
 Average teacher attendance has been around 85% and average student attendance at
around 72% for past several years in both primary and upper primary schools.
 Nationally in 2018, 4 out of 10 government primary schools visited had less than 60
students enrolled.
 The fractions of schools with girls toilet has been doubled to 66.4%.
 In 2018 survey, about 8 out of 10 schools had a playground available for students, either
within premises or nearby.
 Physical education teacher are scarce in rural India.
 More than half of children enrolled in class 5 can read at least a standard 2 level text.
 The percentage of all children in standard 3 who can read at least at standard 2 levels
has increased slowly over the past years.
 Enrollment in private schools has been increased from 2006 to 2014 but since then it is
plateaued at around 30% for the age group 6-14.

ASER (Annual Status of Education Report):

 It is an annual survey that aims to obtain reliable estimates of children’s schooling and
basic learning levels for each state and rural district in India & to measure the change in
these basic learning and school statistics over time.
 It is the largest citizen-led survey in India, and is also the only annual source of
information on children’s learning outcomes available in India today.
 This report has been published annually since 2005 except the year 2015.

Pratham

 It is an innovative learning organization created to improve the quality of education in


India.
 As one of the largest non-governmental organisations in the country, Pratham focuses on
high-quality, low-cost and replicable interventions to address gaps in the education
system.
 It was established in 1995 to provide education to children in the slums of Mumbai;
Pratham has grown both in scope and geographical coverage.

Links:
http://www.pratham.org/about-us/about-pratham

Providing affordable medical facilities to people living in rural areas


Question: ‘Ayushman Bharat’ aims to build how many healthcare and wellness centres
that provides insurance coverage and comprehensive medical facilities to people living
in rural areas?

21
http://www.edristi.in/
(a) 1,50,000 (b) 2,00,000 (c) 3,00,000 (d) 50,000
Answer: (a)
Related facts:

 With an objective of medical security to people, ‘Ayushman Bharat’ aims to provide


insurance coverage and comprehensive, need-based healthcare through 1,50,000 health
and wellness centre to 10 crore poor and vulnerable families.
 To improve life expectancy, reduce maternal and infant mortality rates and improve
quality of life, adequate healthcare & wellness centres need to be set up all over the
country. Emphasizing in this regard Vice President Venkaiah Naidu also mentioned that a
healthy mind flourishes in a healthy body.
 The doctor patient ratio is less than 1:1000 as per the norms prescribed by the WHO
which is one of the major challenges faced by medical institutions with primary challenges
being a double burden of diseases-infectious ailments as well as an increased incidence
in life style diseases like diabetes, high blood pressure, obesity, heart disease and
cancers.
 As per The Lancet Global Health, Cardiovascular diseases, respiratory diseases, and
diabetes kill around 4 million Indians annually (as in 2016). It is estimated that 62 per cent
deaths among men and 52 per cent among women are due to non-communicable
diseases.

Links:
http://pib.nic.in/newsite/PrintRelease.aspx?relid=187198

Genetically Modified (GM) Crops


Question: In year 2017-18, with adoption of Bt. Cotton the improved productivity of
cotton was?
(a) 477 kg/hectare (b) 500 kg/hectare (c) 800 kg/hectare (d) 1000 kg/hectare
Answer: (a)
Related facts:

 Minister of State for Ministry of Agriculture & Farmers Welfare


Parshottam Rupala notified that the productivity of cotton
increased from 191 kg per hectare in 2002-03 to 477 kg per
hectare in 2017-18 due to adoption of Bt. cotton in the Country.
 This has also increased the production of cotton from 86.21 lakh
bales in 2002-03 to 348.88 lakh bales in 2017-18. There is a well established regulatory
framework for approval of Genetically Modified (GM) Crops under the Environment
(Protection) Act, 1986 in the Country.
 Evaluation of each application of GM crop is done on a case-to-case basis after a
thorough examination of health, environment, food and feed safety assessment studies
undertaken in a systematic and scientific manner as per set of rules stipulated by various
regulatory agencies under the Rules, 1989 from time to time.

GM Crops:

 Genetically Modified crops are crops whose DNA are modified using genetic engineering
methods which most of the time introduces a new trait to the plant which does not occur
naturally. It is used to provide resistance to certain diseases, pests etc. which leads to
increase in the harvest of crops.

Links:
http://www.pib.nic.in/Pressreleaseshare.aspx?PRID=1559075
22
http://www.edristi.in/
Employment for Divyangs
Question: What is the percentage allocation of number of reserved candidates for
employment in Government & private sectors as per Rights of Persons with Disabilities
Act, 2016?
(a) 5% (b) Not Less than 4% (c) 2% (d) 1%
Answer: (b)
Related facts:

 The Central Government has implemented National Action Plan


for Skill Development with the objective to enhancing
employability for Divyangs (persons with disabilities) in government & private sectors.
 Accordingly, Rights of Persons with Disabilities Act, 2016 mandates reservation under
Section 34(1) for persons with benchmark disabilities in Government Establishments is
not less than 4% of the total number of vacancies. This is applicable to all establishments
irrespective of areas of works
 Further, the Government also implements incentive scheme for providing employment to
persons with disabilities in the Private Sectors.
 Government has also set up 42 Special Employment Exchanges for Handicapped, 38
Special Cells for Handicapped persons in the normal Employment Exchanges, and 21
National Career Service Centres for Differently Abled in the country.

Links:
http://pib.nic.in/newsite/PrintRelease.aspx?relid=187221

Outreach event on Accomplishment and Way Forward for Textile Sector


Question: As per Vice President M. Venkaiah Naidu addressing an outreach event Indian
textile industry is expected to reach US$ 223 billion by which year?
(a) 2022 (b) 2021 (c) 2023 (d) 2024
Answer: (b)
Related facts:

 On 6 January 2019, Vice President M. Venkaiah Naidu addressed an outreach event on


‘Accomplishment and Way Forward for Textile Sector’. He emphasized on to develop a
culture of innovation, wide range of products and newer markets need to be invoked for
growth of textile industry.
 Indian Textile Industry is expected to reach US$ 223 billion by 2021 for which the
availability of raw materials, low cost and skilled manpower is an added advantage.
 Vice President also presented Threads of Excellence Awards to various organizations
and individuals for showing their excellence in the textile industry.
 Indian Textile Industry is facing stiff competition from Bangladesh
and Vietnam. Innovation and value addition needs to be focussed
for improving global competitiveness of the Indian textiles and
apparels. The government has allowed 100 percent FDI in textile
sector and Technology Upgradation Fund Scheme to accelerate
the industry`s growth.
 The textiles sector is the second largest industry after agriculture
in terms of economic contribution and employment generation. Currently over 10 crore
people are employed directly or indirectly in the Textile Industry in which Women account
for 70 per cent of the total workforce employed in the garmenting sector.

Links:
http://pib.nic.in/newsite/PrintRelease.aspx?relid=187214
23
http://www.edristi.in/
Prime Minister Gujarat Visit
Question: Prime Minister inaugurated a gun factory in which of the following place in
Gujarat?
(a) Dahej (b) Vadodara (c) Hazira (d) Sabarmati
Answer: (c)
Related facts:

 Prime Minister was on a three day visit to the state of Gujarat.


The visit was from 17th January to 19th January, 2019 where
PM inaugurated various developmental projects in the state.
 Prime Minister inaugurated Vibrant Gujarat Global Summit 2019 on 18th January in
Gandhinagar. Earlier he inaugurated the Vibrant Gujarat Trade Show in Gandhinagar.
 He also inaugurated Sardar Vallabhbhai Patel Institute of Medical Sciences and Research
in Ahmedabad, a super-specialty public hospital built by Ahmedabad Municipal
Corporation having all modern amenities, including an air ambulance.
 He also inaugurated Ahmedabad Shopping Festival, first ever event of its kind. This will
provide an opportunity for city enterprises to showcase their products.

Hazira Gun Factory:

 Prime Minister visited Hazira in Surat where he inaugurated the Hazira Gun (howitzer)
Factory. It is built by Larsen & Tubro (L&T) and is countries first private factory in its field.
 It will manufacture K9 Vajra self propelled Howitzer guns under Make in India initiative.
Earlier L&T won the contract worth Rs. 4,500 crore by Ministry of Defence for the supply
of 100 units of K9 Vajra-T 155 mm/52 calibre tracked self propelled gun system to the
Indian Army.
 This factory will also manufacture and integrate advanced armed platforms like future
ready combat vehicles, infantry combat vehicles and main battle tanks.
 The Indian Army inducted two howitzers including one K9 after a gap of 3 decades. K9
Vajra is built by L&T in collaboration with a South Korean firm Hanwha Corporation..

K9 Vajra:

 The K9 VAJRA-T is a variant of K9 Thunder known to be the world’s best 155mm/52


Cal.SPH in terms of number of systems in active service. Currently, K9 Thunder is serving
as one of the main conventional deterrent forces for a number of armies including the
Korean Army.
 K9 Thunder works effectively in extreme weather conditions, jungles, barren deserts and
severely cold areas.

Link:
https://indianexpress.com/article/india/pm-modi-ministry-of-defence-private-weapon-
manufacturing-5546091/

World’s Top Container Ports


Question: Which Port of India is included in World`s top 30 container ports?
(a) Jawaharlal Nehru Port Trust (b) Cochin Port (c) Panambur Port (d) Chennai Port
Answer: (a)
Related facts:

 As per latest Lloyds Report,Jawaharlal Nehru Port Trust


(JNPT) is the only Indian port that got listed in the Top 30
24
http://www.edristi.in/
container ports globally. JNPT improved its ranking by 5 positions reaching to 28th spot in
the latest ranking.
 Many efforts and strategic initiatives are being implemented at JNPT in order to enhance
overall port efficiency. Various new processes activated under the ‘ease of doing
business’ initiative have not only helped in overall growth of the port business, but also
allowed the EXIM trade to save time and cost which in turn have accentuated the growth
story.
 The JNPT has also launched their App service to facilitate better tracking of consignment
and ease the trade process for their EXIM partners. The App will allow traders to access
all the relevant information about their consignment and port related updates regarding
traffic and weather and will keep updating the relevant information at regular intervals.
 Some key projects are also being initiated in the port like on-going project work on the
mega 4th Terminal, developing a centralized Parking Plaza, improving the port connecting
roads and widening of Highways and development of Coastal Berth.
 The Port is also implementing, specific processes at the operational level to make it
standardized and more efficient across terminals, among many other initiatives in the
pipeline.
 JNPT has been credited for helping India leverage its position in the World Bank Ranking
in trading across the borders, from 146 to 80, a jump of 66 points.
 JNPT closed 2018 on a high note crossing the 5 Million TEUs traffic milestone and in
December 2018 alone JNPT handled 4.45 Lakh TEUs which again is the highest volume
of container handled recorded in a single month, at the port.

Lloyd`s list of one Hundred Ports, 2018

 The list is released by Lloyd showing the ranking of various ports around the globe on the
basis of the volume of trade done from the ports. The volume of trade in ports has grown
by 6% as compared to previous year.
 This year also saw the dominance of Chinese ports having the highest and busiest
volume of trade.
 Shanghai is ranked first in the list while Singapore, Shenzhen and Ningbo-Zhoushan are
ranked as 2nd, 3rd and 4th respectively all from China.

Links:
http://pib.nic.in/newsite/PrintRelease.aspx?relid=187678

State Visit of Prime Minister of Norway to India


Question: Which state PM came at Raisina dialogue held in New Delhi, India?
(a) Canada (b) England (c) France (d) Norway
Answer (d)
Related facts:

 Prime Minister of Norway, Erna Solberg was on a State visit to


India from 7-9 January, 2019 at the invitation of Prime Minister
Shri Narendra Modi.
 Solberg delivered the Inaugural address at the Raisina Dialogue. She also addressed
India-Norway Business Summit.
 India and Norway enjoys close and multifaceted ties including economic and technical
cooperation.
 More than 100 Norwegian companies have invested in India in areas such as ship-
building, petroleum related services, hydropower, clean energy and IT services.
 Several major Indian companies are also present in Norway. India and Norway have
shared interest in sustainable use of ocean resources for growth of our economies.
25
http://www.edristi.in/
Visit of South African President
Question: Consider the following statements in reference to the visit of South African
President to India and choose the correct option:
1) This was the first visit of South African President Cyril Ramaphosa to India after he
took office.
2) He became the second South African President after Jacob Zuma to be the chief guest
of Republic Day Celebration.
3) He also delivered lecture at the first IBSA Gandhi- Mandela Freedom Lecture.
Correct option:
(a) 1 and 3 (b) 2 and 3 (c) 1 and 2 (d) All of the above
Answer: (a)
Related facts:

 The President of the Republic of South Africa Matamela Cyril Ramaphosa was on visit to
India on 25-26 January, 2019. This was the first state visit to India by him after he took the
office of President. He was also the Chief Guest of 70th Republic Day celebration of India
on 26 January 2019. He became the second president after Nelson Mandela to be the
chief guest of Republic Day celebration.
 He also delivered the first the Indian Council for World Affairs instituted First IBSA (India
Brazil South Africa) Gandhi Mandela Memorial Freedom Lecture in Delhi. President Ram
Nath Kovind held a banquet in his honour at Rashtrapati Bhawan.
 Prime Minister Modi and President Ramaphosa held delegation level talks at Hyderabad
House on January 25, 2019. The two leaders held discussions in the spirit of the strategic
partnership, strong friendship and historical links between the two countries.
 Both leaders acknowledged the significance of the 100th birth anniversary celebrations of
Nelson Mandela and the 150th birth anniversary celebrations of Mahatma Gandhi as an
invaluable legacy of peace, non-violence and compassion.
 President Ramaphosa and Prime Minister Modi jointly addressed the India-South Africa
Business Forum on January 25, 2019, with a focus to grow business ties between the two
countries.During the visit, both the parties agreed to cooperate, share best practices,
technology and expertise on the Ease of Doing Business Reform Programme. Both are
committed to expand cooperation in the fields of trade and investments between business
entities in South Africa and India.
 They also agreed to cooperate in the field of Small and Medium Enterprises (SMEs)
which play an invaluable role in job creation and creating trade and investment
opportunities and also to enhance cooperation in the field of the Oceans Economy and to
cooperate in multilateral forums on the Blue Economy including in the framework of Indian
Ocean Rim Association (IORA).

Strategic Partnership:

 Both the parties decided to update the Strategic Partnership by agreeing on a


comprehensive roadmap. This is also called the Red Fort Declaration which was signed
by the then Indian PM H.D. Deve Gowda and South African President Nelson Mandela in
March 1997.
 A Three-Year Strategic Programme of Cooperation (2019-2021) was signed by the two
sides in which the partnership between both the countries in all the aspects would be
enhanced like security cooperation, trade, tourism etc.

Links:
https://www.mea.gov.in/bilateral-
documents.htm?dtl/30952/IndiaSouth+Africa+Joint+Statement+during+State+Visit+of+Pr
esident+of+South+Africa

26
http://www.edristi.in/
Ministry of Women & Child Development – Year End Review 2018
Question: Which scheme is launched by Central Government to provide cash incentive
amounting to Rs. 5,000 to women during pregnancy and lactation period?
(a) Pradhan Mantri Matri Vandana Yojana (b) Pradhan Mantri Matru Vandana Yojana
(c) Pradhan Mantri Matri Shishu Vandana Yojana
(d) Pradhan Mantri Matru Shishu Vandana Yojana
Answer: (b)
Related facts:

 The Prime Minister on 31st December, 2016, announced Pan-Indian implementation of


maternity benefits for pregnant women and lactating mother. The programme is known as
Pradhan Mantri Matru Vandana Yojana which provides Direct Benefit Transfer of Rs.
5,000 in three stages to eligible beneficiaries.
 Beti Bachao Beti Padhao is one the flagship programmes of the Government with focus
on improving the efficiency of welfare services intended for girls. 161 BBBP districts for
the time period between April-March, 2015-16 & 2016-17 indicating that, an improving
trend in Sex Ratio at Birth (SRB) is visible in 104 districts. The girl’s enrolment in
secondary education has increased to 80.97% against 76% in 2013-14. Construction of
girl’s toilets in every school in selected districts has been achieved.

Initiatives regarding Women welfare:

 One Stop Center; so far, 279 OSCs have become operational. These centres have
offered support to over 1.93 Lakh women.
 Women Helpline; so far, women help lines have become operational in 32 States/UTs.
They have managed more than 20.23 lakh calls of women.
 The Sexual Harassment at Workplace (Prevention, Prohibition and Redressal) Act, 2013;
Students, apprentices, laborers, domestic workers and even women visiting an officer are
included in the Act.
 Grievance Redressal Cell; The cell has processed close to 39,347 complaints in the one
year since its inception.
 Swadhar Greh; The Ministry has constructed Krishna Kutir at Vrindavan, Mathura of UP of
1,000 beds for their safe stay health services, nutritious food, legal and counseling
services to widows.

Children welfare initiative:

 POSHAN Abhiyaan aims to reduce mal-nutrition from the Country in a phased manner,
through the life cycle concept, by adopting a synergized and result oriented approach.
Target of Abhiyaan is to bring down stunting of the children in the age group of 0-6 years
from 38.4% to 25% by the year 2022. More than 10 crore people will be benefitted by this
programme. 315 districts were covered in Financial Year 2017-18.

Other major provisions:

 The ‘Women of India Festival’ is organised annually by the Ministry with the aim to
encourage women entrepreneurs and farmers, and also actively promote organic
products ranging from food and fabric to wellness and personal care. The total sales by
the women farmers and entrepreneurs who came from 26 States were a record of over
Rs. 2.75 crore. Participants can enroll themselves in Mahila e-Haat, which is an online
marketing portal set-up by the Ministry to meet the aspirations and needs of women
entrepreneurs.

27
http://www.edristi.in/
 Rashtriya Mahila Kosh (RMK) is a society and an apex micro-finance organization
established in 1993 under the aegis of Ministry of Women & Child Development (MWCD)
to meet the credit needs of poor and assetless women in the informal sector for various
livelihood support and income generating activities at concessional terms. RMK’s Corpus
is at Rs 284 crore as on 27th December, 2018.
 Mahila Shakti Kendra Scheme is a centrally sponsored scheme for implementation from
2017-18 to 2019-20 to empower rural women through community participation. It aims to
provide an interface for rural women to approach the government for availing their
entitlements and also empowering them through training and capacity building. The
scheme has been approved for implementation in 23 States /UTs.
 Joint guidelines for construction of 2 lakh Anganwadi Centre (AWC) buildings in most
backward districts by 2019 under MGNREGS in convergence with ICDS Scheme have
been jointly issued by Ministry of Rural Development and WCD to fulfill the shortage of
AWC buildings (around 4.5 lakhs) in the country.
 The Government has, recently, enhanced honorarium to Anganwadi Workers from ₹
3,000/- to ₹ 4,500/- per month; AWWs at mini-AWCs from ₹ 2,250/- to ₹ 3,500/- per
month; AWHs from ₹ 1,500/- to ₹ 2,250/- per month; and introduced performance linked
incentive of ₹ 250/- per month to AWHs effective from 1st October, 2018.
 The Ministry of WCD has been giving awards to the meritorious children and Individuals
/Institutions for several decades. The scheme has been revamped in 2018 to make it
more broad based and inclusive. The National Child Award, now called the Pradhan
Mantri Rashtriya Baal Puraskar is to be given in two categories: Baal Shakti Puraskar and
Baal Kalyan Puraskar.
 National Institute of Public Cooperation and Child Development, popularly known as
NIPCCD, an autonomous organisation under the aegis of the Ministry of Women and
Child Development, Government of India is a premier organization devoted to promotion
of voluntary action and research, training and documentation in the overall domain of
women and child development. It aims to develop and promote voluntary action in social
development; take a comprehensive view of child development and to promote and
develop relevant need-based programmes in pursuance of the National Policy for
Children.

Links:
http://pib.nic.in/newsite/PrintRelease.aspx?relid=187560

One Family One Job Scheme


Question: Which State has launched ‘One Family One Job’ Scheme at a “Rojgar Mela”
(employment fair)?
(a) Sikkim (b) Telangana (c) Maharashtra (d) Goa
Answer: (a)
Related facts

 On 14th January 2019,Sikkim Chief Minister Pawan Kumar Chamling launched the “One
Family One Job scheme” in Gangtok.
 The scheme envisions employment to a member of every family in the State who does
not have a government job in the state.
 At a “Rojgar Mela” (employment fair) organized at the Paljor stadium, about 20,000
unemployed youths were given temporary appointment letters.
 In order of its implementation,the longest serving Chief Minister in independent India
Pawan Kumar Chamling, personally handed out temporary appointment letters to two
beneficiaries from each of the 32 Assembly constituencies in the state during the Rojgar
Mela.

28
http://www.edristi.in/
 At present recruitments are being made for Group C and Group D posts in 12 government
departments.
 In its emphasizing action plan,unregularized government employees would also be
regularized over the period of next five years.
 Under this scheme, all loan debts in the farming and agriculture sector would be revoked.

Links:
https://timesofindia.indiatimes.com/city/kolkata/sikkim-cm-launches-one-family-one-job-
scheme/articleshow/67500365.cms

PM released Commemorative Coin of Rs 350


Question: PM released commemorative coin of Rs 350 to mark 350th birth anniversary of
which Sikh guru in January 2019?
(a) Guru Govind Singh (b) Guru Teg Bahadur (c) Guru Angad Dev (d) Guru Arjun Dev
Answer: (a)
Related facts:

 On 13 January 2019, Prime Minister Narendra Modi released a


commemorative coin of Rs 350 on Guru Gobind Singh in New Delhi.
 The Tenth Sikh Guru – Guru Gobind Singh has been a source of inspiration for many
through his teachings and ideals. He lauded the lofty ideals and values of the Guru
Gobind Singh Ji – the selfless service to humanity, devotion, heroism and sacrifice and
urged the people to follow his path.
 Recalling Guru Gobind Singh’s fight for the weaker sections, the Prime Minister in his
Mann Ki Baat radio program broadcast on 30th December, 2018 said that Guru Gobind
Singh Ji believed that the biggest service is to alleviate human sufferings.

Links:
http://pib.nic.in/newsite/PrintRelease.aspx?relid=187456

4th Edition of Raisina dialogue


Question: What was the theme of the 4th edition of Raisina Dialogue 2019 organised in
New Delhi?
(a) A World Reorder: New Geometries; Fluid Partnerships; Uncertain Outcomes
(b) The New Normal: Multilateralism with Multipolarity
(c) Asia: Regional and Global Connectivity (d) None of the above
Answer: (a)
Related facts:

 Ministry of External Affairs in partnership with Observer Research Foundation has


organized the 4th edition of the Raisina Dialogue in New Delhi from 8-10 January 2019.
 The Raisina Dialogue is India’s flagship annual geopolitical and geostrategic conference.
 It was inaugurated by Prime Minister of Norway Ms Erna Solberg in the presence of Prime
Minister Shri Narendra Modi and External Affairs Minister Smt. Sushma Swaraj.
 The theme of the dialogue was “A World Reorder: New Geometries; Fluid Partnerships;
Uncertain Outcomes”.
 The Dialogue, which has grown steadily in scope and scale since the first edition in
2015,brought together over 600 delegates from 93 countries, including influential political
leaders, strategic thinkers, policy practitioners, technology innovators, business
representatives and academics on a common platform where they offered their vision and
propose solutions that will serve us all in the decades ahead.

29
http://www.edristi.in/
Links:
https://www.mea.gov.in/press-
releases.htm?dtl/30888/4th+Raisina+Dialogue+January+810+2019

Release of Recombinant ELISA kits


Question: Among the Recently released Recombinant Enzyme-linked immune sorbent
assay (ELISA) kits, one for Glanders and other will be used to diagnose-
(a) Gonorrhea (b) HIV (c) Equine Infectious Anaemia (d) Leukaemia
Answer: (c)
Related facts:

 On 9 January 2019,Union Minister of Agriculture & Farmers Welfare Shri Radha Mohan
Singh released recombinant Enzyme-linked immune sorbent assay (ELISA) kits: one for
Glanders and other for Equine Infectious Anaemia(EIA).
 Both kits are highly economical as compare to imported kits and will prove to be a
milestone in the control and eradication of these notifiable diseases from India.

Glanders & EIA:

 Glanders is a fatal infectious and notifiable disease of equines including horses, donkeys
and mules.The disease is caused by a bacterium known as Burkholderia mallei and has
zoonotic potential.
 The organism is also considered as potential bio-weapon and categorized under ‘Tier 1
Select Agent’.
 After continuous research efforts of more than eight years, National Research Centre on
Equines (NRCE) has been able to develop a recombinant Hcp1 antigen ELISA as an
alternate to complement-fixation test (CFT).
 The ELISA has been duly validated in India and the OIE Reference Laboratory, Germany
and showed excellent sensitivity (97.2%) and specificity (99.6%).
 This technology has been transferred to eight State Disease Diagnostic Laboratories
following approval of DADF, Ministry of Agriculture & Family Welfare and commercialized
for transformation into ready to use kit.
 Equine infectious anaemia (EIA) is a chronic, debilitating and persistent infectious
disease of equines caused by a retrovirus. It is an OIE notifiable disease.NRCE has also
developed recombinant p26 protein-based ELISA as an alternative to Coggin’s test.
 This technology will provide sustainable and homogeneous source of antigen and
harmonized protocol to ensure regular surveillance of EIA.

Links:

http://pib.nic.in/newsite/PrintRelease.aspx?relid=187374

Pravasi Teerth Darshan Yojna


Question: Consider the following statements regarding Pravasi Teerth Darshan Yojna:

1. It was launched by the Prime Minister in January 2019

2. Under this scheme People of Indian-Origin (PIO) will be taken on a govt. sponsored tour
of religious places twice a year

3. People belonging to Girmitiya Countries will be given first preference

30
http://www.edristi.in/
Of the above correct statement/s is/are:
(a) Only 1 (b) Only 2 (c) Only 3 (d) All of the above
Answer:(d)
Related facts:

 On 22nd January 2019, Prime Minister Narendra Modi launched


the Pravasi Teerth Darshan Yojna.
 Under this scheme a group of Indian diaspora will be taken on
a govt. sponsored tour of religious places in India twice a year.
 The first batch of 40 people of Indian origin will be selected on Pravasi Bhartiya Divas.
The selected people will be taken to religious places of all major religions in India.
 It is noteworthy that 15th Pravasi Bhartiya Divas convention was held in Varanasi from
21st Jan – 23rd Jan 2019.
 People of Indian – origin aged 45 to 65 can apply and a group will be selected out of them
with first preference given to people form ‘Girmitiya Countries’.
 Girmitiya Countries are countries including Mauritius, Fiji, Suriname, Guyana, Trinidad
and Tobago, and Jamaica.

Links:

https://timesofindia.indiatimes.com/india/pm-launches-pravasi-teerth-darshan-
yojana/articleshow/67647538.cms

Foundation and Inauguration of National Highway Projects


Question: Who laid foundation and inauguration of 39 National Highway Projects in
Andhra Pradesh?
(a) Nitin Gadkari (b) Narendra Modi (c) Kiren Rijiju (d) Venkaiah Naidu
Answer: (a)
Related facts:

 Union Minister for Road Transport & Highways Nitin Gadkari inaugurated and laid the
foundation stone of 39 National Highways Projects in Andhra Pradesh on 21stJanuary,
2019.
 These include construction of 11 ROBs under Setu Bhartam programme. These projects
have a length of 1384 kms, and involve investment of Rs 16,878 crore.
 The total length of National Highways in Andhra Pradesh has increased from 4193 km in
2014 to 7246 km in 2018.
 Overall investment in Andhra Pradesh is Rs 1,00,000 crore for development of National
Highways. About 2,520 Km of roads amounting to Rs 44,000 crore have been envisaged
under Bharatmala. Under Setu Bhartam, 31 ROBs have been approved with cost of about
Rs 2,500 crore. Out of these, 18 ROBs amounting to Rs 1,310 crore have been
sanctioned which are at construction/award stage.
 In addition, 136 CRF works for total length of 1,805 km amounting to Rs 1,769 crore have
been sanctioned since May 2014 in Andhra Pradesh. Out of these, 56 works for total
length of 835 km amounting to Rs 570 crore have been completed.
 A roadmap has been developed for increasing Vizag Port capacity to 211 MMTPA from
existing 152 MMTPA by 2025 by government.

Links:

http://pib.nic.in/PressReleaseIframePage.aspx?PRID=1560803

31
http://www.edristi.in/
MoU signed for Inter-Ministerial Cooperation for Promotion and
Facilitation of Agricultural Biotechnology Research and Education
Question: Recently the Department of Biotechnology (DBT) and which of the following
organisation has entered into Memorandum of Understanding (MoU) for Inter-Ministerial
Cooperation for Promotion and Facilitation of Agricultural Biotechnology Research and
Education?
(a) ICAR (b) CIAE (c) NCF (d) CACP
Answer: (a)
Related facts:

 On 15 January 2019,Department of Biotechnology (DBT)


and ICAR have entered into Memorandum of Understanding (MoU) for Inter-Ministerial
Cooperation for Promotion and Facilitation of Agricultural Biotechnology Research and
Education.
 The MoU between ICAR and DBT is for mutual collaboration to explore the possibility of
cooperation, convergence and synergy to promote and accelerate the progress of
research and training in various disciplines of agricultural biotechnology between ICAR
and DBT.
 The MoU will be implemented with the aim to collaborate with one another in mutually
agreed-upon research programmes in the areas of agricultural biotechnology, funding of
projects, policy issues, regulatory aspects, and other specified areas of National interest.
 The Department of Biotechnology (DBT) has undertaken several activities and programs
to promote Biotechnology Research & Development in the areas of Agriculture
Biotechnology.
 The Indian Council of Agricultural Research (ICAR) has been coordinating, guiding and
managing research and education in Agriculture including horticulture, fisheries and
animal sciences in the entire country.

Objectives of the partenership:

 Establishing National platform/centers for services related to genomics, genotyping, data


banking, agriculture bioinformatics, GM food detection, validation of technologies such as
vaccines, diagnostics of veterinary/ fisheries use, molecular markers in crop and animal
breeding.
 Formulating and introducing courses/training programmes in the area of IPR, biosafety,
biodiversity conservation and germ plasm exchange, genomic selection and breeding for
faculty and students through appropriate mechanisms.
 Putting forward research prioritization with other ministries and foreign collation like Indo-
UK SIC, BRICS, African countries and ‘Look East’ programme in relation to agricultural
biotechnology.

Links:

http://pib.nic.in/newsite/PrintRelease.aspx?relid=187504

BES Expo 2019


Question: BES Expo 2019 was hosted by which of the following city?
(a) Mumbai (b) Pune (c) Nasik (d) New Delhi
Answer: (d)
Related facts:

32
http://www.edristi.in/
 25th International Conference and Exhibition on Terrestrial and Satellite Broadcasting,
Broadcast Engineering Society Expo (BES Expo) 2019 took place in New Delhi at Pragati
Maidan from 17 -19 January.
 BES Expo 2019 was organised by Broadcast Engineering Society (India)
and endorsed by DRM UK and supported by Prasar Bharati, Ministry of
Communications & IT and others. This expo is assumed to be the biggest
broadcast technology show in India.
 It was inaugurated by Information and Broadcasting Secretary Amit Khare and TRAI
(Telecom Regulatory Authority of India) Chairman R. S. Sharma.
 The theme of the conference was ‘Next Gen Broadcasting in the IT World’.
 An exhibitor’s directory was released by the dignitaries and BES awards were given to the
recipients. This grand expo received participation of around 300 companies from 25
countries who displayed the latest equipment for radio and TV broadcasting equipments.
Over 1000 delegates attended the event.

Link:
http://www.newsonair.com/Main-News-Details.aspx?id=358227

Agri Vision 2019


Question: Consider the following statements in reference to the Agri Vision 2019:
a) The theme of the event was Envisioning Agro Solutions for Smart and Sustainable
Agriculture
b) This two day event took place in Hyderabad
c) The event was inaugurated by Vice President M. Venkaiah Naidu
d) Agriculture sector contribute for 18% of GDP and almost 50% employment to total
workforce
Choose the correct option:
(a) 1, 2 and 3 (b) 1, 3 and 4 (c) 2, 3 and 4 (d) All of the above
Answer: (d)
Related facts:

 Vice President M. Venkaiah Naidu inaugurated Agri Vision


2019 on 17th January, 2019 at Hyderabad. This is a two-day conference on Envisioning
Agro Solutions for Smart and Sustainable Agriculture.
 t was jointly organised by Prof. Jayashankar Telangana State University, CII and Cornell
University, USA. The two day conference had 8 sessions and 44 speakers coming from
various public and private institutions, industries and also experts from USA and farmers
participated in the deliberations held on various subjects.
 The sessions held on topics like Combationg Climate Change: Advances in crop breeding
and Natural Resource Management (NRM), AI based Agriculture, Economics behind
enhancing Farmers income, technology in Agri Entrepreneurship and many others.
 Long term solutions for the issue of farming sector which contributes majorly to the
prosperity of farmers and make them financially sufficient were also discussed.
 Other critical issues such as growing challenge of declining productivity, diminishing and
degrading natural resources, rapidly growing demand for food, stagnating farm incomes,
fragmented land holdings etc. that poses threat were reviewed to tackle it in effective
manner in future.
 It is worth to mention that Agriculture sector accounts for 18 % of India’s GDP while it
provides employment to 50 % of the workforce of the country. This shows the irrational
productivity of farming sector and low income generation.

Links:
http://www.uniindia.com/two-day-agri-vision-concludes-in-hyderabad/south/news/1472194.html

33
http://www.edristi.in/
First India Central Asia Dialogue
Question: Which country will host the first India- Central Asia Dialogue?
(a) Turkmenistan (b) Kazakhstan (c) Uzbekistan (d) Kyrgyzstan
Answer: (c)
Related facts:

 Central Asian country Uzbekistan will host the first the First India- Central Asia Dialogue
in the city of Samarkand.
 This will take place on January 12 and 13, 2019 and will be co-chaired by Foreign
Minister Sushma Swaraj and Foreign Minister of Uzbekistan Abdulaziz Kamilov.
 The Foreign Minister of Afghanistan will participate as a special invitee in a session
dedicated to connectivity issue in the region. Foreign Minister of the Kyrgyz Republic,
Tajikistan and Turkmenistan and the First Deputy Foreign Minister of Kazakhstan will also
represent their respective countries.
 This initiative is an important steps in promoting cooperation seeing the shared history
and cultural heritage of India and Central Asia and will explore ways to substantially
enhance Indias involvement in business and development sector in Central
Asia.
 This dialogue came after the landmark visit of PM Narendra Modi to all
five Central Asian countries in 2015 and Foreign Ministers Central Asia tour
in August 2018. This dialogue, along with participation of Afghanistan, will strengthen
India`s engagement with Central Asia across all sectors.

Link:
https://www.mea.gov.in/press-
releases.htm?dtl/30897/Visit_of_External_Affairs_Minister_to_Samarkand_Uzbekistan_for_the_
First_IndiaCentral_Asia_Dialogue_January_1213_2019

India US 2+2 Inter-Sessional Meeting


Question: Where did the recent India – US 2+2 Inter-Sessional Meeting took place?
(a) New York (b) New Delhi (c) Tokyo (d) Washington DC
Answer: (b)
Related facts:

 A 2+2 Inter-Sessional meeting between India and US took place in


New Delhi on January 11, 2019.
 The Inter-Sessional meeting primarily focussed on the progress of
defence and foreign policy issues and bilateral cooperation between the two countries.
The meeting was set to follow up on the decisions made during the India-US Ministerial
2+2 Dialogue whose first round took place in New Delhi in September 2018.
 The delegation of India was jointly led by Joint Secretary (AMS) from the Ministry of
External Affairs Gourangalal Das, and Joint Secretary (PIC) from the Ministry of Defence,
Shambhu S Kumaran.
 The US delegation was led jointly by Principal Deputy Assistant Secretary of State Alice
Wells and Assistant Secretary of Defence Randall Schriver.

Links:
https://www.aninews.in/news/national/general-news/india-us-hold-22-inter-sessional-meeting-in-
new-delhi201901112107590002/

34
http://www.edristi.in/
124th Constitutional Amendment Bill
Question: Which of the following statements are correct in reference to the 124th
Constitutional amendment passed by both the houses of Parliament?
1) This bill provisions 10% reservation in government jobs to forward caste poor people
2) This reservation would add to the existing 49.50% reservation and would further
increase it to 60%
3) The Bill amends the Article 15 & 16 of constitution
4) Gujarat becomes the first state to approve its implementation
Correct option:
(a) 1, 2 and 3 (b) 1, 2 and 4 (c) 2, 3 and 4 (d) All of the above
Answer: (c)
Related facts:

 Parliament passed the historic 124th Constitutional Amendment Bill that guarantees 10%
quota reservation to Economically Weaker Section of forward caste in higher education
and government jobs.
 This provides reservation to people of General category having annual income of less
than 8 lakhs per annum per family, having less than 5 acre of agricultural land.
 People who have a house less than 1,000 sq feet in a town (or 100 sq yard in a notified
municipal area) will also avail benefits under this act.

Important dates:

 The bill needed a special majority to get passed in both the Houses. This bill was brought
to the table of Lok Sabha on 8th January and got passed the same day with majority (323
in favour, 3 in against).
 The following day it was brought in Rajya Sabha, last day of winter session after an
extension of 1 day. It passed too the same day (165 in favour and 7 against).
 The President gave his assent to the bill on 12th January validating the bill. This will now
be known as Constitution (103rd Amendment) Act, 2019.
 Gujarat, the next day (13th January) became the first state to approve the implementation
of 10% quota reservation.

Constitutional aspect of the bill:

 The Act amends Article 15 and 16 of Constitution as it adds a new clause which allows
states to make “special provision for the advancement of any economically weaker
sections of citizens”.
 Article 15 has prohibition of discrimination on grounds of religion, race, caste, sex or place
of birth. Article 16 has equality of opportunity in matters of public employment.

Judgements of Courts:

 The judgement of Supreme Court in 1992 case of Indra Sawhney & others vs Union Of
India says the provision of reservation should not be more than 50%.
 The reservation of 69% in Tamil Nadu is protected by Ninth Schedule barring Judicial
Review.

Historical background:

 Government of Kerala under Chief Minister VS Achyutanand in 2008 brought 10%


reservation of seats in graduation and PG courses in government colleges and 7.5%

35
http://www.edristi.in/
seats in universities for the economically backward among the forwards. An appeal is
pending in Supreme Court.
 In 2008 and 2015, the Rajasthan Assembly passed Bills to provide a 14% quota to the
economically backward classes (EBCs) among the forward castes.

Links:
https://indianexpress.com/article/explained/reservation-bill-mandal-commission-quota-narendra-
modi-5527623/

10% reservation implementation in educational institutions


Question: Which of the following statements are correct in reference to Economically
Weaker Section (EWS) reservation implementation?
1) The proposed reservation will be applied in educational institution from 2019-20
academic year.
2) This will be applied in both government and private educational institutions.
3) This will be applied only in higher education.
Choose the correct option:
(a) 1 and 2 (b) 1 and 3 (c) 2 and 3 (d) All of the above
Answer: (d)
Related facts:

 The 124th Constitutional Amendment Bill for reservation of Economically Weaker Section
in general category will be applicable to educational sector (both public and private
institution) from academic year 2019-20.
 A gazette notification signed by the Joint Secretary in the Ministry of Social Justice and
Empowerment regarding the same was released on 14th January 2019 affirming this as
the date on which the provisions of the 103rd Amendment Act,2019 came into force.
 It is worth to mention that the bill for the amendment in the Constitution regarding the
EWS reservation was passed by the Parliament on January 9th 2019.
 This will reform the admission process and seat allotment in around 40,000 colleges and
900 universities. This quota will be over and above the existing quotas for Scheduled
Castes, Scheduled Tribes and Other Backward Castes.

Links:
https://www.timesnownews.com/india/article/reservation-for-general-category-in-india-news-
gujarat-ministry-of-social-justice-and-empowerment-notification-implementation-10-per-cent-
quota-for/347530

6th Women of India Organic Festival


Question: The 6th Women of India Organic Festival was held with which of the following
aim?
(a) To encourage organic foods among sick women
(b) To provide organic fruits & dairy products amongst women farmers
(c) To encourage Indian women entrepreneurs and farmers to connect with more buyers across
the country (d) All of the above
Answer: (c)
Related facts:

 The Ministry of Women and Child Development organized the


6th Women of India Organic Festival with an aim to encourage
Indian women entrepreneurs and farmers to connect with more buyers across the country
and empower them with financial inclusion.
36
http://www.edristi.in/
 This event took place at Leisure Valley, Sector 10, Chandigarh from 12th to 14th January
2019. This festival has been held annually in New Delhi since 2015 and was celebrated in
Chandigarh for the first time.
 Over 200 women participated in the festival which showcased over 1,000 varieties of
organic products including fabric, wellness, grains, seeds, jewellery, bakery items and a
lot more.
 The Union Minister, Maneka Gandhi as well as MP of Chandigarh Kirrron Kher also
interacted with the participating women farmers and entrepreneurs who put 72 stalls of
various farming and organic products and 15 food stalls in food court. Visitors showed
their keen interest to purchase various products exhibited in the festival.
 It was mentioned by the minister that government is stressing on the needs for promoting
organic farming throughout the country for better, healthy and environment friendly
lifestyle especially encouraging women farmers and entrepreneurs with a well-knit
marketing infrastructure for organic farm products.

Links:
http://pib.nic.in/newsite/PrintRelease.aspx?relid=187483

Three New AIIMS approved


Question: Which of the following districts are selected for building three new AIIMS?
(1) Samba (2) Rajkot (3) Pulwama (4) Gorakhpur
Correct option:
(a) 1, 2 and 3 (b) 1, 2 and 4 (c) 2, 3 and 4 (d) 1, 3 and 4
Answer: (a)
Related facts:

 On 10 January 2019, Union Cabinet under the chairmanship of Prime Minister Narendra
Modi approved the setting up of three new AIIMS (All India Institute of Medical Sciences).
Two of them will be in Samba and Pulwama district of Jammu & Kashmir while one will be
in Rajkot district of Gujarat.
 The AIIMS will be set up in Vijaynagar, Samba district of Jammu & Kashmir at a cost of
Rs. 1,661 crore; Awantipura, Pulwama district of Jammu & Kashmir at a cost of Rs 1828
Crore and Rajkot, Gujarat at a cost of Rs 1195 crore.
 Setting up new AIIMS in the states will lead to employment generation for nearly 3000
people in various faculty & non faculty posts in each of the AIIMS. This will also generate
various indirect employments in the area.
 Each AIIMS will have 750 beds catering around 1500 OPD patients and around 1000 IPD
patients per month. It was also stated that each new AIIMS will add 100 UG (MBBS)
seats and 60 B.Sc (Nursing) seats.

Links:
http://pib.nic.in/PressReleseDetail.aspx?PRID=1559476

Sittwe Port
Question: In which of the following country Sittwe Port, developed with the help of India,
is located?
(a) Bangladesh (b) Myanmar (c) Sri Lanka (d) Iran
Answer: (b)
Related facts:

 In a written reply to a question in Rajya Sabha, Minister of state for Shipping, Road
transport and chemical & fertilizer Mansukh L Mandaviya notified that Sittwe Port in
37
http://www.edristi.in/
Myanmar, constructed with India’s assistance, is ready for operation as the infrastructure
there is fully developed.
 The construction of Sittwe Port is part of the Kaladan multi-modal transit transport project.
 The main purpose of this project is to create a multi-modal sea, river and road transport
corridor for shipment of cargo from the eastern ports of India to Myanmar through Sittwe
Port as well as North Eastern part of India via Myanmar.
 India and Myanmar signed a bilateral Memorandum of Understanding (MoU) on 22nd
October, 2018 for operationalisation of the port at Sittwe and Inland Water Transport
(IWT) Terminals at Sittwe and Paletwa.
 Once fully operational, the project would encourage investment and trade and also open
up alternate routes for connectivity to India’s north-east region.

Links:
https://economictimes.indiatimes.com/news/defence/india-all-set-to-take-over-ops-in-myanmars-
sittwe-port-after-chabahar/articleshow/67437859.cms

Government approval for setting up Indian Institute of Skills


Question: Which of the following city is not included among the three cities selected for
setting up of Indian Institute of Skills (IISs), recently approved by Ministry of Skill
Development and Entrepreneurship?
(a) Kanpur (b) Mumbai (c) Chennai (d) Ahmedabad
Answer: (c)
Related facts:

 On 7 January 2019, Ministry of Skill Development and Entrepreneurship has approved the
proposal to set up Indian Institute of Skills (IISs) in 3 locations viz. Kanpur, Mumbai and
Ahmedabad.
 The IISs are conceptualized with an objective to provide advanced skills in highly
specialized areas – for example defence, aerospace, oil & gas and other emerging
business domains.
 These institutes are expected to emerge as premier training institutions in the country and
make vocational training aspirational for the youth.
 Government of India will establish state of the art training institutions referred as Indian
Institutes of Skill (IISs) in Mumbai, Ahmedabad and Kanpur on Not-for-profit Public
Private Partnership (N-PPP) basis.

Links:
http://pib.nic.in/newsite/PrintRelease.aspx?relid=187254

Gandhian Resurgence Summit in Prayagraj


Question: Kumbh Mela 2019 is being organised at which of the following place in India?
(a) Ujjain (b) Haridwar (c) Prayagraj (d) Nasik
Answer: (c)
Related facts:

 On 17th January 2019 President of India, Shri Ram Nath Kovind


inaugurated the Gandhian Resurgence Summit organised from 17-20 January 2019 by
the Parmarath Niketan, the Harijan Sevak Sangh and the Global Interfaith WASH
Alliance.
 About 300 Gandhians from different parts of the country and organisations participated in
the Summit.Earlier,First Lady Ms Savita Kovind honoured 9 women from traditional boat
community for their contribution to the Kumbh.
38
http://www.edristi.in/
 It is worth to mention that Prayagraj Kumbh 2019 holds a emphasizing accomplishment of
opening of doors to Akshaya Vat (sacred tree) and Saraswati Koop (sacred well) for
common men after 450 years.

Links:
http://www.pib.nic.in/Pressreleaseshare.aspx?PRID=1560147

World Book Fair in New Delhi


Question: Who inaugurated the World Book Fair in New Delhi?
(a) Narendra Modi (b) Ram Nath Kovind (c) Smriti Irani (d) Prakash Javadekar
Answer: (d)
Related facts:

 The New Delhi World Book Fair was inaugurated by Human


Resource & Development Minister Prakash Javadekar. The fair
was organized by National Book Trust, India at Pragati Maidan
from 5- 13 January, 2019.
 This was the 27th edition of 9 day long annual event. The theme of book fair was
‘Readers with Special Needs’.
 Over 20 countries took part in this fair. Sharjah was recognized as the guest of honour
country and had shown the rich and dynamic culture and literature of UAE. There was a
special book exhibition on and by Mahatma Gandhi and his associates to mark his 150th
birth anniversary.
 The New Delhi World Book Fair is one of the oldest book fairs of India. It started in 1972
and was inaugurated by then President V.V. Giri.

Links:
http://www.newsonair.com/Main-News-Details.aspx?id=357561

Renukaji Multipurpose Dam Project


Question: Consider the following statements in reference to Renukaji Multipurpose Dam
Project:
1) An agreement for this Dam project is signed between 6 states
2) This would lead to construction of three storage projects
3) The projects would be constructed on River Ganga and its
tributaries
4) Renukaji Dam project has been conceived as a storage project on
Giri River
Correct option:
(a) 1, 2 and 3 (b) 1, 3 and 4 (c) 1, 2 and 4 (d) 2, 3 and 4
Answer: (c)
Related facts:

 An agreement for Renukaji Multipurpose Dam Project was signed in New Delhi in the
presence of Union Water Resource Minister Nitin Gadkari on January 11 between six
states namely Uttar Pradesh, Haryana, Himachal Pradesh, Delhi, Rajasthan and
Uttarakhand. Chief Ministers of all six states were present during the signature.

Renukaji Multipurpose Dam Project:

 Under this project, proposal is for the construction of three storage projects on river
Yamuna and two of its tributaries; Tons and Giri in the hilly regions of Uttarakhand and
39
http://www.edristi.in/
Himachal Pradesh of Upper Yamuna Basin. These include Lakhwar project on river
Yamuna in Uttarakhand, Kishau on river Tons in Uttarakhand and Himachal Pradesh and
Renukaji on river Giri in Himachal Pradesh.
 These three projects were identifies as National Projects in 2008 whose cost would be
borne by Centre and beneficiary state as 90% and 10% respectively.
 Renukaji Dam project has been conceived as a storage project on Giri River in Sirmour
district of Himachal Pradesh envisaging construction of 148 mtr. high rock filled dam to
supply 23 cumec water to Delhi and other basin states.
 This project will also generate 40 MW of power during peak flow. This project is proposed
to be executed by Himachal Pradesh Power Corporation Ltd. (HPPCL).
 The live storage of the dam is 0.404 MAF and total submergence area of 1508 hectares in
the territory of Himachal Pradesh. The flow of Giri River will increase about 110% after the
construction which will meet the drinking needs of Delhi & other basin states.
 The total projected cost of the project on Price Level 2015 is Rs. 4596.76 crores out of
which the cost of irrigation/drinking water component is Rs. 4325.43 crores and the cost
of power component is Rs. 277.33 crore.
 Stored water of Renukaji Dam will be used by UP, Haryana & NCT of Delhi from
Hathnikund Barrage, by NCT of Delhi from Wazirabad Barrage and by UP, Haryana and
Rajasthan from Okhla Barrage.

Projects for Concession Agreement:

 The trans-Ganga/Yamuna areas in Prayagraj do not have any sewage treatment facility
which leads to pollution of rivers. Two projects for sewage management have been
sanctioned in the trans-Ganga/Yamuna areas and Operation & Maintenance (O&M) of
exiting sewage assets costing Rs. 908.16 crores.
 The projects would lead to creation of new capacity of 72 MLD, rehabilitation of 80 MLD,
operation & maintenance of existing STPs of 254 MLD capacity and 10 Sewage Pumping
Stations.

Links:
http://pib.nic.in/newsite/PrintRelease.aspx?relid=187401

Postage Stamp to commemorate Rosogulla Inventor


Question: Who is said to be the inventor of famous sweet Rosogulla?
(a) Sudhir Chandra Das (b) Mrinal Chandra Das (c) Pravin Chandra Das (d) Nabin Chandra Das
Answer: (d)
Related facts:

 A postage stamp was released to commemorate Nabin Chandra Das, the inventor of
famous Bangla dessert, Rosogolla at the inaugural function of three day Rosogulla
Festival in Kolkata marking 150th year of its invention.The Mayor of Kolkata Mr. Firhad
Hakim released the stamp.
 The festival named Bagbazar-O-Rasogolla Utsob was organized by Bagbazar Gourimata
Udyaan from December 28 to 30.
 On the occasion,a statue of the Nabin Chandra Das was unveiled and a book titled
‘Bagbazar’ authored by Haripada Bhowmick was also launched.
 On 14 November 2017, the GI (Geographical Identification) tag was given to West Bengal
for Rosogulla and hence forth this day is observed as Rosogulla Day.Prior to this, tussle
between Odisha and West Bengal government was witnessed in the year 2015 over the
invention.

40
http://www.edristi.in/
New Prime Minister of Taiwan
Question: Who is appointed as the new Prime Minister of Taiwan?
(a) Tsai Ing-wen (b) Su Tseng-chang (c) Chen Shui-bian (d) Tsai
Sungen
Answer: (b)
Related facts:

 Su Tseng-chang is appointed as the new Prime Minister of Taiwan by President Tsai Ing-
wen on 11th January 2019. This appointment took place during a major Cabinet reshuffle
following heavy loss of ruling Democratic Progressive Party in recent local polls.
 This appointment came after the resignation of former Prime Minister William Lai who
resigned along with the entire cabinet after the defeat. Taiwan is also facing severe
threats from China which considers this island its own.
 Su is a former premier appointed in 2006 and was Chairman of Presidents Democratic
Progressive Party for two terms.

Links:
https://www.cnbc.com/2019/01/11/taiwan-appoints-new-premier-and-cabinet-after-poll-defeat-
resignations.html

Prime Minister’s visit to Manipur and Assam


Question: Consider the following statements in reference to the visit of Prime Minister
Narendra Modi to Manipur and Assam.

1. This visit took place on January 4th, 2019.


2. An Integrated Check-Post was inaugurated at Moreh, Imphal.
3. Prime Minister also addressed a public rally at Silchar, Manipur.
4. Projects worth Rs. 1,500 crore have either been inaugurated or their foundational
stone had been laid down at the event.
Correct option:
(a) 1, 3 and 4 (b) 1,2 and 4 (c) 1, 2 and 3 (d) 2 and 4 only
Answer: (b)
Related facts:

 On 4th January 2019 Prime Minister Narendra Modi was on visit of Manipur and Assam.
 During his visit to Imphal, several projects worth over Rs. 1500 crore was inaugurated
and foundation stone has been laid to improve “ease of living” for the people of the state.
 PM inaugurated an Integrated Check Post at Moreh, a town bordering Myanmar. This
Check-Post will facilitate works of custom clearance, foreign currency exchange,
immigration clearance etc.
 Dolaithabi Barrage Project was also inaugurated. This is a 78.75 m long barrage built
across River Iril at Dolaithabi village of Senapati district in Manipur. This project was
approved by Planning Commission in June 1992 and completed in 2018.
 He inaugurated 400kV Double Circuit Silchar- Imphal Line in Imphal. It is constructed by
Power Grid Corporation of India Limited which will help in meeting the long- term power
requirement of state. The state would now be sharing the power from Pallatana and
Bonagaigaon generation projects.
 He also inaugurated the FCI (Food Corporation of India) Food Storage Godown at
Sawombung which will facilitate buying of grains. He also inaugurated projects related to
water supply and tourism.

41
http://www.edristi.in/
 He also mentioned the efficiency of PRAGATI system set up in Prime Ministers Office
(PMO) which enables monitoring of stalled projects through video conferencing and
resolves the issues of projects.
 He addressed a public rally at Ramnagar in Silchar. This also launched the BJPs
campaign for the upcoming general elections in the North-East.
 He addressed a public rally at Ramnagar in Silchar. This also launched the BJPs
campaign for the upcoming general elections in the North-East.

Links:
http://india-
wris.nrsc.gov.in/wrpinfo/index.php?title=Dolaithabi_Barrage_Medium_Irrigation_Project_JI0298
2

PM visits Agra
Question: Which of the following project is launched by PM Narendra Modi for better
water supply in Agra?
(a) Yamuna (b) Ganga (c) Gangajal (d) None of the above
Answer: (c)
Related facts:

 The Prime Minister Narendra Modi was on visit of Agra on 9 January where he laid
foundation and inauguration of various projects. With an estimated cost of Rs. 2,880
crores, Prime Minister launched Gangajal Project.

Gangajal Project

 It was originally launched in 2005 in collaboration with Japan International Cooperation


Agency with expected completion year by 2012.
 The initial cost of the project was Rs. 345 crores which came upto Rs. 2,880 crores after
failing several deadlines.
 With this project, Agra will get 140 ceusec of water per day from Palra headworks in
Bulandshahr’s Upper Ganga canal. Mathura and Vrindavan will also receive water
through these pipelines.

Other inaugurations:

 Prime Minister also laid foundation of Integrated Command and Control Centre for Agra
Smart City. This project which costs around Rs. 285 crores will develop Agra into a
modern smart city, befitting its stature as a premium tourist destination. CCTVs for the
purpose of safety and surveillance will be installed throughout Agra city by this project.
 Foundational stone for the upgradation of SN Medical College in Agra was also laid by
PM which will create 100 bed maternity wing in the Women’s hospital with an estimated
cost of Rs. 200 crores under Ayushman Bharat Yojana.
 Under AMRUT (Atal Mission for Rejuvenation and Urban Transformation) scheme, the
foundation stone of sewerage network project for the western part of Agra was also laid.
This will lead to improved sanitation facility in over 50,000 houses.
 Prime Minister also addressed a rally in Kothi Meena Bazaar in Agra.

Links:
http://pib.nic.in/PressReleseDetail.aspx?PRID=1559287

42
http://www.edristi.in/
Longest Single Lane Steel Cable Bridge Inauguration
Question: India’s longest single lane steel cable bridge is inaugurated in which state?
(a) Maharashtra (b) Punjab (c) Arunachal Pradesh (d) Tamil Nadu
Answer: (c)
Related facts:

 India’s longest single lane steel cable suspension bridge of 300 metre was inaugurated in
Arunachal Pradesh by the Chief Minister Pema Khandu. It was built at a cost of 4,843
crore and funded by Ministry of Development of North Eastern Region.
 The bridge is built across Siang River at Yingkiong in Upper Siang district of Arunachal
Pradesh. It is named Byorung Bridge. This will reduce the distance between Yinkiong and
Tuting near China border by almost 40 kms.
 This will also boost the defence preparedness as well as expected to benefit around
20,000 people on either banks of river.
 Chief Minister also laid foundation of District Secretariat Building. 268 projects worth Rs.
3,800 crore under the Pradham Mantri Gram Sadak Yojana (PMGSY) have been
sanctioned by Centre to the state of Arunachal Pradesh.

Links:
http://www.theweekendleader.com/Headlines/20507/india-s-longest-single-lane-steel-cable-
suspension-bridge-opens.html

Khelo India Youth Games


Question: Which state will host the 2nd edition of Khelo India Youth Games?
(a) Jharkhand (b) Maharashtra (c) Punjab (d) Kerala
Answer: (b)
Related facts:

 Pune in Maharashtra will host the second edition of Khelo India Youth Games which will
start on January 9th, 2019.
 This event will be represented by more than 6,000 athletes from 29 different States and
Union Territories in 18 different disciplines in the age group of 10 to 21 years.
 Famous athletes like Youth Olympic Gold Medalist Manu Bhaker, Saurabh Chaudhari as
well as weight lifter Jeremy Lalrinnunga will compete in the games.
 This event is to be inaugurated by Union Minister of Sports Col. Rajyavardhan Rathore.

Links:
https://youthgames.kheloindia.gov.in/news/khelo-india-youth-games-2019-an-event-that-
unearths-talent-and-binds-indias-youth

Film Festival organised in New Delhi


Question: Which Film Festival was inaugurated in New Delhi on January 4?
(a) International Film Festival (b) Indian Panorama Film Festival
(c) Bring Your Own Film Festival (d) Kashish Film Festival
Answer: (b)
Related facts:

 Indian Panorama Film Festival organised from 4-13 January 2019 was inaugurated by
Ministry of Information & Broadcasting Secretary Amit Khare at Siri Fort Auditorium in
New Delhi.

43
http://www.edristi.in/
 This ten day festival was organized by Directorate of Film Festival, Ministry of Information
& Broadcasting. The opening film of the festival was ‘Olu’ directed by Shaji N. Karun.
 The festival will conclude on January 13th, 2019. Prominent director Mrinal Sen who
passed away in recent days was remembered.
 26 Feature Films and 21 Non-Feature Films were screened in the festival. Indian
Panorama is a flagship component of International Film Festival of India. It works for the
promotion of arts by screening best of contemporary Indian films. This was started in
1978.

Links:

http://pib.nic.in/PressReleseDetail.aspx?PRID=1558716

Union Cabinet approves National Commission for Homeopathy Bill,


2018
Question: The newly drafted National Commission for Homeopathy Bill, 2018 will replace
which of the following institution?
(a) Ayush Department (b) Central Council for Homeopathy
(c) Homeopathy and Naturopathy Council (d) None of the above
Answer: (b)
Related facts:

 On 28 December 2018, Union Cabinet chaired by Prime Minister Shri Narendra


Modihasapproved the draft National Commission for Homeopathy Bill, 2018replacing
the existing regulator Central Council for Homeopathy (CCH) with a new body
ensuring greater transparency.
 This bill also provides for the constitution of a National Commission with three
autonomous boards entrusted with conducting overall education of Homeopathy by
Homeopathy Education Board.
 The Board of assessment and rating to assess and grant permission to educational
institutions of Homoeopathy and Board of ethics and registration of practitioners of
Homoeopathy to maintain National Register and ethical issues relating to practice are
under the National Commission for Homoeopathy.
 The board proposes for a common entrance exam and an exit exam needed to be
cleared by all graduates to get license for practice. It also proposes an eligibility test for
the teachers before their appointment and promotions.
 This Commission aims to bring reforms in the medical education of Homeopathy in lines
with the National Medical Commission proposed for setting up for Allopathy system of
medicine.

Links:

http://164.100.158.168/PressReleseDetail.aspx?PRID=1557605

All 640 districts covered under Beti Bachao Beti Padhao


Question: Consider the following statements with reference to the Beti Bachao Beti
Padhao scheme of Union Government:
1) This scheme was launched by Central Government to address the issue of declining
Child Sex Ratio (CSR) and encouraging Girl Child Education
2) It was launched on January 22, 2015 by Prime Minister Narendra Modi
3) It was launched by Ministry of Women and Child Development
4) This scheme has covered all 640 districts of the country
44
http://www.edristi.in/
Correct option is:
(a) 1, 3 and 4 (b) 1, 2 and 4 (c) 2, 3 and 4 (d) 1, 2 and 3
Answer: (b)
Related facts:

 The ambitious scheme of Central government Beti Bachao Beti Padhao(BBBP) has
covered all the 640 districts (as per 2011 census) since 2018-19 out of which 405 districts
are covered through Multi-sectoral intervention, Media and Advocacy while the rest 235
districts are covered through alert Media & Advocacy outreach.
 As per data,out of 161 selected districts 104 have shown improvement in Sex Ratio at
Birth(SRB),119 districts have reported progress in first trimester registration against the
reported Anti Natal Care registrations and 146 districts have reported improvement in
institutional deliveries.
 With an initial funding of 50 crore in 2014-15, it was raised to 280 crore in 2018-19.

About the Scheme:

 Beti Bachao Beti Padhao scheme was launched on January 22, 2015, to address the
issue of declining Child Sex Ratio (CSR), enabling Girl Child Education, related issues of
women empowerment and aims to change mindsets to value girl child.
 This is a tri-ministerial effort of Ministry of Women and Child Development, Health &
Family Welfare and Human Resource Development aiming to empower girl child.
 This scheme aims to stop female-foeticide. The CSR of India declined from 927 girls per
1000 boys in 2001 to 918 in 2011 citing major lacks in the laws as well as in its
implementations.
 The key elements include nation-wide awareness and advocacy campaign and multi-
sectoral action which include effective implementation of Pre Conception and Pre Natal
Diagnostic Techniques Act, Pre/Post Natal mother care, improving girls enrolment in
schools, trainings etc.

Links:
http://164.100.158.168/PressReleseDetail.aspx?PRID=1558565

PM releases commemorative stamp and coin on Paika Rebellion


Question: Consider the following statements regarding Paika Bidroha (Rebellion) of
Odisha
(1) The Paika Rebellion is widely regarded as the first war of Independence against
British rule
(2) This rebellion took place under the leadership of Baxi Jagabandhu Bidyadhar.
(3) This rebellion took place in the year of 1817
(4) PM Modi released a commemorative coin and stamp on the memory of Paika
Rebellion
Correct option:
(a) 1,2 and 4 (b) 1, 3 and 4 (c) 2 and 4 (d) All of the above.
Answer: (d)
Related facts:

 Prime Minister Modi on his visit to Odisha released a commemorative stamp and coin on
the Paika rebellion at the IIT Bhubaneswar on 24th December. The setting up of a Chair
on the Paika Rebellion, in Utkal University, Bhubaneswar, was also announced.
 Human Resource Minister Prakash Javadekar announced that a chapter of little known
Paika Rebellion will be added in the academic books from next year as the First War of
Independence.
45
http://www.edristi.in/
About Paika Rebellion:

 This year marks the 200th anniversary of Paika rebellion which was led by Baxi
Jagabandhu Bidyadhar against the Britishers in the year 1817. This rebellion predates
sepoy mutiny of 1857 which is regarded as the first war of independence.
 Paika were peasant militia under Gajapati ruler of Odisha rendering military services in
wartimes and farming in peace. Suppressed by the repressive policies of East India
Company, the revolt under the leadership of Baxi Jagabandhu Bidyadhar, militia chief of
army of Gajapati king rose the rebellion in 1817 (March- April) which spread quickly with
the support of many other tribal groups in Odisha.
 This progressed with burning of police stations, administrative offices and treasury while
they marched towards Khurda which led to fleeing of British officials.
 Some rebels fought a guerilla war till 1819 but were captured and killed. Baxi Jagabandhu
was arrested in 1825 and died in 1829 in custody. Many others were killed and
imprisoned.

Links:
http://pib.nic.in/newsite/PrintRelease.aspx?relid=186768

Official Visit of Bhutan`s PM


Question: Which of the following statement/s is/are correct about the development of
Indo-Bhutan relations with reference the present visit of Bhutanese PM to India?
1) 2018 is the Golden Jubilee year of diplomatic relations between India and Bhutan
2) This is the first foreign visit of Bhutanese PM Mr. Tshering after taking charge
3) India also announced financial help worth 4,500 crore to Bhutan
4) PM Modi also announced launching of Rupay cards in Bhutan
Correct option:
(a) 1 and 3 (b) 2 and 4 (c) 1 and 4 (d) All of the above
Answer: (d)
Related facts:

 To celebrate Golden Jubilee year of the establishment of


formal diplomatic relations between India and Bhutan, Dr.
Lotay Tshering Prime Minister of Bhutan was on a three day State visit to India from 27-
29 December, 2018.
 He was accompanied by Dr. Tandi Dorji, Minister for Foreign Affairs, Loknath Sharma,
Minister for Economic Affairs and senior officials from the Royal Government of Bhutan.
 India and Bhutan acknowledged the exemplary ties of friendship and cooperation, based
on utmost trust, goodwill and mutual understanding at all levels.
 Prime Minister Narendra Modi commended Dr Lotay’s “Narrowing the Gap” vision for
Bhutan on the lines of PM Modi’s “Together with all, development for all.”
 Several announcements were made by Indian counterparts. India announced to
contribute Rs. 4,500 crore to the 12th Five Year Plan of Bhutan (July,2018- June,2023).
India earlier provided Rs. 4,500 for the 11th Five Year Plan that lasted between 2013-
2018.
 India also re-determined its focus on infrastructural development promised to be done
including the timely completion of Mangghe-Deshchu project as well as various other
hydro projects.
 Rupay cards will be launched soon in Bhutan which will improve person to person contact
between countries. ISRO is planning to build a ground station in Bhutan which will benefit
Bhutan in the field of telemedicine, disaster relief and climate trends through the service
of South Asian satellites.

46
http://www.edristi.in/
Links:
https://www.mea.gov.in/incoming-visit-
detail.htm?30853/Joint+Press+Statement+on+the+State+Visit+of+Prime+Minister+of+Bhutan+t
o+India+December+28+2018

Technologies developed by ICAR significantly contributed to increase


in agriculture production
Question: Which schemes developed by ICAR have significantly contributed to increase
in agriculture production?
(a) PAHAL (b) SAUBHAGYA (c) USTTAD (d) ARYA and READY
Answer: (d)
Related facts:

 On 20th December 2018, Union Minister of Agriculture and Farmers’ Welfare Radha
Mohan Singh notified that the technology & technological systems developed by the ICAR
have symbolically contributed to an increase in food grain, horticultural crops, milk, fish
and eggs production.
 The ICAR has included this measure in the field of agriculture which will provide job
opportunities to the younger generation along with income enhancement of the farmers.
 In this direction, numerous steps have been taken at different levels including providing
training to the farmers at Krishi Vigyan Kendras (KVK’s) on various subjects related to
agriculture under the Entrepreneurship Development Programs (EDPs) and support with
technical resources.
 The Minister said that in order to make agriculture more attractive to rural educated youth,
the Attracting and Retaining Youth in Agriculture (ARYA) scheme is proving to be very
effective.
 The Student READY (Rural Entrepreneurship Awareness Development Yojana)
programme has also been initiated to modernize graduates of agriculture and associated
subjects for ensuring and assuring proficiency & for developing entrepreneurs in these
areas.
 The ICAR has also setup a network of 25 Agri-business Incubation (ABI) Centers.
 The efforts of these centers are resulting in new startups which are coming in the market.

Links:
http://pib.nic.in/newsite/PrintRelease.aspx?relid=186641
http://www.5dariyanews.com/news/255758-Technologies-developed-by-ICAR-have-
significantly-contributed-to-increase-in-agriculture-production

PaiSA
Question: Consider the following statements regarding PaiSA:

1. It is a portal for Affordable credit and interest subvention access.


2. It is developed by Andhra Bank.

Choose the correct options of the following:


(a) 1 only (b) 2 only (c) Both 1 and 2 (d) Neither 1 nor 2
Answer: (a)
Related facts:

47
http://www.edristi.in/
 On 26th November 2018, PaiSA a “portal for affordable credit and interest subvention
access” was launched by Sh Durga Shankar Mishra, Secretary, Ministry of Housing and
Urban Affairs.
 It is a centralized electronic platform for processing interest subvention on bank loans to
beneficiaries under Deendayal Antyodaya Yojna- National Urban Livelihoods Mission
(DAY-NULM).
 It has been designed and developed by Allahabad bank which is the nodal bank.
 It is an effort by the government to connect directly with the beneficiaries, ensuring that
there is greater transparency and efficiency in delivery of services.
 DBT of subvention on monthly basis under DAY-NULM will give the necessary financial
support to small entrepreneurs in timely manner.
 All 35 states/UTs & all Scheduled commercial banks are expected to be on board the
PaiSA portal the year end.
 More than 300 delegates comprising officers from the States, Urban Local Bodies, and
Town Planning Offices across the country and Banks participated in the workshop.

Links:
http://pib.nic.in/newsite/PrintRelease.aspx?relid=185954

Year End Review-2018-Department of Science & Technology


Question: Recently which scheme was developed by Department of Science &
Technology for a period of five years?
(a) NM-ICPS (National Mission on Interdisciplinary Cyber-Physical
Systems)
(b) TARE (Teacher Associate ship for Research Excellence)
(c) OVDF (Overseas Visiting Doctoral Fellowship)
(d) DIA (SERB Distinguished Investigator Award)
Answer: (a)
Related facts:

 On 6th December, 2018, Department of Science and Technology has projected a new
programme Interdisciplinary Cyber Physical Systems (ICPS) to cater and stimulate the
field of research & development.
 Computing and communication systems that bridge with physical world are referred to as
Cyber Physical Systems (CPS). A Cyber Physical System (CPS) use computer-based
algorithms, to integrate internet with its users.
 NM_ICPS Mission will focus on the next generation technology and bring all technology in
a particular manner.
 Under the NM-ICPS, total 25 Technology Innovation Hubs (TIH, AIH and TTRP) will
connected to Academics, Industry and Central Ministries & in other organization for
developing the solution of the problems related the technology.
 In early 2018 we saw the launch of three Science and Engineering Research Board
(SERB)’s Schemes by Union Science & Technology Minister viz. TARE (Teacher
Associate ship for Research Excellence), OVDF (Overseas Visiting Doctoral Fellowship)
and DIA (SERB Distinguished Investigator Award).

Links:

http://pib.nic.in/newsite/PrintRelease.aspx?relid=186493

48
http://www.edristi.in/
Shi Yomi becomes 23rd district of Arunachal Pradesh
Question: Which district has recently been declared the 23rd district of Arunachal
Pradesh?
(a) Shi Yomi (b) Mechuka (c) Pidi (d) Tato
Answer :(a)
Related facts

 On 9thDecember 2018, Shi Yomi has been declared as the 23rd district of Arunachal
Pradesh.
 The district, consisting of four circles Mechuka, Pidi, Tato and Monigong. Shi Yomi was
carved out of West Siang district.
 On 29th August, 2018, the Arunachal Pradesh Assembly had passed a bill for the creation
of three new districts — Pakke-Kesang, Lepa Rada and Shi Yomi.
 After the formation of Pakke-Kesang and Lepa Rada (in the future), the total number of
districts in the state will rise to 25.
 The Pakke-Kessang district has been carved out of East Kameng district, with five
administrative units, and Lepa Rada by bifurcating the Lower Siang district, with four
administrative units.

Links:

https://www.business-standard.com/article/pti-stories/shi-yomi-becomes-23rd-district-of-
arunachal-pradesh-118120900529_1.html

Odisha Government launches KALIA scheme


Question: Which of the following state has launched the KALIA scheme?
(a) Odisha (b) Karnataka (c) Gujarat (d) Madhya Pradesh
Answer: (a)
Related facts:

 On 21st December, 2018, Odisha Government has approved the comprehensive


agricultural scheme KALIA (Krushak Assistance for Livelihood and Income
Augmentation) scheme, worth over Rs 10,000 crore, for the overall development of
farmers in the state.
 This is a progressive and inclusive scheme and will make a direct impact on poverty by
way of massive investment in this sector and making benefits each of the neediest
through Direct Benefit Transfer (DBT) mode.

Provisions for Agricultural Households:

 This scheme covers all the small and marginal farmers of the state (over 30 lakh) and Rs
10,180 crore will be spent over a period of three years till 2020-21.
 The scheme will provide financial assistance to farmers with ₹10,000 a year at the rate of
Rs 5,000 each for Kharif and Rabi season. Additionally, crop loans of up to Rs 50,000 are
made interest-free under the scheme.
 Beneficiaries will be identified and selected by Gram Panchayats. Under this scheme,
about 10 lakh households will be covered over two years at a cost of Rs 100 crore.

Other Provisions:

49
http://www.edristi.in/
 Ten lakh landless households will be supported with a unit cost of Rs 12,500 to take up
activities like goat rearing units, mini layer units, duckery units, fishery kits for fishermen
and women, mushroom cultivation and bee keeping.
 Financial assistance of Rs 10,000 per household will be delivered to take care of
sustenance of farmers who are not able carry out cultivation due to old age, disability,
disease or other reasons.
 The KALIA scheme also facilitates life insurance cover of Rs 2 lakh and additional
personal accident cover of Rs 2 lakh to both cultivators and landless agriculture laborers
covering about 74 lakh households.

Links:

https://indianexpress.com/article/india/odisha-govt-announces-kalia-agriculture-scheme-
development-farmers-5504628/

Kaiga Generating Station sets world record


Question: Which Indian nuclear power unit has set up a world record of 941 days of
continuous operation recently?
(a) Kaiga (Unit-1) (b) Kakrapara (c) Kudankulam (d) Kalpakkam
Answer: (a)
Related facts:

 On 10th December, 2018, Unit-1 of Kaiga Generating Station (KGS) has registered a
world record of 941 days of continuous operation.
 Prior to this, the record was held by Heysham-2 Unit-8 (610 MWe AGR) of UK, which had
worked for 940 consecutive days.
 This landmark reflects fully matured capabilities of nation in nuclear power generation of
PHWR.
 Kaiga Nuclear Power Plant, which has been operating since 2000, is located near the
river Kali in the Uttara Kannada district of Karnataka. It is being operated by Nuclear
Power Corporation of India Limited (NPCIL). It has four units. The power station currently
operates two PHWRs (Pressurized Heavy Water Reactor).

Nuclear Power in India:

 Nuclear power is the fifth-largest source of electricity in India after coal,


gas, hydroelectricity and wind power.
 At present, India has 22 nuclear reactors in operation in 7 nuclear power plants .Total
installed nuclear power capacity in India is 6780 MW.
 In India, there are seven Atomic Power Stations located in Gujarat, Karnataka,
Maharashtra, Rajasthan, Tamil Nadu and Uttar Pradesh.
 Atomic Energy Regulatory Board (AERB) formed in the year 1983 is a regulatory body for
nuclear energy in India.

Links:

http://164.100.117.97/WriteReadData/userfiles/PIB%20DAE%20ACHIVEMENTS%20CDR.pdf

International Film Festival of Kerala


Question: Which film was conferred on the Best Film in the 23rd International Film Festival of
Kerala?
(a) The Dark Room (b) The Shape of Water (c) The Silence (d) None of these
50
http://www.edristi.in/
Answer: (a)
Related facts:

 In December 2018, the 23rd edition of International Film Festival of Kerala (IFFK) was
organized at Nisagandhi Auditorium in Thiruvananthapuram from 7-14 December 2018.
 Around 160 films were exhibited during festival to deepen the roots of unity among all
sections, including organizers and art lovers.
 The Golden Crow Pheasant Award for Best Film was given to ‘Dark Room’ directed by
Iranian filmmaker Rouhallah Hejazi.
 Lijo Jose Pellissery has been conferred the Silver Crow Pheasant for best director for
‘EE.Maa.Yove’ and the NETPA Award for best Asian film in the competition.
 Anamika Haksar received the Silver Crow Pheasant for best debut director for Taking the
Horse to Eat Jalebis

Links:

https://www.deccanchronicle.com/nation/current-affairs/141218/curtains-on-23rd-international-
film-festival-of-kerala.html

Australia grabs world’s biggest LNG exporter crown from Qatar


Question: Which country recently became the world’s largest exporter of liquid natural
gas (LNG)?
(a) Australia (b) Qatar (c) Russia (d) United States
Answer: (a)
Related facts:

 According to the data released by Refinitiv Eikon, on 10th December 2018 Australia
became the world largest exporting country in the world of liquefied natural gas (LNG).
 According to the data released for the month of November, Australia is at the first place
whereas Qatar is replaced to second place in the list.
 In November, Australia exported 6.5 million tonnes of LNG, while Qatar exported 6.2
million tonnes of LNG.
 It is worth noting that Qatar is currently producing 77 million tonnes annually, which it
plans to increase to 110 million tonnes by 2024.

Links:

https://in.reuters.com/article/australia-qatar-lng/australia-grabs-worlds-biggest-lng-exporter-
crown-from-qatar-in-nov-idINKBN1O908E

Atal Ayushman Uttarakhand Yojana


Question: Atal Ayushman Uttarakhand Yojana was launched at which of the following
place?
(a) Dehradun (b) Rudraprayag (c) Nainital (d) Pitharogarh
Answer: (a)
Related facts:

 On 25th January, 2018, Uttarakhand Chief


Minister Trivendra Singh Rawat launched the Atal

51
http://www.edristi.in/
Ayushman Uttarakhand Yojana in order to cover all residents of state under a health
insurance cover at a function in Race Course, Dehradun.
 With its launch Uttarakhand became the first hill state in the country to offer universal
health coverage. On the occasion, the CM distributed ‘golden cards’ to the beneficiaries.

About the scheme:

 On the lines of Ayushman Bharat, under the scheme each household in the state will be
able to avail medical treatment of up to Rs 5 lakh annually.
 The scheme will benefit 23 lakh households and will cover 1,350 critical diseases.
 A total of 99 government and 66 private hospitals have been specified in which people
will be able to get cashless treatment under the scheme.
 A MOU with private hospitals was also signed and it was notified that soon free of cost
OPD facilities will be available for children and elderly people in the state.

Links:

http://www.newsonair.com/Main-News-Details.aspx?id=357152

Asiatic Lion Conservation Project launched


Question: What is the fund contribution ratio of Centre-State in the Asiatic Lion
Conservation Project?
(a) 50:50 (b) 60:40 (c) 70:30 (d) 90:10
Answer: (b)
Related facts:

 On 20th December, 2018, The Ministry of Environment, Forest and


Climate Change, Government of India has launched the “Asiatic Lion
Conservation Project”.
 This project seeks to strengthen the ongoing measures for conservation and recovery of
Asiatic Lion with the help of state-of-the –art techniques/ instruments, regular scientific
research studies, disease management, Modern surveillance/ patrolling techniques.
 A total budget of Rs. 9784 lakh for 3 years has been allocated by government which is to
be funded from the Centrally Sponsored Scheme- Development of Wildlife Habitat (CSS-
DWH) with the contributing ratio being 60:40 of Central and State share.

Asiatic Lion:

 Asiatic Lion (scientific name Panthera leo leo) is an ‘Endangered’ specie in IUCN Red
List.
 Asiatic lions are slightly smaller than African lions. The most striking morphological
character which is always seen in Asiatic lions and rarely in African lions is a longitudinal
fold of skin running along its belly.
 Its population is confined to the Gir National Park and environs in the Indian state
of Gujarat.

Conservation Concerns:

 With comprehensive protection measures by the State Government and the Center
Government, number of Asiatic lions have increased to the current population of over
500.This figure was at 50 by late 1890.
 As per Census 2015, population of Asiatic Lions was marked to 523 in Gir Protected Area
Network of 1648.79 sq. km. that includes Gir National Park, Gir Sanctuary, Pania
52
http://www.edristi.in/
Sanctuary, Mitiyala Sanctuary adjoining reserved forests, Protected Forests, and
Unclassed Forests.
 Asiatic Lion is included in the list of 21 critically endangered species for recovery
programme and financial assistance under the species recovery component of CSS-
DWH.
 The project activities are conceptualized to encourage habitat improvement, scientific
interventions, disease control and veterinary care supplemented with adequate eco
development works for the fringe population in order to ensure a stable and viable Lion
population in the Country.

Links:

http://pib.nic.in/newsite/PrintRelease.aspx?relid=186688

Year End Review-2018-Department of Science & Technology


Question: Recently which scheme was developed by Department of Science &
Technology for a period of five years?
(a) NM-ICPS (National Mission on Interdisciplinary Cyber-Physical
Systems)
(b) TARE (Teacher Associate ship for Research Excellence)
(c) OVDF (Overseas Visiting Doctoral Fellowship)
(d) DIA (SERB Distinguished Investigator Award)
Answer: (a)
Related facts:

 On 17 December 2018, Department of Science & Technology (DST) under the Ministry of
Science & Technology has released its Year End Review featuring its year-long
achievements in 2018.
 Among one of the biggest developments, on 6th December 2018, Department of Science
and Technology has projected a new programme “Interdisciplinary Cyber Physical
Systems (ICPS)” to cater and stimulate the field of research & development.
 National Mission on Interdisciplinary Cyber-Physical Systems (NM-ICPS):
 A Cyber-Physical System (CPS) is a mechanism that is controlled or monitored by
computer-based algorithms, tightly integrated with the Internet and its users. In cyber-
physical systems, physical and software components are deeply intertwined, each
operating on different spatial and temporal scales, exhibiting multiple and distinct
behavioral modalities.
 The objective of NM-ICPS Mission is to established total 25 technology hubs in the
country in which 15 are Technology Innovation Hubs, six are Application Innovation Hubs
and remaining four will be Technology translation Research Hubs.
 Plaaning and Projects:
 The Global Cooling Prize launched in November this year is an innovation challenge that
aims to spur development of a residential cooling solution that has at least five times (5x)
less climate impact than today’s standard products.
 This technology could prevent up to 100 gigatons (GT) of CO2-equivalent emissions by
2050. Over US$3 million will be awarded in prize money after the 2-year competition.
 A major partnership Indo-Korean Center for Research and Innovation (IKCRI) was
launched in India, which will act as the hub for systematic operation and management of
all cooperative programmes in research and innovation between the two countries
including innovation & entrepreneurship and technology transfer.
 In early 2018 three Science and Engineering Research Board (SERB)’s Schemes was
launched by Union Science & Technology Minister viz. TARE (Teacher Associate ship for

53
http://www.edristi.in/
Research Excellence); OVDF (Overseas Visiting Doctoral Fellowship) and DIA (SERB
Distinguished Investigator Award).
 Other initiatives by Ministry of Science and Technology:
 AWSAR – Augmenting writing skills for articulating research.
 SRIMAN- Scientific research infrastructure and maintenance Networks.
 Festival of innovation and entrepreneurship (FINE)
 India’s first supercritical Brayton Cycle CO2 test facility at IISC Bangalore.

Links:
https://en.wikipedia.org/wiki/Cyber-physical_syste

Year End Review Department of Biotechnology


Question: What does SAEN stands for?
(a) Secondary Agriculture Entrepreneurial Network (b) South Asian Entrepreneurial Network
(c) Secondary Aeronautical Entrepreneurial Network (d) None of the
above
Answer: (a)
Related facts:

 On 20th December 2018, Department of Biotechnology released its Year end review
featuring its year-long achievements.
 Major highlights:
 DNA Technology (Use and application) Regulation Bill, 2018 has been approved Union
cabinet. This is a Bill to provide for the regulation of the use and application of
Deoxyribonucleic Acid (DNA) technology with the aim to establish the identity of certain
category of persons including the victims, offenders, suspects, under trials, missing
persons and unknown deceased persons and provides provision for establishment of a
DNA Regulatory Board (DRB).
 India-UK cancer research initiative launched in collaboration with ‘Cancer Research UK
(CRUK) focusing on affordability of cancer prevention and care, and the potential to
make.
 The Department of Biotechnology (DBT), in collaboration with the Welcome Trust, is
supporting a three-tier fellowship programme on biomedical research at post-doctoral
level.
 Post Graduate Certificate/Diploma was implemented to provide high quality hands on
training in tools and techniques in Medical Biotechnology, Agricultural Biotechnology and
Computational Biology.
 Indo-US collaboration on Genome Engineering / editing was launched.
 1st clean energy international incubator has been set up under Mission Innovation.
Startups from 23 participating EU countries can potentially come & incubate in India and
likewise startups from this incubator can go to the partnering countries facilitating access
to global opportunities.
 SAEN (Secondary agriculture entrepreneurial network) was launched in 2018. The project
aims to promote new enterprises and to support existing industry in the secondary
agriculture sector.
 A major mission program on Antimicrobial Resistance (AMR) was launched with the
vision to develop indigenous and cost-effective therapies against AMR.
 A new regional center, BIRAC Regional Bio-innovation Centre (BRBC) was established at
Venture Centre, Pune.
 DBT’s Indo-US Vaccine Action Programme (VAP) & the Vaccine Grand Challenge
Programme (VGCP) scored high with achievements like the lowest cost Rotavirus
vaccine, Vaccine for falciparum malaria and a comprehensive program for prevention and
control of sickle cell anemia and thalassemia has been initiated.
54
http://www.edristi.in/
 Under Bio Design Programme, two technologies namely Intra-Osseous Device (Ozyn-D)
and a “Chest Tube Fixator and Sealing Device (PleuraGoh)” were licensed to start-up
companies.
 New Wheat variety Unnat PBW343, resistant to leaf rust and stripe rust was developed by
PAU. Two bacterial blight resistant Basmati rice varieties Pusa Basmati 1728 and Pusa
Basmati 1718 were developed. DBT-PGGF “Plant Genotyping and Genomics Facility”
(PGGF) anchored at NIPGR, New Delhi in PPP mode was established.

Links:
http://pib.nic.in/newsite/PrintRelease.aspx?relid=186662

States’ Startup Ranking 2018


Question: Which State was declared the best performer in States’ Startup Ranking
2018’?
(a) Gujarat (b) Karnataka (c) Kerala (d) Odisha
Answer: (a)
Related facts:

 The Department of Industrial Policy and Promotion (DIPP) announced the first ever States
Start-up Ranking 2018 at an event in New Delhi on 20th December 2018.
 States have been identified as leaders across various categories such as start-up leaders,
incubation hubs, seeding innovation, scaling innovation, regulatory change champions,
procurement leaders, communication champions, North-Eastern leader and Hill State
leader.
 On the basis of performance in these categories, the states have been recognized as the
Best Performer, Top performers, Leaders, Aspiring Leaders, Emerging States and
Beginners, as follows:
 Best Performer – Gujarat
 Top Performers – Karnataka, Kerala, Odisha and Rajasthan.
 Leaders- Andhra Pradesh, Bihar, Chhattisgarh, Madhya Pradesh and Telangana.
 Aspiring Leaders – Haryana, Himachal Pradesh, Jharkhand, Uttar Pradesh and West
Bengal.
 Emerging States – Assam, Delhi, Goa, Jammu & Kashmir, Maharashtra, Punjab, Tamil
Nadu and Uttarakhand.
 Beginners – Chandigarh, Manipur, Mizoram, Nagaland, Puducherry, Sikkim and Tripura.
 Fifty-one officers from States and Union Territories have been identified as Champions,
who have made significant contributions towards developing their State’s Start-up
ecosystem.
 The key objective of the exercise was to encourage States and Union Territories to take
proactive steps towards strengthening the Start-up Ecosystems in their states.
 DIPP consulted all stakeholders of the start-up ecosystem and came up with 7 key reform
areas as the basis of the States start-up ranking Frame work.

Links:
http://pib.nic.in/newsite/PrintRelease.aspx?relid=186652

Paradip Hyderabad Pipeline Product Project and Bokaro-Angul Section


of Jagdishpur-Haldia & Bokaro-Dhamra Gas Pipeline Project
Question:The foundation stone of ‘Paradip Hyderabad Pipeline product Project’ (PHPL)
and ‘Bokaro-Angul Section of Jagdishpur – Haldia & Bokaro – Dhamra Gas Pipeline
Project’ was laid down by?

55
http://www.edristi.in/
(a) PM Narendra Modi (b) Dharmendra Pradhan (c) Naveen Patnaik (d) Nitin Gadkari
Answer: (a)
Related facts:

 The foundation stone of Paradip Hyderabad Pipeline Product Project and ‘Bokaro-Angul
section of Jagdishpur-Haldia & Bokaro-Dhamra Gas Pipeline Project’ was laid down by
Prime Minister Narendra Modi on 24th December 2018.
 Highlights of the Projects:
 Paradip Hyderabad Pipeline Product Project is 1212 km long pipeline is being built by IOC
Ltd, at a Sanctioned cost of Rs. 3,800 crores.
 It is capable of transporting 4.5 MMTPA of petrol, diesel, kerosene and Aviation Turbine
Fuel (ATF).
 The pipeline originates at Paradip and traverses through three States – Odisha (329km),
Andhra Pradesh (723km) and Telangana (160km) before terminating at Hyderabad.
 Bokaro-Angul pipeline is 667 km long sections of landmark Jagdishpur-Haldia-Bokaro-
Dhamra Pipeline project (PM Urja Ganga) is being built by the Gas Authority of India Ltd.
At a sanctioned cost of Rs. 3,437 Crores.
 This pipeline is part of the landmark Pradhan Mantri Urja Ganga project and traverses
367 km across 5 districts in Odisha and 360 km across 6 districts in Jharkhand.

Links:
http://pib.nic.in/newsite/PrintRelease.aspx?relid=186682

National mission on Interdisciplinary Cyber Physical System


Question: (NM-ICPS) National mission on Interdisciplinary Cyber Physical System aims
to benefit:
(a) Health sector (b) Education sector (c) Energy sector (d) Environment sector (SDG’s)
(e) All of the above
Answer: (e)
Related facts:

 The Union Cabinet headed by the Prime Minister has


approved the launching of National Mission on
Interdisciplinary Cyber-Physical Systems (NM-ICPS) which is to be implemented by DST
(Department of Science & Technology).

Objectives of the Mission:

 Cyber Physical Systems (CPS) and associated technological system within reach in the
country,
 CPS technology to address specific issues relating to (i) National and (ii) Regional issues.
 To produce newer generation manpower skilled in CPS,
 Further it aims to speed up translational research.
 To accelerate entrepreneurship and start-up ecosystem development in CPS.
 To stimulate advanced research in CPS, Technology development and higher education
in field of Science, Technology and Engineering disciplines.
 To place India in level with other advanced countries and extract several direct and
indirect benefits.

Implementation Strategy and Impact:

56
http://www.edristi.in/
 The Mission aims to establish: 15 numbers of Technology Innovation Hubs (TIH), 06
numbers of Application Innovation Hubs (AIH) and 04 numbers of Technology Translation
Research Parks (TTRP).
 These Hubs & TTRPs will connect to Academics, Industry, Central Ministries and State
Government & cater to provide innovative solutions at reputed academic, R&D and other
organizations across the country.
 Strong Steering and Monitoring Mechanisms in the form of Mission Governing Board
(MGB), Inter-Ministerial Coordination Committee (IMCC), Scientific Advisory Committee
(SAC) and other Sub-Committees will guide and monitor the Mission implementation.
 The Hubs & TTRPs aims to focus in four areas along which the Mission implementation
would proceed, viz

I. Technology Development;
II. HRD & Skill Development;
III. Innovation, Entrepreneurship & Start-ups Ecosystem Development
IV. International Collaborations.

 The Mission will feed the Central Ministries/ Departments and State Govts and also the
Industry to effectively use the CPS technologies in their projects and schemes for the
benefit of the society.
 NM-ICPS is a Pan India Mission and covers entire gamut of India that includes Central
Ministries, State Governments, Industry and Academia.

Links:
http://pib.nic.in/newsite/PrintRelease.aspx?relid=186157

Ministry of Corporate Affairs- Year Ender-2018


Question: What is the rank of India in the Doing Business 2019 report published by World
Bank?
(a) 100th (b) 78th (c) 79th (d) 77th

Answer: (d)
Related facts:

 On 20th December 2018, Ministry of Corporate Affairs has released its Year Ender-2018
which highlights the details of Year-long achievements of Ministry.
 India has advanced its ranking on the World Bank’s “Doing Business 2019”report
released on 31st October, 2018.
 According to the report, India has moved up 23 spots to 77th rank as compared to 100th
position in 2017 by further improving its rank in six out of ten performance evaluating
criteria’s relating to starting and doing business in India.
 In conduct to objective of providing greater Ease of Doing Business to all shareholders,
transparency in corporate structure has been enhanced and better corporate conformity
so as to increase the efficiency of the processes under Companies Act, 2013, the
Ministry of Corporate Affairs (MCA) has taken several landmark initiatives / decisions.

Provisions regarding E-Governance:

 Introduction of RUN – Reserve Unique Name web service for name: To introduce a web
based service named as RUN – Reserve Unique Name for constructing the Name
Reservation process Rapid, Steady, Elementary and condensing the number of rules
with effect from 26th January 2018 for Companies and from 2nd October 2018 for LLPs
(Limited Liability Partnership).

57
http://www.edristi.in/
 Re-engineering the process of allotment of DIN (Director Identification Number).
 Exemption of MCA fee for company incorporation.
 Deployment of e-forms due to IFSC & exemption notifications, amendment to
Companies Act, CRL-1, implementation of Condonation of

Delay Scheme (CODS).

 eKYC drive for directors of all companies.


 Integrated form for LLP (FiLLiP) incorporation.
 Setting up Central Registration Centre (CRC) for “name reservation” and “incorporation”
for LLPs.
 National Company Law Tribunal:
 To accelerate matter related to issues which needs resolving for bankruptcy and failure,
MCA further proposed to set up 8 special courts under the National Company Law
Tribunal so to deal with the failure cases.
 These courts are proposed to be set up in Mumbai, Delhi, Chennai, Kolkata and
Hyderabad. The aim is also to strengthen the insolvency process for faster resolution of
NPAs.

Indian Accounting Standard:

 MCA has notified Indian Accounting Standard (Ind AS) 115 in order to bring about more
transparency in accounting, which would be effective from 1st April 2018.
 Dematerialization of Securities of Unlisted Public Sector Companies:
 For Corporate structure bodies, dematerialization of securities, especially in terms of
KYC and investor protection, the Government’s focus is on Digital India and thrives to
enable the provisions available under section 29(1) (b) of the CA-13; the Ministry has
modified the applicable rules so as to apply the dematerialization requirements for public
companies, in addition to listed companies.

Investor Education & Protection Fund (IEPF):

 The Investor Education & Protection Fund (IEPF) authority disclosed its new logo in
2018 in order to provide for strong brand presence and acceptance.
 The IEPF authority also signed a Memorandum of Understanding (MoU) with the CSC e-
governance Services India, wherein the CSC would determine village level
entrepreneurs for investor awareness projects.
 MCA is earnestly looking at further modifications in IEPF.

Competition Matters:

 For enlarging the opportunity for debate of competition in India and to bring the best
practices from around the world on competition issues, Competition Commission of India
(CCI) successfully anchored the 17th International Competition Network (ICN) Annual
Conference in March 2018 in New Delhi.
 Nearly 500 professional attended the conference from 70 plus countries, which included
heads of competition agencies, representatives and shareholders consisting of legal and
economic professionals, international organizations and academics.

Links:
http://pib.nic.in/newsite/PrintRelease.aspx?relid=186630

58
http://www.edristi.in/
India China Film Festival
Question: India-China Film Festival organized during the 1st India-China High Level
Mechanism on Cultural and People-to-People Exchanges was hosted by which of the
following city?
(a) New Delhi (b) Shanghai (c) Mumbai (d) Beijing
Answer :(a)
Related facts:

 Three day India-China film Festival started on 22nd December, 2018, at the Siri Fort
Auditorium II, New Delhi.
 Organizer of the film festival was the ‘Directorate of Film Festivals, Ministry of
Information & Broadcasting and Government of India.
 This festival seeks to strengthen people-to-people connect and deepen cultural
understanding between the two countries.
 A total of seven films including four Chinese and three Indian film s screened during the
festival.
 The opening film of the festival was ‘CZ12’ also known as ‘Chinese Zodiac’. Indian films
of this festival were – ‘Dangal’ (Hindi), ‘Maacher Jhol’ (Bengali) and ‘Ventilator’ (Marathi)

Links:
http://pib.nic.in/newsite/PrintRelease.aspx?relid=186658

Agriculture Export Policy 2018


Question: By which year Indian Government is resolved to double farmers’ income
according to Agricultural Export Policy 2018?
(a) 2020 (b) 2021 (c) 2022 (d) 2024
Answer: (c)
Related facts:

 On 6th December, 2018, Union Cabinet headed by the Prime Minister approved
Agricultural Export policy 2018 which aimed to double farmers’ income by 2022.
 The Agriculture Export Policy aims to Harness export potential of Indian agriculture,
through suitable policy instruments, to make India global power in agriculture and raise
farmers’ income.
 The export policy seeks to double agricultural exports from present ~US$ 30+ Billion to
~US$ 60+ Billion by 2022 and reach US$ 100 Billion in the next few years thereafter,
with a stable trade policy regime.
 Agriculture Export Policy has been organized in two categories:
 The Strategic class includes parameters like: (i) Infrastructure support with logistics, (ii)
Integrated approach to boost exports, (iii) Larger participation of State Governments in
agriculture exports, (iv) Key Focus on Clusters, (v) To promote exports for value-added
exports and (vi) Marketing and promotion of Brand India.
 The operational part of the structure includes: (i) Attract private investments production
and processing, (ii) Establishment of strong quality regime, (iii) R & D and (iv)
Miscellaneous.

Links:
http://pib.nic.in/newsite/PrintRelease.aspx?relid=186182

59
http://www.edristi.in/
PM Inaugurates Bogibeel Bridge
Question: India’s longest rail-cum-road bridge was inaugurated recently in which State of
India?
(a) Arunachal Pradesh (b) Assam (c) Mizoram (d) Odisha
Answer: (b)
Related facts:

 On 25th December, 2018, India’s longest rail-cum-road bridge over the Brahmaputra
River in Assam was inaugurated on the birth anniversary of former prime Minister Atal
Bihari Vajpayee.
 The 4.9 kilometre long Double-decker Bridge is situated 17 km downstream of Dibrugarh
and Dhemaji. The bridge connects Dhemaji and Dibrugarh districts of Assam.
 This is also Asia’s 2nd longest rail-cum-road bridge having a serviceable period of
around 120 years and is constructed at an estimated cost of Rs 5,900 crore.

Features:

 On the lower deck the bridge has two railway lines and a three-lane road on the upper,
strong enough to withstand movement of heavy military tanks.
 This bridge is based on the design of Oresund Bridge that joins Sweden and Finland
across the Oresand strait.
 It is India’s first bridge to have fully welded steel-concrete support beams that can
withstand earthquakes of magnitudes up to 7 Richter scale.

Background:

 The Bogibeel project was a part of the 1985 Assam Accord and was sanctioned in 1997-
98. The foundation stone was laid by then Prime Minister HD Deve Gouda on January
22, 1997, but the commencement of work on the project was done by Vajpayee on April
21, 2002. It took 200 months to complete the project as heavy rainfall in the region being
the main cause for the slow progress.

Importance of the Bridge:

 This bridge has significant social, cultural, economic and military importance as it
augments feasibility and accessibility to resources in efficient way.
 The road distance from Dibrugarh to Itanagar will be shortened by 150 km and the
railway travel distance between these two points will shrink by 705 km while this bridge
will provide an alternate and shorter route from Dibrugarh through north bank of
Brahmaputra to Delhi and Kolkata via Rangiya.
 The bridge over the Brahmaputra will boost defence logistics along the farthest point of
India’s border with China as it will facilitate easy movement of troops and supplies from
southern to northern bank.
 This bridge is a lifeline for crores of people in Assam and Arunachal Pradesh as it will
facilitate connectivity between north and south banks of river Brahmaputra in the eastern
region of Assam and Arunachal Pradesh and will boost the economy and improve trade.

Links:
http://www.newsonair.com/Main-News-Details.aspx?id=357095

60
http://www.edristi.in/
Year End Review 2018: Ministry of Tribal Affairs
Question: What is the budget allocation of the Ministry of Tribal affairs in the year 2018-
19?
(a) Rs. 5329.32 Crore (b) Rs. 5957.18 Crore (c) Rs. 6500.00 Crore (d) None of the above
Answer :(b)
Related facts:

 On 20th December, 2018 Ministry of Tribal Affairs released Year


end review 2018.
 Scheduled Tribes component across all Ministries witnesses a
substantial increase form Rs. 31,920.00 Cr. In the year 2017-18 to
Rs. 37,802.94 Cr in 2018-19.
 The scheduled Tribes comprise about 8.6% of India’s population (according to the 2011
census).
 Budget allocation of the Ministry of Tribal Affairs has gone up from Rs. 5329.32 Crore in
the year 2017-18 to Rs. 5957.18 Cr in 2018-19.
 There are 37 central Ministries and Departments having STC (Scheduled Tribes
Component) Funds catering to specific tribal development in various sectors through 299
different schemes.
 With the efforts of Ministry of Tribal Affairs, the STC expenditure has gone up from 85%
of total allocation at revised estimates stage in 2016-17 to 96% in 2017-18.
 Vigorous efforts of MoTA, saw TSP funds 98% increase in expenditure of State Tribal
Sub-Plan funds by States during the years block of 2010-13 (wherein a total of Rs.
1,65,691.00 crore was expended) to 2014-17 (wherein a total of Rs. 3,27,574.00 crore
were expended).

Schemes

 Eklavya Model Residential School is an excellent approach for imparting quality


education to ST children.
 A total of 284 EMRSs have been sanctioned as on date, out of which 219 have been
reported functional, with about 65,231 students enrolled therein.
 The Cabinet Committee on Economic Affairs has decided that by the year 2022, every
block with more than 50% ST population and at least 20,000 tribal persons, will have an
Eklavya Model Residential School.
 Government has conceptualized and planned setting up of permanent museums in
States where Tribals lived, struggled against the Britisher’s, refused to bow down to alien
rule and were far ahead in making sacrifices.
 Ministry has decided to construct a state-of-the-art Museum for Tribal freedom fighters of
national importance in Gujarat with a total outlay of Rs. 108.55 crore and MoTA’s
contribution of Rs. 50.00 crore.
 The second national level museum is to come-up in Jharkhand with an outlay of Rs.
36.66 crore and MoTA’s contribution OR Rs. 25.00 Crore.
 National fellowship and Scholarship scheme for higher education of ST students –
Financial assistance in the scheme for 2018-19 is Rs. 100.00crores

Links:
http://pib.nic.in/newsite/PrintRelease.aspx?relid=186633

61
http://www.edristi.in/
Ministry of Commerce & Industry to setup trade promotion bodies in 15
countries
Question: Ministry of Commerce & industry is examining the proposal to set up trade
promotion bodies in how many countries?
(a) 15 (b)20 (c)25 (d) None of the above
Answer: (a)
Related facts:

 Suresh Prabhu informed the participants of the 15th Global


SME Business Summit (19th December, 2018) about the
commitment of the Ministry of commerce & industry towards
the growth of the SMEs.
 The concept of trade has been changing where larger enterprises increasingly rely on
outsourcing the manufacturing activities. This practice has proved quite advantageous to
the MSMEs where they can participate in the exports through manufacturing of
intermediate products.
 Ministry of Commerce & Industry is examining the proposal to setup trade promotion
bodies in 15 countries.
 Ministry is also working on a comprehensive strategy to boost the MSME sector by
granting new patents in record time. New Intellectual Property regime is being launched
to strengthen the IPRs of Indian MSMEs as well as protecting IPRs of the other
countries.
 The 2018-19 Budgets has given a big thrust to Micro, Small and Medium Enterprises
(MSMEs) sector to boost employment and economic growth. The budgetary allocation
has gone up from Rs. 6481.96 crore in 2017-18 to Rs.6552.61 crore in 2018-19.

Links:
http://pib.nic.in/PressReleaseIframePage.aspx?PRID=1556657#

National Medical Devices Promotion Council


Question: Although the Medical Devices Industry (MDI) has been growing in double
digits but is predominantly import-driven with imports accounting for over __ of the
domestic market.
(a) 65 Percent (b) 85 Percent (c) 45 Percent (d) None of the Above
Answer: (a)
Related facts:

 4th WHO Global Forum on Medical Devices was


organized on 14th December 2018 at Andhra Pradesh
Medtech Zone, in Vishakhapatnam.
 To give a fillip to the medical device sector, a National
Medical Devices Promotion Council under the Department of Industrial Policy and
Promotion (DIPP) in the Ministry of Commerce & Industry is been proposed.
 Although the industry has been growing in double digits but is predominantly import-
driven with imports accounting for over 65% of the domestic market.

Objectives:

 Medical devices industry (MDI) plays a critical role in the healthcare ecosystem and is
indispensable to achieve the goal of health for all citizens of the country.

62
http://www.edristi.in/
 The setting-up of the council will spur domestic manufacturing in this sector, attract
investment and to promote exports in this fast growing sector.
 The council will be headed by Secretary, DIPP, and it will have representatives from
health care industry and quality control institutions.

Links:
http://pib.nic.in/newsite/PrintRelease.aspx?relid=186385

National e-Vidhan Application (NeVA) Project


Question: NeVA aims to bring all the _ of the country together, in one platform thereby
creating a __ without having the complexity of multiple
applications.
(a) Legislatures, massive data depository (b) high court, huge network
(c) district courts, metropolitan area network (d) None of the above
Answer: (a)
Related facts:

 Ministry of Parliamentary affairs gave details on the status of implementation of National


e-Vidhan application (NeVA) Project on 18 December, 2018.
 Paperless assembly or e-Assembly is a concept involving of electronic means to
facilitate the work of assembly. NeVA aims to bring all the legislatures of the country
together, in one platform thereby creating a massive data depository without having the
complexity of multiple applications.
 NeVA will completely eliminate the process of sending out a notice/request for collection
of data.
 E-Vidhan is a Mission Mode Project (MMP) included in Digital India Programme and
Ministry of Parliamentary affairs (MoPA) is the ‘Nodal Ministry’ for its implementation in
all the 31 states/UTs with Legislatures.
 The total project cost of the project is over 738 crores and funding of NeVA is on the
pattern of central sponsored scheme i.e. 60:40; and 90:10 for North East & hilly states
and 100% for UTs.
 The funding for e-Vidhan is provided by the MoPA and technical support by Ministry of
electronics and information technology (MeitY).

Links:
http://pib.nic.in/newsite/PrintRelease.aspx?relid=186569

Multi-Disciplinary Training Centre


Question: Ministry of MSME inaugurated a Multi-Disciplinary Training centre of Khadi
and Village Industries Commission In which city?
(a) New Delhi (b) Chandigarh (c) Kanpur (d) Surat
Answer: (a)
Related facts:

 A Multi-Disciplinary Training Centre of ‘Khadi and Village


industries commission’ was inaugurated on 13th December
2018.
 Delhi based training centre ‘Samadhan’ is an industrial training institute set up on PPP
mode with the MSME Ministry, to provide comprehensive industry and business related
guidance and quality knowledge to entrepreneurs, industries, corporate and institutions.
 The MSME ministry also inaugurated a mustard-oil manufacturing unit at KVIC office at
Gandhi Darshan Rajghat.

63
http://www.edristi.in/
 MSME Minister, Giriraj Singh also planted ‘Moringa saplings’ in the premises of the
mustard oil Manufacturing unit at the Multi-disciplinary Training Centre (MTDC).

Links:
http://pib.nic.in/PressReleaseIframePage.aspx?PRID=1555863

Ministry of Steel initiative of Make in Steel – Make in India


Question: A contest was conducted to popularized steel and inviting ideas by the
Ministry of Steel is:
(a) #myLOVESTEELideas (b) # myAFFECTIONSTEELideas (c) # myCUTESTEELideas
(d) None of the Above
Answer: (a)
Related facts:

 A contest was conducted to popularize use of steel and


inviting ideas in steel sector called #myLOVESTEELideas.
 The per capita consumption ha s risen from 59 kg in 2013-14 to 69 kg in 2017-18. India
has produced 103 Million Tonnes of steel in 2017-18.
 The capacity of steel production has increased from 97 Million Tonnes in 2012-13 to 138
Million Tonnes in 2017-18.
 The Ministry of Steel utilized MyGov platform to crowd source ideas for increasing steel
consumption in India. A contest was conducted inviting ideas in steel sector called
#myLOVESTEELideas:
 Flagship programmes like 100 Smart Cities Mission, Housing for all Mission, Atal
Mission for Rejuvenation and Urban Transformation and high-speed bullet trains and
metro trains all will hugely contribute in increasing steel demand in our country.
 In 2018, the scrapping policy of Ministry of steel is being finalized.

Links:
http://pib.nic.in/newsite/PrintRelease.aspx?relid=186572

Indian National Centre for Ocean Information (INCOIS)


Question: What is the full form of ‘ITC Ocean’?
(a) International Training Centre for Operational Oceanography
(b) International Training Council for Operational Oceanography
(c) Indian Training Centre for Operational Oceanography
(d) None of the above
Answer: (a)
Related Facts:

 Union Minister Dr. Harsh Vardhan inaugurated and dedicated, the newly constructed
International Training Centre for Operational Oceanography Centre in the INCOIS
campus to the former Prime Minister Shri Atal Bihari Vajpayee, at Hyderabad on 22nd
December, 2018.
 The UNESCO category 2 centers established as a training facility at INCOIS will give an
opportunity to the South Asian and African states bordering the Indian Ocean and the
small island nations in the Pacific to benefit from the expertise and experience of
INCOIS in the areas of ocean science and management.
 To scale up the activities of ITCOocean, a full-fledged facility consisting of an Academic
building and International Guest House has been constructed on the INCOIS campus.
 INCOIS is a leading operational oceanography institute in the region. Tsunami early
warnings from the institute are delivered to 25 countries on the Indian Ocean Rim on
real-time basis.
64
http://www.edristi.in/
 Under the aegis of Regional Integrated Multi-hazard Early warning System for Africa and
Asia (RIMES), INCOIS is also providing ocean state forecast and other related warnings
to 5 countries (Sri Lanka, Maldives, Seychelles, Madagascar and Cameroon).

Links:
http://www.newsonair.com/Main-News-Details.aspx?id=356989

India’s First Water Metro


Question: In which of the following cities The Water Metro, an integrated water-transport
project has started?
(a) Kochi City (b) Kozhikode (c) Kollam (d) Thiruvananthapuram
Answer: (a)
Related facts:

 India’s first and German-Government backed Water Metro,


an integrated water-transport project connecting 10 islands near the Kochi city, spread
across 76 km, finally started.
 The Water Metro, being developed as one of the fastest means of transport, will help
people commute easily to hubs which generate employment. The project is expected to
be completed in four years.
 During the Phase I, it is proposed to operate 7 routes with a fleet requirement of 43
boats. The entire water transport project will be fully operational by 2019. In Phase II, it
is proposed to operate 9 routes with a total of 16 routes and a fleet requirement of 78
boats.
 The total cost of the Water Metro project is Rs 820 crore. The KfW Development Bank
will be providing EUR 85 million as long term soft loan and the State Government of
Kerala will be contributing Rs 102 crore.
 Total cost of the project is estimated at Rs 747 crore; it excludes Rs 72 crore, which
would be required for land acquisition. The Indo-German Bilateral Cooperation, under
the ‘Climate Friendly Urban Mobility Plan’, will finance the project

Links:
https://www.indiatoday.in/education-today/gk-current-affairs/story/water-metro-project-331174-
2016-07-25

Assembly Election in 5 States


Question: Consider the following statements regarding Assembly Elections in Rajasthan,
Madhya Pradesh, Chhattisgarh, Telangana and Mizoram:

1. Congress-I emerged as the largest party in Madhya Pradesh with 114 seats.
2. BJP with 78 seats was at second place in Rajasthan Assembly Election.
3. Zoramathanga is currently serving as the 5th Chief minister of Mizoram
Of the above correct statement/s is/are:

(a) Only 1 (b) Only 1 & 3 (c) Only 2 & 3 (d) Only 3
Answer: (b)
Related facts:

 General Elections Results to the State Legislative Assemblies of Chhattisgarh, Madhya


Pradesh, Mizoram, Rajasthan and Telangana, 2O18 were announced by Election
Commission of India on 11th December, 2018.

65
http://www.edristi.in/
 The Assembly Elections in these states were held between November 12 and December
7,2018.
 2018 Assembly Elections in Chhattisgarh:
 The Chhattisgarh Legislative Assembly elections for a total of 90 Assembly seats were
held in two phases: the first phase of elections for 18 seats in South Chhattisgarh took
place on 12 November, 2018, and the second phase of elections for the remaining 72
held on 20 November 2018.
 The Indian National Congress by winning 68 seats emerged as the largest party while its
opponent Bhartiya Janata Party secured 15 seats. Janta Congress Chhattisgarh (J) and
Bahujan Samaj Party won 5 and 2 seats respectively.
 On 17th December 2018 Bhupesh Baghel sworn in as the 3rd Chief Minister of
Chhattisgarh in the state capital Raipur.
 2018 Madhya Pradesh Legislative Assembly Elections:
 The Madhya Pradesh Legislative Assembly elections for a total of 230 Assembly seats
were held on 28th November, 2018.
 The Congress-I emerged as the largest party by winning 114 seats while its opponent
BJP won 109 seats. Bahujan Samaj Party, Samajwadi Party and Independents secured
2, 1 and 4 seats respectively.
 On 17th December, 2018, Kamal Nath was sworn in as the 18th Chief Minister of
Madhya Pradesh in the state capital Bhopal.
 2018 Assembly Elections in Rajasthan:
 The Rajasthan Legislative Assembly elections for a total of 200 Assembly Seats were
held in single phase on 7th December, 2018.
 The Congress-I emerged as the largest party by winning 99 seats while its opponent
BJP won 73 seats. Bahujan Samaj Party, Rashtriya Loktantrik Party and Independents
secured 6, 3 and 13 seats respectively. Communist Party of India (Marxist) and Bhartiya
Tribal Party won 2 seats each while Rashtriya Lok Dal won 1 seat.
 On 17th December, 2018, Ashok Gehlot sworn in as the 12th Chief Minister of
Rajasthan in the state capital Jaipur.
 2018 Mizoram Pradesh Legislative Assembly Elections:
 The Mizoram Legislative Assembly elections for a total of 40 Assembly seats were held
in on 28th November, 2018.
 Mizo National Front emerged as the largest party by winning 26 seats whiles its
opponent Congress-I won 5 seats.
 Bharatiya Janata Party and Independent secured 1 and 8 seats respectively.
 On 15th December, 2018, Zoramathanga was sworn in as the 5th Chief Minister of
Mizoram in the state capital Aizawl. Prior to this, he had served as the Chief Minister of
Mizoram from December 1998 to December 2008, for two consecutive terms.
 2018 Assembly Elections in Telangana:
 The Telangana Legislative Assembly elections for a total of 119 Assembly Seats was
held on 7th December, 2018 to constitute the second Legislative Assembly since the
formation of the State in 2014.
 Telangana Rashtra Samithi by winning 88 seats emerged as the largest party while its
opponent Congress-I secured 19 seats All India Majlis-E-Ittehadul Muslimeen and
Telugu Desam won 7 and 2 seats respectively. Bharatiya Janata Party, All India Forward
Bloc and Independent secured 1 seat each.
 Kalvakuntla Chandrashekar Rao (abbreviated as K.C.R.) founder of the Telangana
Rashtra Samithi, the previous Chief Minister of Telangana was re-elected as the Chief
Minister.

Links:
http://eciresults.nic.in/

66
http://www.edristi.in/
International
UNGA adopts Russian resolution on International Information Security
Question: Recently United Nations General Assembly (UNGA) adopted a resolution on
International Information Security initiated by which of the following country?
(a) USA (b) China (c) India (d) Russia
Answer: (d)
Related facts:

 Russia came up with a resolution on international information


security (IIS) which has been adopted by United Nations
General Assembly (UNGA) on 5th December 2018, with an
objective to provide efficient solutions for information security.
 For strengthening of IIS, new innovative decisions have been taken with the aim to
protect the interests of all countries regardless of their level of technological
development.This document is the world’s first code of conduct in the digital sphere
includes a provisional list of 13 international rules, norms and principles of responsible
behavior of states in this globe.
 Russia & India had combined their efforts so as to achieve egalitarian, comprehensive
and transparent process on international information security by using information and
communications technologies. The resolution was supported by several countries and
co-authored by over 30 countries & opposed mainly by United States, countries of
European Union.

Links:
https://www.business-standard.com/article/news-ani/unga-adopts-russian-resolution-on-
international-information-security-118122801027_1.html

US, Israel withdraw from UNESCO

Question: Which of the following countries have officially disassociated itself from
UNESCO?
(a) Russia and Qatar (b) Japan and UK (c) United States and Israel (d) Iran and France
Answer: (c)
Related facts:

 On 1st January 2019, United States and its ally Israel


officially withdrew from the United Nations Educational,
Scientific and Cultural Organization (UNESCO).
 Prior to this,the two nations had announced their intention to quit the international
organization in October 2017, citing anti-Israel bias. This is the second such instance
when the US has withdrawn itself from a major multilateral commitment after President
Donald Trump took charge, first being its withdrawal from the 2015 Paris climate change
agreement. It underlines Trump’s ‘America First’ policies. This is not the first time that
the US has walked out of the UN body. The nation previously withdrew itself from the
195-member organization once before in 1984 under former president Ronald Reagan’s
administration, as it viewed the agency as mismanaged, corrupt and used to advance
Soviet interests. It was under President George W Bush that the US returned to the body
in 2002, but relations soured again in 2011 when Barack Obama pulled the plug on
funding to the body after its members voted to admit Palestine as a full member

Links
https://en.unesco.org/
67
http://www.edristi.in/
Tourism Minister co-chairs the 7th ASEAN-India Tourism Ministers
meeting
Question: Tourism Minister co-chairs the 7th ASEAN-India Tourism Ministers will meet in
which country?
(a) Vietnam (b) Thailand (c) Singapore (d) Philippines
Answer: (a)
Related facts:

 On 18 January 2019, Union Minister for Tourism, Shri. K.


J. Alphons attended the Seventh Meeting of Tourism
Ministers of ASEAN and India held in Ha Long City, Viet Nam.
 K J Alphons co-chaired the Tourism Ministers’ meeting with the Minister of Culture,
Sports and Tourism of Viet Nam, Mr. Nguyen Ngoc Thien.
 The Tourism Ministers of Brunei Darussalam, Cambodia, Indonesia, Lao PDR, Malaysia,
Myanmar, Philippines, Singapore and Thailand also attended the meeting.
 During the meeting, the Ministers welcomed the decisions of the ASEAN-India Informal
Breakfast Summit held on 15th November, 2018, in Singapore.
 Ministers launched the ASEAN-India Tourism Cooperation Year 2019, and looked
forward to enhancing cooperation with a view to increasing two-way tourist visits and
promoting greater people-to-people exchanges between ASEAN and India.

Links:
http://pib.nic.in/newsite/PrintRelease.aspx?relid=187591

New President of Brazil


Question: Who sworn in as the new President of Brazil?
(a) Michel Temer (b) Fernando Haddad (c) Jair Bolsonaro (d) Hemilton Mourao
Answer (c)
Related facts:

 On 1st January 2019, far-right candidate and former army


captain Jair Bolsonaro sworn in as new president of Brazil.
He succeeded Michel Temer.
 Mr Bolsonaro was elected with 55% of the popular vote
against 45% for Fernando Haddad from the left-wing Workers Party.
 The President of Brazil is both the head of state and the head of government of the
Federative Republic of Brazil.
 The president leads the executive branch of the federal government and is the
commander-in-chief of the Brazilian Armed Forces.

Links:
https://www.bbc.com/news/av/world-latin-america-45458730/jair-bolsonaro-four-things-
about-brazil-s-new-president

Nepal’s central bank announces ban of Indian notes above Rs 100


Question: Nepal Rastra Bank (NRB), wrote to the Reserve Bank of India (RBI) to legalize
Indian currency notes in the denominations of?
(a) Above 100 (b) Above 200 (c) Above 500 (d) Above 1000
Answer: (a)
Related facts:

68
http://www.edristi.in/
 On 6th January, 2019, The Nepal Rastra Bank (NRB), Nepal’s central monetary
authority, wrote to the Reserve Bank of India (RBI) to legalize Indian currency notes of
denominations higher than Rs 100 in Nepal.
 The NRB notified that the country’s banking system, including banks, financial
institutions and NRB, still posses Indian currency denominations of Rs 500 and Rs 1,000
worth Rs 48 million.
 The NRB has asked India’s central bank to issue a notification under the Foreign
Exchange Management Act (FEMA), which will make Indian banknotes of denomination
more than Rs 100 legal tender in Nepal and to provide the exchange facilities to get bills
of such denominations.

Links:
https://timesofindia.indiatimes.com/business/india-business/nepals-central-bank-
announces-ban-of-indian-notes-above-rs-100/articleshow/67623689.cms

Angola decriminalizes homosexuality


Question: Which country has decriminalized homosexuality?
(a) Chad (b) Niger (c) Zambia (d) Angola
Answer: (d)
Related facts:

 Angola has decriminalized homosexuality after shedding the divisive “vices against
nature” provision in its law, widely interpreted to be a ban on homosexual conduct as per
reported by Amnesty International.
 Taking things one step further, the government has also prohibited discrimination
against people on the basis of sexual orientation. And so anyone refusing to employ or
provide services to individuals based on their sexual orientation may face up to two
years in prison.
 The changes came on January 23 as Angola’s Parliament adopted its first New Penal
Code since it gained independence from Portugal in 1975 and removed the provision,
inherited from its Portuguese colonizers.
 While there have been no known prosecutions under the law, provisions like this one
curtail the rights and freedoms of lesbian, gay, bisexual and transgender (LGBT) people,
subjecting their intimate lives to unwarranted scrutiny.
 Last year Angola gave legal status to Iris Angola (country’s only gay rights lobby group)
which was established in 2013 – a move that can now be seen as a forerunner for this
latest step toward equality.

Links:
https://www.hrw.org/news/2019/01/23/angola-decriminalizes-same-sex-conduct

World Dynamic City Index 2019


Question: Name the city of India chosen as the World’s most dynamic city?
(a) Pune (b) Bengaluru (c) Mumbai (d) Hyderabad
Answer: (b)
Related facts:

 Silicon Valley of India, Bengaluru has emerged as the world’s most dynamic city.
Hyderabad, the city of pearls, has secured second place in most dynamic cities across
the world.
 Other cities in top 10 are Delhi (4th), Pune (5th) and Chennai (7th). Overall At 15th
position, Kolkata was the sixth Indian city ranked amongst top 20.This is the 6th edition
of report, City Momentum Index released by JLL.
69
http://www.edristi.in/
 The absence of European and American cities displays a marked East-west growth
divide reflecting Asia’s continued rapid urbanization and economic growth, driven by
globalization, innovation and demographic factors. The only non-Asian city on the list is
Nairobi in sixth place.
 Overall, Indian and Chinese cities dominate the rankings, accounting for three quarters
of the top 20.

JLL City momentum index

 It measures momentum for 131 of the world’s most commercial active cities.
 This is done by tracking a range of socio-economic and commercial real estate
indicators over a three year period to identify the urban economics and real estate
markets undergoing the rapid expansion.

Links:
https://www.thehindubusinessline.com/economy/bengaluru-most-dynamic-city-
hyderabad-ranked-second-jll-cmi/article26001782.ece

24th meeting of Conference of Parties (COP) 24 held in Katowice,


Poland
Question: Where did 24th meeting of Conference of Parties (COP) 24 held recently?
(a) Scotland (b) Wales (c) London (d) Poland
Answer: (d)
Related facts:

 The 24th Meeting of Conference of Parties (COP-24) to the United Nations Framework
Convention on Climate Change (UNFCCC) was organized at Katowice, Poland from 2nd
December 2018 to 14th December 2018.
 COP-24 is very important as it is expected to finalise set of rules for implementation of
the Paris Agreement adopted in 2016.
 The Indian delegation was led by Union Environment Minister Dr. Harsh Vardhan.
 Parties admitted to trigger the work on setting up the new collective finance goals post-
2020 from the floor of $ 100 billion.

Indian scenario in context of COP-24:

 India strongly supports the objective of the Paris Agreement in order to mitigate the
global hazards of climate change by keeping the temperature rise this century below 2℃
above pre-industrial levels and to seek efforts to limit the temperature increase to 1.5℃.
 The Agreement also aims at focussing on developing country parties’ and they need to
adapt to adverse impacts of climate change, in a manner that it does not put an extra
burden on them.
 India exhibited the same spirit of commitment and leadership as shown in the past.
 India’s expectation from COP-24 is that the decisions should be in conformity with the
underlying principles of UNFCCC, its Kyoto Protocol and Paris Agreement including
equity, CBDR-RC, and climate justice.

Links:
http://pib.nic.in/newsite/PrintRelease.aspx?relid=186071

70
http://www.edristi.in/
Future for Work Report
Question: Consider the following statements in reference to The Future for Work report
released recently and choose the correct options:

1) The report is released by ILO (International Labor Organisation).

2) The report is released on how to achieve a future of work that provides decent and
sustainable work opportunities for all.

3) The report also mentions that around 190 million people in the world are
unemployed.

4) The report is prepared by ILO Global Commission on Future of Work.

Correct options:
(a) 1, 2 and 3 (b) 1, 3 and 4 (c) 1 and 4 (d) All of the above.
Answer: (d)
Related facts:

 UN`s International Labour Organisation (ILO) released a report on Future of Work in


Geneva on January 22, 2019. The report is prepared by ILO Global Commission on
Future of Work on how to achieve a future of work that provides decent and sustainable
work opportunities for all. The report also contains the challenges and threats to the
economy in the coming time and the present scenario of unemployment.
 The report posed the need of decisive action is needed to tackle the widening inequality,
increasing uncertainity and reinforcing exclusion with destructive political, social and
economic repercussions.

About the Report:

 The report is a result of 15 months review by a 27 member which was co-chaired by


Swedish PM Stefan Lofven and South African President Cyril Ramaphosa.
 As per ILO, world`s 190 million population faces unemployment, while 300 million live in
the condition of extreme poverty. It is also mentioned that wage gap is increasing while
wage growth is declining.
 It is also noted that only 15% of households in emerging countries have access to
internet while Two-thirds of jobs in developing countries are susceptible to automation.
 The proposed transition to greener economy as per Paris Climate Agenda would lead to
job loss of around 6 million workers after the promise of creating 24 million new jobs.
 The report mentions the need to setup an international governance system for policing of
gig economy and ensuring following of minimum rights and protections by online
platforms like Uber and Swiggy.
 The report also recommends development of rural economy as priority where the future
of many of workers lie. It also urged to ensure a universal labour guarantee for a living
wage, limiting working hours and work safety norms.

About Global Commission on Future of Work:

 This formation of ILO marks the second stage in the ILO Future of Work Initiative. Its job
is to undertake an in-depth examination of the future of work that can provide the
analytical basis for the delivery of social justice in the 21st century.
 The commission also identifies the key challenges facing the world of work and making
practical recommendations about how these may be addressed in the future.

71
http://www.edristi.in/
 The Global Commission is composed of 28 members, all outstanding individuals,
eminently equipped to contribute to its work, reflecting a balance of gender, tripartite
representation, geographical regions, and representing diverse disciplines and sectors of
society and the economy.

Links:
https://www.thehindu.com/news/national/ilo-urges-universal-labour-
guarantee/article26063573.ece

Edelman Trust Barometer 2019


Question: In the recent released Edelman Trust Barometer 2019, which country topped
the trust index among the informed public and general population
segment?
(a) India (b) Japan (c) China (d) France
Answer: (c)
Related facts:

 The Edelman Trust Barometer is released by Edelman Intelligence just ahead of World
Economic Forum (WEF) on 21 January, 2019. The latest report ranked China on top of
the trust index among the informed public and general population segment.

Key Highlights:

 The overall Global Trust Index has increased to 52 with 3 point increase. The report is
made on the basis of an on line survey done in 27 markets covering over 33,000
respondents.
 India is one of the most trusted nations globally in government, business, non-
governmental organisations (NGOs) and media. The country`s brands are least trusted
posing a worry infront.
 Switzerland, Germany and Canada are the most trusted in terms of the trust in
companies headquartered in each market with a trust score of 70. Japan is second with
69 percent.
 The report also mentions that companies with headquarters in India, Mexico and Brazil
are least trusted, followed by China and South Korea.
 The score of India and Brazil is 40% and of China is 41 % respectively.
 There are 14 markets in which majority of mass population do not believe that they will
be better off in five years.
 75 percent of people trust My Employer to do what is right, significantly more than NGOs
(57%) and media (47%).
 Only one in five feels that the system is working for them and around half of population
believes that the system is falling them.

Links:
https://www.thehindubusinessline.com/economy/india-among-the-most-trusted-nations-
globally-report/article26051565.ece

Global Aviation Summit, 2019


Question: Global Aviation Summit, 2019 will be held in which city?
(a) Mumbai (b) Delhi (c) Hyderabad (d) Chennai
Answer :(a)
Related facts:

72
http://www.edristi.in/
 The Ministry of Civil Aviation, Government of India, in collaboration with FICCI has
organized first of its kind Global Aviation Summit on 15-16 January, 2019 in Mumbai.
 The theme of the summit was “Flying for all – especially the next 6 Billion”.
 The two-day summit hosted a conference, expo and endeavored meaningful
engagements via G2G, G2B, B2B meetings and other networking opportunities.
 Global Aviation Summit provides a platform to the aviation fraternity to explore
opportunities, discuss the challenges facing the sector and understand how technology-
driven innovations will change air travel in the future.
 The Summit is the most comprehensive platform for civil aviation, witnessing a vivid
representation from the global aviation ecosystem entailing global leaders from airlines,
manufacturers, investors, vendors, cargo, space industry, banking institutions, skill
development agencies and states & UTs of India.
 The International Civil Aviation Organization has predicted a 100 percent increase in
global air travel by the year 2030, creating a need for extensive efforts to bring the entire
aviation ecosystem to pace.

Links:
https://www.globalaviationsummit.in/about.php

14th UN Conference on Land Degradation and Desertification


Question: Which South Asian country will host the 14th UN Conference on Land
Degradation and Desertification?
(a) Bangladesh (b) Nepal (c) India (d) Sri Lanka
Answer: (c)
Related facts:

 The 14th session of the Conference of Parties (COP -14) to United Nations Convention
to Combat Desertification (UNCCD) will take place in India from 7-18 October, 2019 at
Vigyan Bhawan in New Delhi.
 Earth Observation data on the trends in land degradation dating from 2000, gathered
from 120 of the 169 countries affected by desertification will be tabled. The first report on
desertification and climate change prepared by the Intergovernmental Panel on Climate
Change (IPCC), the global authority on climate change will be also received.
 Participants from 197 Parties to the United Nations Convention to Combat
Desertification (UNCCD) attending the Conference can assess trends in land
degradation, desertification and drought more accurately.
 Ahead of COP14, government representatives will gather for a preparatory meeting from
28-30 January in George Town, Guyana.
 The government is committed to achieve land degradation neutrality by 2030 under the
Sustainable Development Goals target 2030.
 The last Conference of Parties, hosted by the government of China, was held in October
2017 in Ordos, Inner Mongolia.
 The UNCCD was established in 1994 in Paris, France and entered into force in
December 1996. It is the only internationally legally binding framework set up to address
the problem of desertification

Links:
https://www.unccd.int/news-events/india-hosts-next-un-conference-land-degradation

Henley Passport Index


Question: Which country topped the list of the latest released Henley Passport Index
2019?
(a) USA (b) Japan (c) United Arab Emirates (d) Finland
73
http://www.edristi.in/
Answer: (b)
Related facts:

 Henley & Partners released its quarterly revised Passport Index of 2019 in which Japan
is ranked as the country with most powerful passport in the world. Japan retained its no.
1 spot in the rankings as it the friendliest passports with access of 190 countries.
 India is ranked 79th with access to 61 countries. China ranked of 69 with access to 74
countries this year.
 Singapore as well as South Korea (both Asian) are ranked second joint with access to
189 countries. Asian countries have performed significantly well as compared to others.
Other countries in Top Five includes France & Germany (both 3rd), Denmark, Finland,
Italy, Sweden (all 4th) and Luxembourg, Spain (both 5th).
 The worst rankings were given to Afghanistan and Iraq for having access to only 30
countries. The rankings of USA & UK got dropped to 6th with access to 185 countries.
 The Henley Passport Index is based on data provided by the International Air Transport
Authority (IATA) and covers 199 passports and 227 travel destinations. It is updated in
real time throughout the year, as and when visa policy changes come into effect.

Links:
https://www.henleypassportindex.com/global-ranking

Democracy Index
Question: Which rank does India have in the latest Democracy Index released by the
Economist Intelligence Unit (EIU)?
(a) 21st (b) 31st (c) 41st (d) 51st

Answer: (c)
Related facts:

 The latest list of Democracy Index was released by the Economist Intelligence Unit (EIU)
ranking 167 countries by 60 indicators across five categories which are electoral process
and pluralism, the functioning of government, political participation, democratic political
culture and civil liberties. India ranked 41st in the list with a score of 7.3/10 and classified
as Flawed democracy.
 The report has 4 classifications of democracies terming them as Authoritarian regime
(scoring from 0 to 4), Hybrid regime (4 to 6), Flawed democracy (6 to 8), and Full
democracy (8 & above).

Facts & findings of report:

 This overall puts a positive outlook of global democracy with improvement in women`s
participation and willingness of people to engage in lawful demonstrations despite a
growing disillusionment with formal political institutions.
 Only 4.5% of the world lives in a full democracy. 42 countries experienced a decline
while 48 showed improvement. The report also mentions that democracy has decreased
in Europe and many other advanced democracies.
 The report mentions the threat to democracies around the world have become more
obvious: frizzled Arab Spring, lifetime rule adoption in China, Populists with autocratic
tendencies have won elections in the Philippines, Brazil and Mexico and subverted
democratic institutions in Hungary, Turkey and Poland.
 Political participation showed most improvement as compared to any other measure in
the Index.

Rankings:
74
http://www.edristi.in/
 The index ranked a total of 167 countries in which Norway topped the list with a score of
9.87 while North Korea ranked last with a score of 1.08.
 Japan is ranked 22nd while USA is ranked 25th. Australia and New Zealand are the only
Full Democracies in the whole Asia-Pacific region.
 Indian neighbours Bhutan, Nepal, China and Afghanistan were ranked 94th, 97th, 130th
and 143rd respectively.

Links:
http://www.eiu.com/topic/democracy-index

New Non Permanent Members of UN Security Council


Question: Which of the following statement/s is/are correct in reference to recent events
associated with UNSC?

1) Five new members namely Belgium, Dominican Republic, Germany, Indonesia and
South Africa are appointed as the new non-permanent members of the UNSC

2) UNSC is composed of total 10 members including five non permanent members

3) The Presidency of the Council is given to the Dominican Republic

4) The Presidency is rotated every month and the tenure of non-permanent members
last for 1 year

Correct option:
(a) 1, 3 and 4 (b) 2, 3 and 4 (c) 1 and 3 (d) All of the above.
Answer: (c)
Related facts:

 On 3 January 2019, Belgium, Dominican Republic, Germany, Indonesia and South


Africa were appointed new non-permanent members of the United Nations Security
Council for the period of 2 years. Dominican Republic is elected the President of the
UNSC.
 The outgoing members are; Bolivia, Ethiopia, the Netherlands, Kazakhstan and Sweden.

United Nations Security Council:

 The United Nations Security Council is the most powerful body of United Nations,
responsible for maintaining International peace and security as well as accepting new
membership to the UN. It is one of the 6 main organs of UN.
 It is constituted of 15 member countries having 5 permanent members with veto power
and 10 non-permanent member countries.

Election process:

 With the Article 23 of Charter, amended in 1963 and enforced in 1965, General
Assembly increased the number of members of non-permanent countries from 6 to 10.
With Rule 142 of the rules of procedure, Assembly elects 5 non-permanent members
each year.
 The non permanent members have 5 African and Asian states each, 1 Eastern
European state, 2 Latin American states and 2 Western European and other states.
Non-permanent members are elected by a secret ballot with two-third majority.

75
http://www.edristi.in/
Links:
https://www.washingtonpost.com/world/europe/un-security-council-gets-new-members-and-1-
gets-presidency/2019/01/02/9bb995ae-0eae-11e9-8f0c-
6f878a26288a_story.html?noredirect=on&utm_term=.e3e3e69d25a7

Japan withdrew from International Whaling Commission


Question: Which of the following statement/s is/ are correct about International Whaling
Commission?

1) International Whaling Commission was setup in Washington in 1946.

2) It main purpose is to provide proper conservation of whales.

3) India is not a member of IWC

4) Recently Japan withdrew from IWC and to resume commercial whaling by July
2019.

Correct option is:


(a) 1, 2 and 3 (b) 1, 3 and 4 (c) 1,2 and 4 (d) All of the above.
Answer: (c)
Related facts:

 Recently Japan announced its withdraw from IWC (International Whaling Commission).
This step comes after continuous argument from Japan over failure of IWC in achieving
dual goal of preservation as well as orderly development of whaling industry.
 Japan will resume hunting whales in its own territorial waters and its exclusive economic
zones from June next year. Japan joined IWC in 1951.
 Prior to this, with the sharp decrease in whale population, IWC agreed a moratorium on
commercial whaling in 1986 which is binding on member countries. This helped the re-
emergence of healthy population of whales and saving several species from extinction.
 Japan, Iceland and Norway however continued to hunt whale. Japan did it on the name
of scientific research. Whale meat is a rich source of protein in Japan which has a rich
seafood culture. Whale meat is also distinguished with the traditional culture of Japan.
 IWC was established in Washington DC on 2nd December 1946 to provide proper
conservation of whale stocks as well as orderly development of whaling industry. India
joined the IWC in 1981.

Links:
https://www.washingtonpost.com/world/japan-to-leave-international-whaling-commission-
resume-commercial-hunt/2018/12/26/2c32fb20-08c9-11e9-892d-
3373d7422f60_story.html?noredirect=on&utm_term=.f18713b5268c

Marijuana Legalisation
Question: Which country recently approved use of marijuana for medical purposes?
(a) India (b) Canada (c) Thailand (d) Slovenia
Answer: (c)
Related facts:

 The military appointed National Legislative Assembly of Thailand legalized usage of


marijuana (cannabis) for medical purpose, first country in Southeast Asia to make this
law.
76
http://www.edristi.in/
 The amendment in drug law will allow the use of cannabis and kratom, a locally grown
plant used as stimulant and painkiller.
 Marijuana can be useful in treating a wide range of conditions, including glaucoma,
epilepsy, chronic pain and the side effects of chemotherapy.
 With this step Thailand joins the list of countries like Colombia, Canada, California in
United States and Britain which allows use of marijuana for medical purposes.
 It is worth to mention that recreational use of the drugs remains illegal in Thailand and is
subjected to prison terms and fines proportional with the quantities involved.

Links:
https://www.aljazeera.com/news/2019/01/led-coalition-syria-withdrawal-begun-
190111092622492.html

India stood at 12th rank in the misperception index


Question: Recently India ranked …….. in the misperception index.
(a) 11th (b) 12th (c) 13th (d) 14th

Answer: (b)
Related facts:

 According to the Market & Opinion Research International Perils of Perception Survey
2018, done by Ipsos, India ranked 12th out of the 37 countries surveyed in the
“misperception index”, behind numerous other countries like, Italy, Singapore, Hong
Kong, South Korea, Chile and Argentina, among others.
 Approximately 1,000 people aged between 16 and 64 were surveyed in India.
 Some of the Core topics on which Indians are misinformed are Economy, Immigration,
Sex, Unemployment & the Population of Muslims.
 Few other topics where Indians remain ill-informed include global warming, the country’s
demographic dividend, and the health of its citizens.

Links:
https://www.business-standard.com/article/news-ians/india-perceives-poorly-12th-on-
misperception-index-118122101276_1.html

World Press Freedom Index 2018


Question: Which country is at the first spot in the World Press Freedom Index 2018
published by Reporters Without Borders (RSF)?
(a) Netherlands (b) Sweden (c) Iceland (d) Norway
Answer: (d)
Related facts:

 The World Press Freedom Index 2018 compiled by Reporters Without Borders (RSF)
features India at 138th rank slipped down from 136th in 2017.
 In the published list,Norway holds top spot followed by Sweden,Netherlands while North
Korea is the most threatening place(180th rank) for journalists.
 World Press Freedom Index published by RSF annually since 2002 measures the level
of media freedom in 180 countries.
 Out of the 80 journalists killed worldwide in 2018, 49 were meticulously targeted
because their reporting posed a threat to the interests of certain people in positions of
political, economic, or religious power or organised crime.
 Out of 49 journalists killed, 15 journalists were killed in Afghanistan, 11 journalists were
killed in Syria, 9 in Mexico, 8 in Yemen, and 6 each in India and the United States. Some
positive news from Iraq was also featured where no journalist was killed for the first time.
77
http://www.edristi.in/
 The United States of America shares the spot for being the fifth most-deadliest place for
journalists with India & is behind Afghanistan, Syria, Mexico and Yemen.
 It is worth to note that World Press Freedom Day is observed on 3 May every year to
raise awareness of the importance of freedom of the press.

Links:
https://rsf.org/en/ranking_table

Inclusive Wealth Report 2018


Question: Consider the following statements regarding Inclusive Wealth Report 2018:

1. It evaluates the capacities and performances of the nations around the world to
measure sustainability of economy and well-being of their people
2. It is a biennial report of the UN Environment
Select the correct statements

(a) 1 only (b) 2 only (c) Both 1 and 2 (d) Neither 1 nor 2
Answer: (c)
Related facts:

 The Inclusive Wealth Report (IWR) is a biennial effort led by the UN Environment to
evaluate the capacities and performances of the nations around the world to measure
sustainability of economy and well-being of their people.
 The (IWR) index as a part of Inclusive Wealth Report measures the wealth of nations
through a comprehensive analysis of a country’s productive base and the country’s
inclusive wealth which is the social value (not dollar price) of all its capital assets,
including natural capital, human capital and produced capital.
 The report finds Republic of Korea, Singapore and Malta as the nations which have had
the most economic growth.
 The Inclusive wealth (IW) in 135 countries was higher in 2014 compared to the level in
1990 and the global growth rate of IW was 44% over the indicated period, which implies
an average growth rate of 1.8% per year. However, during the same period the global
GDP growth per year was 3.4%, which is close to two-fold of the annual growth rate of
growth in IW.
 While 44 of 140 countries (including India) are seeing their GDP rise, their inclusive
wealth index is registering a decline.
 In India when the average rate of Gross State Domestic Product (GSDP) during 2005-15
for almost all the states was around 7-8%, 11 states registered a decline in their natural
capital.

UN Environment:

 The United Nations Environment Programme (UN Environment) is the leading global
environmental authority that sets the global environmental agenda, promotes the
coherent implementation of the environmental dimension of sustainable development
within the United Nations system, and serves as an authoritative advocate for the global
environment.

Links:
https://www.unenvironment.org/resources/report/inclusive-wealth-report-2018

78
http://www.edristi.in/
New Capital of Burundi
Question: Which is named as the new capital of Burundi?
(a) Rutana (b) Karuzi (c) Gitega (d) Rumonge
Answer: (c)
Related facts:

 As per a presidential promise made by Pierre Nkurunziza decade ago, a small central
city of Gitega is named as the political Capital of Burundi.
 Previous capital Bujumbura will solely serve as the economic center of country.
 Burundi is a small landlocked country in central Africa. It was a colony of Germany and
later Belgium from which it gained independence in 1962 and became a republic in
1966. It is one of the poorest countries in the world having higher level of poverty,
unemployment and economic crisis as well as severe civil war and genocide.
 It is worth to note that Burundi is the least happy country (156th rank) in World
Happiness Report 2018 while Finland tops the list.India is at 133th rank in the published
list.

Links:
https://www.msn.com/en-xl/africa/top-stories/burundi-names-gitega-as-new-capital/ar-BBRn6ba

Israel becomes a member of the FATF


Question: Which country has recently been made the 38th member of ‘The Financial
Express Task Force’ (FATF)?
(a) Japan (b) South Korea (c) Iran (d) Israel
Answer: (d)
Related facts:

 On 10th December, 2018, Israel became the 38th member of ‘The Financial Action Task
Force’ (FATF). Israel was an observer to the FATF in February 2016.
 Until then it had already been closely involved in the work of the FATF through its
participation in the FATF-style regional body MONEYVAL.
 The Financial Action Task Force (FATF) is an inter-governmental body to set standards
and promote effective implementation of legal, regulatory and operational measures for
combating money laundering, terrorist financing and other related threats to the integrity
of the international financial system.
 Founded in 1989 as an initiative of the G7 countries—Canada, France, Germany, Italy,
Japan, the U.K., and the U.S. Members in this organization (FATF) represent the largest
financial centers around the world.
 FATF develops policies to combat money laundering and terrorism funding. India is the
member of this organization.

Links:

http://www.fatf-gafi.org/publications/fatfgeneral/documents/israel-fatf-member.html

Global debt reaches an all-time high


Question: What is the amount of global debt figured by International Monetary Fund
recently?
(a) 184 trillion dollars (b) 160 trillion dollars (c) 125 trillion
dollars (d) 105 trillion dollars

79
http://www.edristi.in/
Answer: (a)
Related facts:

 On 13th December, 2018, in its report, International Monetary Fund (IMF) figured the
global debt to 184 trillion dollars remarking this to be at its peak level. In the beginning of
this year, the IMF calculated the global debt of 182 trillion dollars.
 In the estimated sum more than half the stake belongs to America, China and Japan.
 The updated loan amount of 184 trillion dollars is equivalent to 225 percent of global
GDP in 2017.
 On average, the world’s debt now exceeds USD 86,000 per person. This figure is more
than two times the average per capita income.
 At the beginning of the millennium, China’s share of global debt was less than three
percent, which now has more than 15%.

Links:

https://www.thehindubusinessline.com/economy/global-debt-hits-all-time-high-of-usd-184-
trillion-imf/article25740582.ece

First-Ever Global Compact on Migration


Question: First ever Global Compact on Migration has been adopted at which of the
following UN Convention?
(a) Morocco (b) UAE (c) Venezuela (d) Austria
Answer: (a)
Related facts:

 On 10th December, 2018, leaders and delegates of 160 countries participated in a


conference supported by the United Nations in Morocco.
 Global Compact for Safe, Orderly and Regular Migration was adopted in this conference.
 Comprised of 23 objectives — all of which contain a commitment — the Global Compact
on Migration outlines actions considered to be relevant policy instruments and best
practices alongside measures for implementation, follow-up and review every four years,
beginning in 2022.
 This agreement is based on the objectives and principles of the United Nations Charter.
 It is important that this global agreement has been adopted for safe, systematic and
regular migration after discussion and consultation between United Nations member
countries, local officials, civil society and migrants over a year.

Related Provisions:

 On 18th December, 1990, the United Nations General Assembly adopted an international
conference on the rights of all migrant workers and the protection of their family
members.
 In view of the number of growing migrants worldwide, on December 4th, 2000, the UN
General Assembly announced to celebrate 18th December, as International Migrants
Day.

Links:

https://www.un.org/press/en/2018/dev3375.doc.htm

80
http://www.edristi.in/
Economics
Common Digital Currency ‘Aber’
Question: Common Digital Currency ‘Aber’ launched by which country?
(a) UAE and Saudi Arabia (b) UAE and Qatar (c) Oman and Qatar (d) Saudi and Oman
Answer: (a)
Related facts:

 The United Arab Emirates Central Bank (UAECB) and the


Saudi Arabian Monetary Authority (SAMA) announced the
launching of common digital currency project ‘Aber’ on
January 29, 2019.
 The currency will be used in financial settlements between the two countries through
Blockchains and Distributed Ledgers technologies..
 This comes under the framework of ‘Proof-of-Concept’, which can be summed up in
closely understanding and studying the dimensions of modern technologies
 The project will study the feasibility of the currency and will determine its impact on
reducing remittances costs alongwith the technical risks.
 It will establish an additional means for the central financial transfer systems of the two
countries and enable banks to directly deal with each other in conducting financial
remittances.
 The use of the currency will be restricted to a limited number of banks initially which will
be widened further in future.

Links:
https://gulfbusiness.com/uae-saudi-reveal-details-common-digital-currency-aber/

Cabinet approves the Amendment to the ‘Framework on Currency Swap


Arrangement for SAARC Member Countries’
Question: What amount Cabinet approved for Amendment to the ‘Framework on
Currency Swap Arrangement for SAARC Member Countries’ to incorporate ‘Standby
Swap’?
(a) USD 200million (b) USD 300 million (c) USD 400 million
(d) USD 500 million
Answer: (c)

 The Union Cabinet has given its ex-post facto approval


for amendment to the Framework on Currency Swap
Arrangement for SAARC Member Countries to
incorporate a Standby Swap amounting to USD 400 million to be operated within the
overall size of the facility of USD 2 billion and build in flexibility with respect to modalities
of its operation, such as period of swap and roll over.
 Due to heightened financial risk and volatility in the global economy, short term swap
requirements of SAARC countries could be higher than the agreed lines.
 The incorporation of ‘Standby Swap’ within the approved SAARC Swap Framework
would provide the necessary flexibility to the framework and would enable India to
provide a prompt response to the current request from SAARC member countries for
availing the swap amount exceeding the present limit prescribed under the SAARC
Swap Framework.
 The Cabinet had previously approved the Framework on Currency Swap Arrangement
for SAARC Member Countries on March 1, 2012 with the intention to provide a line of
funding for short term foreign exchange requirements or to meet the balance of

81
http://www.edristi.in/
payments crises till long-term arrangements are made or the issue is resolved in the
short-term itself.
 Under the Facility, RBI offers swaps of varying sizes in USD, Euro or INR to each
SAARC member country depending on their two months import requirement and not
exceeding US$ 2 billion in total.

Links:
https://www.thehindubusinessline.com/economy/cabinet-clears-400-m-standby-swap-
facility-for-saarc-nations/article26072299.ece

Andhra Pradesh Tops ACI’s Ease of Doing Business Index

Question: Which State/UTs topped the ACIs Ease of Doing Business Index?
(a) Maharashtra (b) Delhi (c) Andhra Pradesh (d) Telangana
Answer (c)
Related facts:

 Andhra Pradesh emerged as the top state in 2018 Ease of


Doing Business (EDB) Index compiled by Singapore based
Asia Competitiveness Institute (ACI) for economies of 21
states of India. Maharashtra is at second spot while Delhi occupies third place.
 EDB index is based upon three parameters called ABC – Attractiveness to Investors,
Business Friendliness and Competitiveness Policies.
 The state rose from 6th to the 1st position in the Business Friendliness, and from the 7th
rank in 2016 to the 3rd rank in 2018 in Attractiveness to Investors.
 However the state dipped from 2nd to 4th rank in Competitiveness Policies. In total
Andhra Pradesh rose from the 5th rank in 2016 to the top spot in 2018.
 The report mentioned that despite Andhra Pradesh’s excellent performance in ACI’s
EDB Index, the government needs to bridge numerous institutional and infrastructural
gaps in order to sustain its position in the future.
 Asia Competitiveness Institute in Singapore was established as a research centre in
2006 with an aim to build intellectual leadership and network for understanding and
developing competitiveness in the Asia region.

Links:
https://www.business-standard.com/article/news-ani/andhra-tops-aci-s-ease-of-doing-
business-index-119010400113_1.html

RBI eases ECBs norms


Question: Consider the following statements regarding External Commercial Borrowings
(ECBs):

1. ECBs is basically a loan availed by an Indian entity from


a non-resident lender

2. Maturity period for ECBs is up to five years


Of the above choose the correct options:

(a) 1 only (b) 2 only (c) Both 1 and 2 (d) Neither 1 nor 2
Answer: (c)
Related facts:

82
http://www.edristi.in/
 Reserve bank of India has decided to reform external commercial borrowings (ECBs)
norms, allowing all companies that are eligible for receiving foreign direct investment, to
raise funds through the ECB route.
 The list of eligible borrowers has been expanded.
 The RBI has decided to keep the minimum average maturity period at three years for all
ECBs, regardless of the amount of borrowing, except for borrowers specifically permitted
to borrow for a shorter period.
 The minimum average maturity period was five years before 2019. The ceiling for
borrowing remains at $750 million.
 All eligible borrowers can now raise ECBs up to $750 million or similar per financial year
under the automatic route replacing the existing sector-wise limits.
 This change increases lending options and allows various new lenders in ECB space
while strengthening the (anti money laundering / combating the financing of terrorism)
framework.
 RBI had capped funds via ECBs at 6.5% of GDP, at current market prices. Based on
GDP figures for March 31, 2018, the Soft limit works out to $160 billion.
 ECB is a basically a loan availed by an Indian entity from a nonresident lender.
 Most of these loans are provided by foreign commercial banks and other institutions.

Links:
https://www.thehindu.com/business/Industry/ecb-norms-eased/article26006851.ece

Minimum wages in MGNREGA


Question: On 7th January 2019, the recommendations of the Committee selected for
review Minimum wages in MGNREGA are under consideration & is in consultation with
which department of the government?
(a) Ministry of Finance (b) Ministry of Rural development
(c) Ministry of Defence (d) Ministry of Human resource & development
Answer: (a)
Related facts:

 Wage rates for workers under the Mahatma Gandhi


National Rural Employment Guarantee Act (MGNREGA),
2005 are revised annually based on Consumer Price
Index-Agricultural Labourers (CPI-AL) by the Central Government. The
recommendations of the Committee are under consideration & are in consultation with
the Ministry of Finance.
 Wages paid are based on measurement of work done i.e. piece rate basis and every
state has its own defined schedule of rates based on which the work output is defined
and used to calculate the wage of MGNREGA beneficiaries.
 The Committee recommended using Consumer Price Index-Rural instead of the existing
CPI-AL for revising MGNREGA wages every year. The Committee also recommended
use of annual average instead of the existing practice of using December month index
only.

Links:
http://pib.nic.in/newsite/PrintRelease.aspx?relid=187258

National Bench of the Goods and Services Tax Appellate Tribunal


Question: Where will the recently approved creation of National Bench of the Goods and
Services Tax Appellate Tribunal (GSTAT) be situated?
(a) New Delhi (b) Mumbai (c) Noida (d) Ghaziabad

83
http://www.edristi.in/
Answer: (a)
Related facts:

 The Union Cabinet, chaired by the Prime Minister Narendra Modi, has approved the
creation of National Bench of the Goods and Services Tax Appellate Tribunal (GSTAT).
 The National Bench of the Appellate Tribunal shall be situated at New Delhi. GSTAT
shall be presided over by its President and shall consist of one Technical Member
(Centre) and one Technical Member (State).
 The creation of the National Bench of the GSTAT would cost Rs.92.50 lakh while the
recurring expenditure would be Rs.6.86 crore per annum.

Goods and Services Tax Appellate Tribunal:

 It is the forum of second appeal in GST laws and the first common forum of dispute
resolution between Centre and States. The appeals against the orders in first appeals
issued by the Appellate Authorities under the Central and State GST Acts lie before the
GST Appellate Tribunal, which is common under the Central as well as State GST Acts.
 Being a common forum, GST Appellate Tribunal will ensure that there is uniformity in
redressal of disputes arising under GST, and therefore, in implementation of GST across
the country.
 Chapter XVIII of the CGST Act provides for the Appeal and Review Mechanism for
dispute resolution under the GST Regime. Section 109 of this Chapter under CGST Act
empowers the Central Government to constitute, on the recommendation of Council, by
notification, with effect from such date as may be specified therein, an Appellate Tribunal
known as the Goods and Services Tax Appellate Tribunal for hearing appeals against
the orders passed by the Appellate Authority or the Revisional Authority.

Links:
http://pib.nic.in/newsite/PrintRelease.aspx?relid=187692

World Economic Situation and Prospects 2019


Question: Who released the report World Economic Situation and Prospects 2019?
(a) World Bank (b) IMF (c) United Nations (d) World Economic Forum
Answer: (c)
Related facts:

 The United Nations released the report World Economic Situation and Prospects 2019.
The report came up with major findings on economic situations in the world alongwith
prospects. It was launched in New York.

Major findings:

 The World Economic Situation and Prospects (WESP) 2019 report, cautioned the steady
pace of expansion in the global economy masks an increase in downside risks that could
potentially exacerbate development challenges in many parts of the world.
 The world economy is estimated to have grown by 3.1 per cent in 2018, as a result of
fiscally induced growth acceleration in US while a slower growth in a few large
economies like Argentina, Canada, China, and Turkey.
 The growth rates in many developed countries have risen close to their potential, with
drop in unemployment rates to historical lows. Among the developing economies, the
East and South Asia regions are on a relatively strong growth trajectory, amid robust
domestic demand conditions.

84
http://www.edristi.in/
 Growth in global industrial production and merchandise trade volumes has been tapering
since early 2018, particularly in the trade-intensive capital and intermediate goods
sectors.
 The global growth rate projected by WESP is of 3.0 per cent in 2019 & 2020. The growth
rate in United States is projected to slow from 2.8 per cent in 2018 to 2.5 per cent in
2019 and 2 per cent in 2020.
 Uneven economic growth is a major threat to be addressed. The developments are not
reaching to the needy people. Various large developing countries saw a decline in per
capita income in 2018.
 A modest recovery is projected in 2019 but the per capita incomes are still likely to
remain stagnant in Central, Southern and West Africa, Western Asia, and Latin America
and the Caribbean.
 The global economy is facing multiple risks, which could severely disrupt economic
activity and can create damages on longer-term development prospects. These risks
include an escalation of trade disputes (like US-China), an abrupt tightening of global
financial conditions, and intensifying climate risks.
 Environmental degradation and increasing climate risk posing severe threat to the
economies of the world. More than half of extreme weather events have been attributed
to climate change in last six years.
 Climate shocks impact causes severe damage to vital infrastructure in developing and
developed countries. There have been some progress has been made in reducing the
greenhouse gas intensity of production but the growth towards environmentally
sustainable production and consumption is not happening fast enough.

Links:
https://www.un.org/development/desa/dpad/publication/world-economic-situation-and-
prospects-january-2019-briefing-no-122/

RIL Quarter Profit Record


Question: RIL became the first Indian private firm to cross the quarter profit of:
(a) 8,000 crore (b) 9,000crore (c) 10,000 crore (d) 15,000 crore
Answer: (c)
Related facts:

 Reliance Industries Limited (RIL) becomes the first Indian private sector firm to cross Rs.
10,000 crore of quarterly profit in total.
 This was achieved in the recent quarter profit of October-
December 2018 of the company. The profit of the firm rose for
the 16th straight quarter. The firm had jump in its revenue by
55.9% to Rs. 1,71,336 crore in October- December 2018.
 This is the highest profit posted by any private company.
However public sector Indian Oil Corporation (IOC) has posted
highest ever quarterly profit of Rs. 14,512.81 crore by any Indian firm in January- March
2013 quarter.
 RIL is oil to telecom conglomerate headquartered in Mumbai. The Chairman of the
company is Asia`s richest person Mukesh Ambani.

Links:
https://economictimes.indiatimes.com/markets/stocks/earnings/ril-first-private-indian-
firm-to-post-rs-10000-crore-profit-10-takeaways-from-q3-
earnings/articleshow/67574840.cms

85
http://www.edristi.in/
Global Economic Prospects-2019
Question: As per the World Bank’s Global Economic Prospects report, India’s GDP is
expected to grow —– in the fiscal year 2018-19?
(a) 7.2 (b) 7.3 (c) 7.4 (d) 7.5
Answer: (b)
Related facts:

 As per World Bank’s Global Economic Prospects (GEP) report


released on 9th January, 2019, India’s GDP is expected to
grow at 7.3 in the fiscal year 2018-19, and 7.5 in the following two years.
 The growth is defined by economic activity which continued to recover with strong
domestic demand. While investment continued to strengthen amid GST harmonization
and a rebound of credit growth, consumption remained the major contributor to growth.
 As per the report, India will continue to be the fastest growing major economy in the
world.
 Meanwhile,China’s economic growth is projected to be 6.2 each in 2019 and 2020 and 6
per cent in 2021, according to the January 2019 Global Economic Prospects report
released by the World Bank.
 Global economic growth is projected to soften from a downwardly revised 3 percent in
2018 to 2.9 percent in 2019 amid rising downside risks to the outlook, the World Bank.

About Global Economic Prospects (GEP):

 Global Economic Prospects GEP is World Bank Group’s flagship report that examines
global economic developments and prospects with special focus on developing and
emerging market economies.
 It is published biannually and partly updated two times a year. It portrays the world
economy in the near and medium context, with projections for up to four years into the
future.

Links
http://www.worldbank.org/en/publication/global-economic-prospects

5 member panel set up by RBI on concerns regarding digital payment


Question: Who has been appointed chairman of a committee to assess digitalization of
payments?
(a) Nandan Nilekani (b) Sunil Mehta (c) Raghuram Rajan (d) E. Sreedharan
Answer: (a)
Related facts:

 The Reserve Bank of India (RBI) constituted a high-level


committee headed by former chairman of the Unique
Identification Authority of India (UIDAI) Nandan Nilekani in order
to encourage digitisation of payments and enhance financial
inclusion through digitisation.
 The committee will submit its report within 90 days.
 The panel would assess the current levels of digital payments; it will suggest measures
to increase the confidence of customers by strengthening the safety and security of
digital payments. It will also suggest a medium-term strategy for deepening of digital
payments.
 Besides Aadhaar architect Nilekani, the other members of the panel are former RBI
deputy governor H.R. Khan, former MD and CEO of Vijaya Bank, Kishore Sansi, former

86
http://www.edristi.in/
secretary in the ministry of IT and Steel Aruna Sharma and chief innovation officer at the
Center for Innovation, Incubation and Entrepreneurship, IIM Ahmedabad, Sanjay Jain.

Links:
https://timesofindia.indiatimes.com/business/india-business/rbi-sets-up-panel-under-nandan-
nilekani-to-boost-digital-payments/articleshow/67437770.cms

Recapitalization of Export-Import Bank of India


Question: Recently Union Cabinet has given approval for recapitalization of which of the
following bank?
(a) IDBI (b) Punjab National Bank (c) State Bank of India (d) EXIM-Bank
Answer: (d)
Related facts:

 On 16 January 2019, Recapitalization of Export- Import Bank


was approved by the Union Cabinet, headed by the Prime
Minister Shri Narendra Modi.

Measures taken:

 Issuance of Recapitalization Bonds by Government of India to the tune of Rs.6,000 crore


for capital infusion in Export Import Bank of India (Exim Bank).
 The equity will be infused in two tranches of Rs. 4,500 crore in FY 2018-19 and Rs.1,500
crore in FY 2019-20 respectively.
 The Cabinet also approved an increase in the authorized capital of Exim Bank from Rs.
10,000 crore to Rs. 20,000 crore. The recapitalisation bonds will be on the lines issued
to Public Sector Banks.

Impact of the decision:

 Exim Bank is the principal export credit agency for India.


 The infusion of capital into Exim Bank will enable it to augment capital adequacy and
support Indian exports with enhanced ability.
 The infusion will give an impetus to anticipate new initiatives like supporting Indian textile
industries, likely changes in Concessional Finance Scheme (CFS), likelihood of new
LoCs in future in view of India’s active foreign policy and strategic intent.

EXIM-Bank of India:

 EXIM Bank of India (Exim Bank) was established in 1982 under an Act of Parliament as
the apex financial institution for financing, facilitating and promoting India’s international
trade.
 The Bank primarily lends for exports from India including supporting overseas buyers
and Indian suppliers for export of developmental and infrastructure projects, equipment,
goods and services from India. It is regulated by RBI.

Links:

http://www.pib.nic.in/Pressreleaseshare.aspx?PRID=1560101

87
http://www.edristi.in/
Integrated E-filing and Centralized Processing Centre 2.0 Project
Question: Integrated E-filing &Centralized Processing Center 2.0 Project is related to
which of the following department?
(a) CPWD (b) Irrigation (c) BSNL (d) Income Tax
Answer: (d)
Related facts:

 On 16 January 2019, Union Cabinet chaired by the Prime


Minister Shri Narendra Modi, has given its approval to
expenditure sanction of Rs.4,241.97crore for Integrated E-filing &Centralized Processing
Center 2.0 Project of the Income Tax Department.
 The Union Cabinet also approved the expenditure sanction for the consolidated cost of
Rs.1,482.44 crore of the existing CPC-ITR 1.0 project upto FY 2018-19.
 The broad objectives of this project includes Faster and accurate outcomes for
taxpayer,first time right approach,enhancing user experience at all stages,improving
taxpayer awareness and education through continuous engagement,promoting voluntary
tax compliance and managing outstanding demand.
 This approval has significant benefits for the Department and taxpayers through various
functionalities such as pre-filling of ITR and acceptance by taxpayer as a means to
improve accuracy.
 It will reduce refund/processing turnaround time drastically as well as facilitate taxpayers
in resolving outstanding tax demands through integrated contact centers.
 The decision will ensure horizontal equity by processing returns filed by all categories of
taxpayers across the country in a consistent, uniform, rule driven, identity blind manner.
This will assure fairness in tax treatment to every taxpayer irrespective of their status.

Links:

http://www.pib.nic.in/Pressreleaseshare.aspx?PRID=1560107

CRISIL report projects Bihar GSDP growth rate highest in FY 18


Question: Consider the following statements regarding ‘States of Growth 2.0’ report by
CRISIL:
1) Report ranks Bihar as the top state in terms of GSDP growth in FY 18
2) In fiscal year 2017 Bihar was at the 8th spot
3) Gujarat occupied third position in the report
4) Expansion in the growth rate came from the sectors with low employment elasticity of
the given correct alternative/s is/are:
(a) Only 1 and 2 (b) Only 2 and 3 (c) Only 3 and 4 (d) All of the above
Answer: (d)
Related facts:

 On 21st Jan 2019, CRISIL report titled States of growth


2.0 ranked Bihar as the top state in terms of GSDP
growth in financial year 2017-18.
 Bihar grabbed the top spot by registering 11.3 % GSDP
growth rate in fiscal year2018. Bihar registered 9.9 %
GSDP growth in the previous year.
 The second spot has been secured by Andhra Pradesh whose GSDP growth in FY 18
was 11.2%. Gujarat listed third with growth of 11.1% percent.
 According to CRISIL report Jharkhand, Kerala and Punjab are placed at the bottom in
terms of growth.

88
http://www.edristi.in/
 Twelve out of seventeen states registered positive growth in comparison with the national
growth rate which is of 6.7 percent for the same FY.
 The report has noted that this growth has not translated into job creation as the expansion
came from the sectors with low employment elasticity.
 According to the report the FRBM Act had helped states recover their fiscal health. Debt
ratios have risen in many states with the assimilation of schemes like UDAY, farm loan
waivers, and Pay Commission hikes.

Links:
https://www.financialexpress.com/economy/bihar-posts-highest-growth-in-fy18-says-crisil-
check-bottom-three/1451301/

India Japan Currency Swap Agreement


Question: Consider the following statements and choose the correct option in reference
to the Currency Swap Agreement between India and Japan.
1) The Union Cabinet under PM Narendra Modi approved the Currency Swap Agreement.
2) The agreement will be signed among Finance Ministry of both the nations.
3) The maximum amount decided for the agreement is USD 75 billion.
4) It also granted setup of an advance single window to boost bilateral investments.
Correct option:
(a) 1, 3 and 4 (b) 1, 2 and 3 (c) 1 and 4 (d) 2 and 4
Answer: (a)
Related facts:

 The Union Cabinet under Prime Minister Narendra Modi


approved the proposal of Agreement for Bilateral Swap
Arrangement between India and Japan. This authorized The Reserve Bank of India and
Bank of Japan to enter into agreement.

Agreement highlights:

 This arrangement will facilitate exchange and re-exchange of a maximum amount of $ 75


billion for domestic currency for the purpose of maintaining an appropriate level of
balance of payments for meeting short-term deficiency in foreign exchange.
 The Agreement will ease India in paying for the imports and address the issue of
depreciation and stabilizing the Balance of Payments. Japan is one of the biggest Asian
countries India trade with.
 Currency Swap Agreement is a foreign exchange agreement for a given amount of
currency for another between two parties for a specific period of time which is to be
returned back afterwards. This promotes the economic cooperation and helps in mutual
trade between nations using local currencies.

Advanced Single Model Window:

 The cabinet also approved a MoU (Memorandum of Understanding) between both nations
for developing an Advanced Model Single Window and its operationalization in Central
and State governments in India for taking necessary administrative procedures for
business operations.
 The Advanced Model Single Window will have best practice in and outside India. This will
promote efficiency in the business clearance in India and would promote the ease of
doing business operations.

89
http://www.edristi.in/
Links:
http://www.pmindia.gov.in/en/news_updates/cabinet-approves-mou-on-maritime-issues-
between-india-and-denmark/?comment=disable

MGNREGA Budget reaches highest level


Question: What is the amount of budget in total allocated to the MGNREGA scheme by
Central government in financial year 2018-19?
(a) Rs. 61,084 crores (b) Rs. 55,084 crores (c) Rs. 60,084 crores (d) None of the above
Answer: (a)
Related facts:

 Central government’s recent allocation of additional 6,084


crores to the prestigious MGNREGA (Mahatma Gandhi
National Rural Employment Guarantee Act.) has brought the
total allocation of this fiscal to all time high at Rs. 61,084 crores.
 The budget for MGNREGA witnessed plummet in its budget allocation having nearly
33,000 crores in 2014-15 to over 60,000 crore in 2018-19.
 The Ministry of Rural Development has made major reforms in the scheme to transform it
into a resource for sustainable livelihood for poor.
 There have been implementation of geo-tagging of resources, AADHAAR linking of Bank
Accounts, DBT transfers for all wages, and material payments and Geographic
Information System (GIS) based planning of works to ensure transparency.

About MGNREGA:

 The Mahatma Gandhi National Rural Employment Guarantee Act, 2005 (MGNREGA) is
the largest social security scheme in the world. It aims to enhance security in rural areas
by guaranteeing at least 100 days of work to every household.

Links:
http://www.newsonair.com/Main-News-Details.aspx?id=358091

Retail Inflation at 18 months lowest


Question: What percent was the retail inflation rate of India as per Consumer Price Index
in December 2018 featured as 18 months lowest?
(a) 2.33% (b) 2.18% (c) 3.64% (d) 2.19%
Answer: (d)
Related facts:

 As per data released by the Ministry of Statistics and Programme


Implementation, the retail inflation in December 2018 based on
the Consumer Price Index (CPI) continued the downward trend and going to 2.19%, an 18
months low. It was 2.33% in November 2018 and 5.21% in December 2017.
 The retail inflation has been lower than the RBI’s medium target of 4% for the fifth straight
month. This happened due to recent decline in crude prices and appreciation of rupee
against dollar.
 The wholesale inflation also slipped down to an eight months low of 3.80% in December
2018.
 However, Industrial growth has slowed down to 0.5% in November 2018 as per sources.
CPI food price index fell 2.51% in December over last year.

90
http://www.edristi.in/
Links:
https://economictimes.indiatimes.com/news/economy/indicators/december-retail-inflation-at-18-
month-low-of-2-19/articleshow/67527302.cms

India Manufacturing Barometer 2019


Question: Consider the following statements regarding India Manufacturing Barometer
2019:
1) This is a report based on surveys conducted jointly by FICCI and PwC
2) The report mentions that 74% of manufacturers hope for a faster growth rate over next
year in their respective sectors
3) Growth rate above 7% is achievable for the Indian economy
4) The companies surveyed in this report contribute 30 % to the manufacturing GDP
Correct option:
(a) 1, 2 and 3 (b) 2, 3 and 4 (c) 1, 3 and 4 (d) All of the above.
Answer: (a)
Related facts:

 Industrial body Federation of Indian Chambers of Commerce and Industries


(FICCI) and global consulting firm PwC released India Manufacturing
Barometer 2019 Report. The report contains details about various variables of Indian
Economy.
 The report was prepared on the basis of surveys conducted over companies that
contribute to 12% approx. of Indian manufacturing Gross Domestic Product (GDP) hailing
from various sectors like textiles, automobile, electrical machinery, chemicals etc.

Report highlights and findings:

 The report mentions that 74% of manufacturers hope for a faster growth rate over next
year in their respective sectors.21 percent business leaders anticipates that growth rate
would be same and 5 percent foresee growth rate to be slower in the coming 12 months.
 More than half (around 58%) also expect of their sector to grow by 5% at least over next
year.
 The report mentions that the growth rate above 7% for Indian economy is achievable in
current scenario looking 7.4% and 7.8% projected by IMF and World Bank respectively for
2019.
 The report also held a survey on GST (Goods & Services Tax) implemented during this
fiscal in which a majority of 51% of the respondents had a positive outlook over it and said
this will help in easing logistic and related processes.
 66% believed tax legislation as the right move towards boosting investment and economic
development.
 85% of business leaders anticipate an increase in turnover from global demand in the
future.
 A total of 58% firms also mentioned that non availability of raw materials locally is the key
reason of dependence on imports.

Links:
https://www.fortuneindia.com/macro/manufacturers-optimistic-of-faster-growth-rate-in-2019-
pwc-ficci-report/102850

91
http://www.edristi.in/
RBI shortlists 6 IT Firms for Public Credit Registry (PCR)
Question: The non-performing assets in the Indian banking system
stand at INR-
(a) 10 lakh crore (b) 18 lakh crore (c) 25 lakh crore (d) 30 lakh crore
Answer: (a)
Related facts:

 RBI has shortlisted six major IT companies to set up Public Credit Registry (PCR) in order
to capture details of all borrowers and wilful defaulters.
 These firms are – TCS, Wipro, IBM India, Capgemini Technology Services, Dun &
Bradstreet Information Services India and Mindtree Ltd.
 PCR is a wide-based digital Registry system for capturing details of all borrowers and
willful defaulters.
 The proposed PCR will also include data from entities like market regulator SEBI. The
Corporate Affairs Ministry, Goods and Service Tax Network (GSTN) and the Insolvency
and Bankruptcy Board of India (IBBI) to enable banks and financial institutions to get a
360-degree profile of the existing as well as prospective borrowers on a real-time basis.
 Setting up of the PCR assumes significance amid rising bad loans in the financial system.
The non-performing assets in the Indian banking system stand at about Rs. 10 lakh crore.

Links:
https://www.thehindubusinessline.com/money-and-banking/rbi-shortlists-tcs-wipro-ibm-to-set-
up-public-credit-registry/article25813810.ece

Outstanding External Commercial Borrowings (ECB)


Question: What was the outstanding stock of ECB as on September 30, 2018?
(a) 126.29 Billion Dollar (b) 110.50 Billion Dollar (c) 150.00 Billion Dollar
(d) 155.65 Billion Dollar
Answer: (a)
Related facts:

 RBI announced a cap on the outstanding Stock of ECB at 6.5 percent of GDP at current
market prices.
 Based on the GDP figures at March-end 2018, the current financial year, the soft limit
works out to $160 billion for the current financial year.
 The outstanding stock of ECB as on 30th September, 2018, stood at USD 126.29 billion.
 External Commercial Borrowings (ECB):
 ECBs refer to commercial loans in the form of bank loans,
securitized instruments (floating rate notes and fixed rate
bonds, non-convertible, optionally convertible or partially or
partially convertible preference shares), buyer’s credit;
supplier’s credit availed of form non-resident lenders with a
minimum average maturity of 3 years.

Links:
https://www.firstpost.com/business/rbi-limits-total-outstanding-of-external-commercial-
borrowings-to-6-5-of-gdp-5766501.html#aid_5766501

India’s Foreign Exchange Reserves


Question: Recently, India’s Forex reserves decline by 613.9 million dollar to –
(a) 393.12 Billion Dollar (b) 350.00 Billion Dollar (c) 425.00 Billion Dollar
92
http://www.edristi.in/
(d) 450.00 Billion Dollar
Answer: (a)
Related facts:

 The country’s foreign exchange reserves declined by 613.9 million dollar to 393.12 billion
dollar in the week to 14th December, 2018.
 In the previous week, the reserves had increased by 16.6 million dollar to 393.734 billion
dollar.
 In the reporting week, foreign currency assets, a major component of the overall reserves,
dropped by 631.6 million dollar to 367.865 billion dollar.
 Expressed in US Dollars, foreign currency assets include the effect of appreciation or
depreciation on Non-US currencies such as the euro, pound and yen held in the reserves.
 The reserves had touched a record high of 426.028 billion dollar in the week to April 13,
2018.
 Gold reserves rose by 37.2 million dollar to 21.187 billion dollar in the reporting week.
 The special drawing rights (SDR) with the IMF dipped by 7 million dollar to 1.45 billion
dollar. The country’s reserve position with the IMF also decline by USD 12.5 million to
USD 2.617 billion

Links:
https://timesofindia.indiatimes.com/business/india-business/forex-reserves-decline-by-usd-613-
9-mn-to-usd-393-12-bn/articleshow/67197236.cms

China announces duty cut on 700 foreign products


Question: Recently which of the following country has announced duty cut on 700
foreign products?
(a) Russia (b) China (c) USA (d) Japan
Answer (c)
Related facts:

 China on 24th December, 2018 announced that it would decrease temporary tariffs on
over 700 foreign products from early 2019.
 Chinese government would further impose zero tariffs on ingredients for livestock feeds
and certain medicines that also include manganese slag.
 The tariff reductions will also be enforced on aircraft engines, robots and other advanced
equipments. Nearly 300 information technology products will be taxes with most-favoured-
nation rates.
 The nation will also scrap export tariffs on 94 items of products starting from the new
year, including fertilizers, iron ore, coal tar and wood pulp, with export tariffs currently as high
as 40 per cent.
 These new tariffs will affect the products from the US, the country with which China is
involved in a trade war.

Links:
https://www.business-standard.com/article/news-ians/china-announces-tariff-cut-on-700-foreign-
products-118122400351_1.html

New President of FICCI


Question: Who has been elected as the new President of FICCI?
(a) Sandip Somany (b) Habeel Khorakiwala (c) Sangita Reddy (d) None of the
Above

93
http://www.edristi.in/
Answer: (a)
Related facts:

 Sandip Somany has been elected as the new President of FICCI, on 17th December
2018.
 He is the CMD of Sanitary ware Firm HSIL’S.
 Sangita Reddy, joint Managing Director of the Apollo Hospitals Group, has now been
elevated as the Senior Vice President of FICCI, whereas Uday Shankar, Chairman &
CEO of Star India, has joined the Chamber as the Vice President.
 Established in 1927, FICCI is the largest and oldest apex business organization in India, it
is a non-government, not-for-profit organization.
 FICCI is the voice of India’s business and industry FICCI provides a platform for
networking and consensus building within and across sectors.
 It is the first port of call for Indian industry, policy makers and the international business
community.

Links:
http://ficci.in/about-us.asp

Scientific

Defense/Science Short Notes


Long Range Surface-to-Air Missile (LRSAM) successfully tested
Question: Long Range Surface-to-Air Missile (LRSAM) which has been successfully test
fired from INS Chennai is co-developed by which countries?
(a) India and France (b) India and Russia (c) India and USA (d) India and Israel
Answer (d)
Related facts:

 The Indian Navy has successfully test-fired the long-range surface-to-air missile (LRSAM)
from warship INS Chennai, off the coast of Odisha.
 LRSAM has been jointly developed by Defence Research and Development Organisation
(DRDO), India and M/s Israel Aerospace Industries (IAI), Israel for the Indian Navy.
 Also known as Barak 8, the missile can counter-attack aircraft, unmanned aerial vehicles
(UAVs) and incoming anti-ship missiles.
 LRSAM is capable of achieving a maximum speed of Mach 2 and has an operational
range of 70km.

Links:
https://www.naval-technology.com/news/indian-navy-lrsam/

Yutu-2, descended on the lunar surface


Question: China named lunar rover as?
(a) Yutu-1a (b) Yutu-2b (c) Yutu-2 (d) Yutu-2a
Answer: (c)
Related facts:

94
http://www.edristi.in/
 On 2nd Jan, 2019, Yutu-2, or literally Jade Rabbit-2, separated from the lander and
descended on the lunar surface. China’s second lunar rover is expected to bring more
scientific discoveries from the alien world.
 The China National Space Administration (CNSA) and The Chinese Lunar Exploration
Program (CLEP) made the announcement after China’s Chang’e-4 probe, comprised of a
lander and a rover, landed on the far side (Von Karman crater) of the moon earlier.
 The 135-kg new rover is 2 kg lighter than its predecessor and is the lightest rover ever
sent to the moon.
 China successfully deployed Chang’e-4 lunar probe to carry out a string of experiments
on the unexplored far side of the moon.
 It will be focusing on advanced technology, including space applications.

Links:
http://www.china.org.cn/china/Off_the_Wire/2019-01/04/content_74341381.htm

Arrow 3 Anti Ballistic Missile


Question: Arrow 3- Anti Ballistic Missile was successfully tested by which country?
(a) Israel (b) France (c) Russia (d) India
Answer: (a)
Related facts:

 Israel successfully tested Arrow 3- Anti Ballistic Missile on 22nd Jan,


2019 by firing an intercepting missile at a target.
 The test by Israel Aerospace Industries and the Israeli Air Force, working
with the U.S Department of Defense Missile Defense Agency, conducted
the test at an unidentified location in central Israel.
 As soon a target missile was launched, the radar (Iron Dome system) identified the target
and passed the data on to the control center and after analyzing the data a flight plan was
programmed, after that Arrow 3 was launched toward the target and completed the
mission successfully.
 The successful test of Arrow 3 will enhance the Israel’s capability to protect itself from
continuing regional threats.

Links:
http://missiledefenseadvocacy.org/missile-defense-systems-2/allied-air-and-missile-
defense-systems/allied-intercept-systems-coming-soon/arrow-israel/

Government allocates Rs 10,900 crore rupees for ISRO


Question: Recently,Union Government has allocated how much of fund for the
development of 40 satellite launch vehicle in next 4-years?
(a) 9,000 cr. (b) 10,900cr. (c) 12,000 cr. (d) 8,000 cr.
Answer: (b)
Related facts:

 On 7 January 2019,ISRO’s Chairman Dr K Sivan notified that the


Union Government has allocated 10,900 crore rupees for the
development of 40 satellite launch vehicles in the next four
years.
 This information was made by ISRO’s head at Graduation Day lecture at the historic St
Joseph’s College, Trichy, TamilNadu, which is celebrating its 175th anniversary.
 He notified that first Indian human space flight programme ‘Gaganyaan’ is being planned
to be launched by 2022,(75th anniversary of country’s independence).
95
http://www.edristi.in/
 Mr Sivan also stated that ISRO will be organizing the Centenary year celebrations of
ISRO founder Dr. Vikram Sarabhai next year.

Links:
http://newsonair.nic.in/Main-News-Details.aspx?id=357646

NASA’s probe discovers a new planet


Question: NASA’s latest planet-hunting probe Transiting Exoplanet Survey Satellite
(TESS) has recently discovered a new planet named as HD 21749b located in
constellation-
(a) Orion (b) Libra (c) Reticulum (d) Sagittarius
Answer:(c)
Related facts:-

 NASA’s latest planet-hunting probe Transiting Exoplanet Survey Satellite (TESS) has
discovered a new planet orbiting around a dwarf star 53 light-years away from the earth.
 The new planet has been named as HD 21749b.
 This is the third new planet confirmed by the Transiting Exoplanet Survey Satellite (TESS)
since its launch in April last year.
 This newly discovered planet is located in constellation Reticulum.
 The planet is about three times the size of earth, which puts it in the category of a “Sub-
Naptune. HD 21749b revolves around its star in 36 days.

Links:

https://www.thehindu.com/sci-tech/science/nasas-probe-discovers-a-new-
planet/article25943858.ece

Government Launches two national level Initiatives in the field of


science communication
Question: To popularise science in India, recently Government of India has launched
how many channels?
(a) 1 (b) 2 (c) 3 (d) 5
Answer: (b)
Related facts:

 On 15 January 2019, Department of Science and Technology (DST) alongwith


Doordarshan,Prasar Bharati launched two science communication initiatives,DD Science
and India Science.It was inaugurated by Union Science & Technology Minister, Dr. Harsh
VardhanThe free-to-air channels have been conceived and supported by the DST and are
being implemented and managed by Vigyan Prasar, an autonomous organisation of DST.
 DD Science is a one-hour slot on Doordarshan National channel and will be telecast
Monday to Saturday from 5 pm to 6 pm
 India Science, an internet-based channel, will be available on any internet-enabled device
and will offer live, scheduled play and video-on-demand services.
 The channels will have science-based documentaries, studio-based discussions, virtual
walkthroughs of scientific institutions, interviews and short films and will be completely
free to access.

Links:
http://pib.nic.in/PressReleaseIframePage.aspx?PRID=1559990

96
http://www.edristi.in/
Telescope X-Calibur
Question: Consider the following statements regarding Telescope X-Calibur:

1. This satellite was launched from Antarctica

2. It is developed to look/examine the activities and movement of Mars

3. It is developed by NASA in US

4. This telescope will go above 99% of Earth’s atmosphere

Correct option:
(a) 1,2 and 4 (b) 1, 2 and 3 (c) 2, 3 and 4 (d) All of the above.
Answer: (c)
Related facts:

 The international team of scientists and engineers of Washington University, US and


several other universities developed the X-Calibur telescope. This was launched on
January 5th, 2019, from McMurdo station in Antarctica.
 The telescope was lifted on a helium balloon intended to reach an altitude of 1, 30,000 ft.
and above, (99% of Earth`s atmosphere). In around eight days it will make a single
revolution of Antarctic continent to gather enough data to make the mission a success.
 X-Calibur will measure the polarisation (the plane in which the electric field of the X-rays
oscillates) to high energy x-ray emission. This will help in gaining insight into how black
holes in a binary orbit with stars grow by gobbling up stellar matters.
 The team will also observe X-rays from neutron stars, the 12 km radius objects with
masses exceeding the mass of our sun. Neutron stars are objects with very small radius
(typically 30km) and very high density, composed predominantly of closely packed
neutrons.
 Researchers also hope to use the Vela X-1 observation revealing acceleration of particles
to high energies by neutron stars. Vela X-1 is a neutron star in binary orbit with a
supergiant star.
 This project was financially supported by NASA and McDonnell Centre for Space
Science.

Links:
https://sites.wustl.edu/xcalibur/

China landed robotic spacecraft to the Moon


Question: Consider the following statements in reference to the spacecraft launched by
China:
1. China successfully landed its spacecraft Chang`e-4 to the moon on January 3rd
2019
2. The spacecraft landed on the Far (darker) side of the Moon
3. China, with this joined the group of US and Russia to successfully land the
spacecraft on the darker side of the moon
4. This spacecraft will conduct various biological experiments and will analyze
unexplored region of moon
Correct option is:
(a) 1, 2 and 3 (b) 1, 3 and 4 (c) 1, 2 and 4 (d) All of the above

97
http://www.edristi.in/
Answer: (c)
Related facts:

 China created history on January 3rd, 2019, with the successful landing of spacecraft
named Chang`e-4 on the darker side (also called Far side) of Moon. This made China the
very first nation to make the first spacecraft soft-landing on the moon’s darker side.
 This uncrewed spacecraft was launched from Xichang Satellite Launch Centre and
landed in the Von Karmen Crater in the South Pole-Aitken Basin of the Far side of the
Moon.
 The spacecraft has sent back its first picture from the surface of Moon. With this, China
joined the group of United States of America and Russia as the nations to make
successful “soft landing” on the Moon.

About Chang`e-4:

 Chang`e-4 is named after the Chinese moon goddess. It was launched on December 7,
2018 and arrived in lunar orbit on 12 December, 2018.
 Chang’e-4’s static lander is carrying two cameras; a German-built radiation experiment
called LND; and a spectrometer that will perform low-frequency radio astronomy
observations.
 The lander also carried a container with six live species from Earth – cotton, rapeseed,
potato, fruit fly, yeast and arabidopsis (a flowering plant) – to try to form a mini biosphere.
Other equipment/experiments include a panoramic camera, radar, an imaging
spectrometer, lunar neutron, low frequency radio spectrum analyser and more.
 It will lead the study of yet not visited Far side of the Moon as well as the mystery of
evolution of Moon. It will study the sheet of melted rock that would have filled the newly
formed South Pole Aitken Basin as well as the broken up rocks and that make up surface.
 It is also important to know that being landed of the Far side of the moon, cannot
communicate directly to stations for which a communication relay satellite Queqiao was
launched in last May to serve the very purpose of communication establishment between
Chang`e-4 and the Space station.

Far side of the Moon:

 The Far or Dark side of the Moon is that hemisphere that always faces away from earth
and is invisible.
 This is because of ‘tidal locking’ phenomenon by which we see only one part of the
Moon as it takes the same time to rotate on its axis by moon as it takes to complete orbit
of Earth. Dark side has thicker, older crust pocked with more craters while near side
have a thinner shell.

Links:

http://www.xinhuanet.com/english/2019-01/03/c_137716998_2.htm

Asteroid Bennu
Question: Recently NASA’s OSIRIS-REx spacecraft has discovered traces of water
forming ingredients i.e. hydrogen and oxygen molecules in which of the following
asteroid?
(a) Pallas (b) Bennu (c) Hygiea (d) Ceres
Answer: (b)
Related facts:

98
http://www.edristi.in/
 NASA’s OSIRIS-REx spacecraft has discovered traces of water forming hydrogen and
oxygen molecules in the asteroid Bennu’s rocky surface.
 OSIRIS-REx spacecraft arrived at asteroid Bennu on December 3, 2018.
 OSIRIS-REx is the first asteroid sample-return mission sent by the American space
agency NASA, to collect samples of the surface regolith and bring them back to Earth for
further study.
 OSIRIS-REx fully known as Origins,Spectral Interpretation,Resource
Identification,Security-Regolith Explorer.
 Under the NASA’s New Frontiers program starting from the New Horizons spacecraft
which passed through Pluto and Juno spacecraft into orbit around Jupiter, OSIRIS-REx is
the third mission in this series.

Links:
https://www.indiatoday.in/education-today/gk-current-affairs/story/water-on-asteroid-bennu-
nasa-spacecraft-1407227-2018-12-11

1st Synthetic material that gets thicker on stretching found


Question: Which is the first Synthetic material that gets thicker on stretching?
(a) Liquid Crystal Elastomer (b) Benzaldehyde (c) Formalin (d) Thermoplastics
Answer: (a)
Related facts:

 According to Dr. Devesh Mistry from Leeds’ School of Physics and Astronomy told Digital
Trends about a new synthetic material Liquid Crystal Elastomer which actually gets
thicker the more it’s stretched.
 Governing branch of science which deals with its study is known as Auxetics. Auxetic
materials get thicker and they are also great at energy
absorption and resisting fracture.
 Some examples of other Auxetic materials include cat skin,
tendons in the human body, and the protective layer in
muscle shells.
 In some cases experiments on these materials prove them to
be weak & porous.
 There may be many potential applications for materials with
these properties including body armour, architecture and medical equipment.

Links:
https://www.dailymail.co.uk/sciencetech/article-6458373/Scientists-create-mind-boggling-
material-gets-THICKER-stretched.html

Bee Vaccine
Question: Which of the following countries’ scientists have developed a vaccine for
bees?
(a) Finland (b) USA (c) Canada (d) Germany
Answer: (a)
Related facts:

 Scientist from Finland have developed what they believe is the


world’s first vaccine to protect bees against disease.
 Bees are vital for growing the world’s food as they help to
fertilize three out of four crops around the globe, by transferring pollen form male to
female flowers.
99
http://www.edristi.in/
 In recent years bee populations around the world have been dying off form “colony
collapse disorder”, a mysterious scourge blamed on mites, pesticides, virus, fungus, or
some combination of these factors.
 UN-led research in 2016 found that more than 40 percent of invertebrate pollinators,
particularly bees and butterflies, are facing extinction.
 The study also found that 16.5% of vertebrate pollinators, such as birds and bats, are
under threat.
 The vaccine, developed by a team at Helsinki University in Finland, works by giving bees
resistance to fight off severe microbial diseases that can be total for pollinator
communities.

Links:
https://www.firstpost.com/tech/science/scientists-engineer-bee-vaccine-to-help-the-pollinator-
fight-off-insect-apocalypse-5742281.html

Sports

Cricket
First Woman Cricketer to play 200 ODIs
Question: Who became the first Woman cricketer to play 200 International Matches in
ODI?
(a) Belinda Clarks (b) Charlotte Edwards (c) Sarah Taylor (d) Mithali Raj
Answer: (d)
Related facts:

 Indian women’s One Day International (ODI) Captain Mithali


Raj became the first female player in the world to play 200
international ODIs.
 She made this record while playing the 3rd ODI against
New Zealand in Hamilton on February 1, 2019. New Zealand won the match by 8 wickets.
 India lost the final match but won the series by 2-1.
 Mithali made debut in 1999 and she was a team member in 200 matches out of 263 ODIs
played by the womens team till then. • She is the highest run scorer in
womens ODI format. She has scored 6622 runs with an average of 51.33. This includes
seven hundreds and 52 fifties.
 She has also played 10 tests and 85 T20s till now.

Links:
https://www.sportskeeda.com/cricket/mithali-raj-becomes-first-woman-cricketer-to-play-
200-one-day-internationals

Jasprit Bumrah becomes top wicket-taker in International cricket in 2018


Question: Currently which player becomes top wicket-taker in International cricket in
2018?
(a) Jasprit Bumrah (b) Ravichandran Ashwin (c) Mohammad Shami (d) Umesh Yadav
Answer: (a)
Related facts:

100
http://www.edristi.in/
 In 2018, India’s speedster Jasprit Bumrah finished as the highest wicket-taker across all
formats by taking 78 wickets.
 Jasprit took 48 wickets in 9 Tests, 22 wickets in 13 ODIs and 8 wickets in 8 T20I matches.
 He went past Kagiso Rabada, who took 77 wickets this year.
 The South African pacer took 52 wickets in 10 Tests, 23 wickets in 14 ODIs and 2 wickets
in 2 T20Is.

Links:
https://www.indiatvnews.com/sports/cricket-jasprit-bumrah-ends-2018-on-high-note-
becomes-highest-international-wicket-taker-in-international-cricket-496251

ICC Awards 2018


Question: Who is awarded Men’s Cricketer of the Year 2018 in ICC Awards?
(a) Virat Kohli (b) Ajinkya Rahane (c) Rohit Sharma (d) Hardik Pandya
Answer : (a)
Related facts:

 On 22 January 2019, ICC announced ICC Awards for the year 2018.Unlike previous year;
this year too the Men’s Cricketer of the Year is awarded to Virat Kohli.
 Various awards are been presented every year by ICC in the annual awards. Some major
awards include Sir Garfield Sobers Trophy (Cricketer of the
Year), Test Player of the Year, ODI Player of the Year,
Twenty20 International Performance of the Year, Emerging
Player of the Year Notable ICC Awardees 2018 for year
2018
 Cricketer of the Year 2018: Virat Kohli (India)
 Test Player of the Year 2018: Virat Kohli (India)
 ODI Player of the Year 2018: Virat Kohli (India)
 wenty-20 International Performance of the Year 2018: Aaron Finch (Australia)
 Emerging Player of the Year 2018: Rishabh Pant (India)
 International Cricket Council (ICC) is the world-wide governing body of cricket. It was
founded as the Imperial Cricket Conference in 1909 by representatives from Australia,
England and South Africa. It was renamed as the International Cricket Conference in
1965, and took up its current name in 1989.

Links:
https://en.wikipedia.org/wiki/ICC_Awards#2018_awards

India wins ODI Series


Question: India won the bilateral ODI Series in Australian soil.Who was awarded with
Man of the Series in the tournament?
(a) Virat Kohli (b) Rohit Sharma (c) Jaspreet Bumrah (d) MS Dhoni
Answer: (d)
Related facts:

 With a match winning 87 runs knock by Mahendra Singh Dhoni in the third and final
match led India to a historic ODI series win by 2-1. Dhoni was declared Man of the
Series.
 He scored 3 consecutive half centuries in the 3 matches scoring a total of 193 runs. He
became the oldest Indian cricketer to win this award. This is his 7th Man of the Series title
in ODI cricket career.

101
http://www.edristi.in/
 India won its first bilateral ODI Series in Australian soil. India also remained
undefeated in Australian soil in all three formats.

Links:

http://www.newsonair.com/Main-News-Details.aspx?id=358228

India wins Border-Gavaskar Trophy


Question: India concluded Test series win over Australia. What is the name of trophy
awarded to India?
(a) Tendulkar-Ponting Trophy (b) Bordar-Gavaskar Trophy (c)
Don Bradman Trophy
(d) Warne-Kumble Trophy
Answer: (b)
Related facts:

 India won the prestigious Border-Gavaskar Trophy after beating Australia in the recently
concluded Test Series by 2-1.
 This is India’s historic first series win on the Australian soil since its first tour in the year
1947-48.
 India reinforced their status as the world’s number one test team.
 With this series win, Virat Kohli became the first Indian as well as Asian captain to win a
series in Australia and retained the Border-Gavaskar Trophy.
 Cheteshwar Pujara was awarded Man of the Series for making a total of 521 runs in the
series including 3 centuries.

Links:
http://www.newsonair.com/Main-News-Details.aspx?id=357499

ICC Hall of Fame


Question: Recently which former cricketer is inducted into ICC Hall of Fame?
(a) Rahul Dravid (b) Kumara Sangakara (c) Ricky Ponting (d) VVS
Laxman
Answer: (c)
Related facts:

 Veteran Australian cricketer Ricky Ponting was inducted formally into ICC (International
Cricket Council) Cricket Hall of Fame on 26th December, 2018. Ponting received the cap
of commemoration from his co player and recipient of ICC Cricket Hall of Fame, Glenn
McGrath at the MCG.
 Ponting was named in the Hall of Fame along with former Indian cricketer Rahul Dravid
and England women wicketkeeper Claire Taylor in July 2018 during the ICC Annual
Conference in Dublin in July.
 He is 25th in the list of Australian Cricketer to be inducted into ICC Cricket Hall of Fame.
Australia won 2 World Cup titles under his captaincy. He retired in 2012 after scoring
13,378 runs in 168 Tests and 13,704 runs in 375 ODIs.
 Ponting was also awarded ICC Player of the Year in 2006, 2007 and ICC Test Player of
the Year in 2006.

Links:
http://www.newsonair.com/Main-News-Details.aspx?id=357179

102
http://www.edristi.in/
Football
AFC Asian Cup 2019
Question: AFC Asian Cup 2019 is hosted by which country?
(a) United Arab Emirates (UAE) (b) Qatar (c) Mexico (d) Saudi Arabia
Answer: (a)
Related facts:

 AFC (Asian Football Confederation) Cup 2019 was hosted by United Arab Emirates
(UAE).
 This was the 17th edition of the AFC Asian Cup.
 The 28 day tournament took place from January 5 to February 1,
2019.
 Qatar won the tournament defeating four time winner Japan by 3-1.
 Final match of AFC Asian Cup 2019 football tournament held at Abu
Dhabi’s Zayed Sports City Stadium in UAE.
 A total of 24 teams participated in this tournament.
 The Asian Football Confederation (AFC) founded in 1954 is the governing body of Asian
football and one of the six Confederations making up FIFA. It is headquartered in Kuala
Lumpur, Malaysia and consists of 46 Member Associations

Links:
http://www.the-afc.com/asiancup/

Lionel Messi grabs record fifth Golden Shoe award


Question: Which of the following player won the Golden Shoe award for the fifth time?
(a) Cristiano Ronaldo (b) Mohamed Salah (c) Harry Kane (d) Lionel Messi
Answer- (d)
Related facts:

 On 18th December, 2018, the Golden Shoe Award of Europe was


given to LionelMessi, captain of both Barcelona and the Argentina
national team.
 Messi scored 34 goals in 36 La Liga appearances in 2017-18 to grab
this title.
 Messi overtook Mohamed Salah of Liverpool (32 goals) and Harry Kane of Tottenham
Club (30 goals).
 Four-time Golden Shoe champion Cristiano Ronaldo (Real Madrid) finished fourth with 26
goals.
 Messi has won the Golden Shoe Award for the season 2009-10,2011-12,2012-13,2016-
17 and 2017-18 (5 times total).

Links:

https://www.bbc.com/sport/football/46609965

103
http://www.edristi.in/
Boxing
Latest AIBA Rankings

Question: Name the Indian boxer, who secured first position in the latest AIBA
Rankings?
(a) Sarjubala Devi (b) Pinki Rani Jangra (c) MC Mary Kom (d) Sakshi
Malik
Answer: (c)
Related facts:

 Prominent Indian women boxer Mary Kom is ranked number one with 1700 points in the
latest International Boxing Association`s (AIBA) world rankings in 45-48kg category.
 She became the most successful boxer in world championship history when she claimed
the 48kg category top honours in Delhi in November last year becoming six time World
Women’s Boxing Champion.
 She achieved Gold Medal last year in the Commonwealth Games and at a tournament in
Poland. She also won Silver in the prestigious Strandja Memorial in Bulgaria.
 Among others from India, Pinki Rani Jangra is placed 8th in 51kg category, Manisha
Maun is placed 8th in the 54kg category and Sonia Lather is placed at 2nd spot in 57kg
category.
 International Boxing Association (AIBA) is a sport organization formed in 1946 and
headquartered in Lausanne, Switzerland. It sanctions Olympic style boxing matches and
awards world and subordinate championships.

Links:
https://d152tffy3gbaeg.cloudfront.net/2018/12/Dec18_48kg.pdf

Sports Miscellaneous
Shakil Ahmed sets world record in indoor rowing
Question: Shakil Ahmed, is associated with?
(a) Swimming (b) Indoor rowing (c) Judo (d) Chess
Answer: (b)

 Shakil Ahmed, who is Asian Cup gold medalist and noted oarsman on 13th January,
2019, has set up a world record in indoor rowing by completing one lakh meters in above-
40 years category in Kolkata.
 A FISA (International Rowing Federation) level one coach, Shakil began his record-setting
indoor rowing feat in Salt Lake stadium complex at 8 am on 13th January, 2019, and
achieved the milestone of covering 1,00,000 meters of simulated distance in 10 hours.
 He also has the record of completing 50,000 meters of simulated rowing in four hours and
13 minutes which is already in the Limca Record Book.

Links:
https://timesofindia.indiatimes.com/sports/more-sports/others/shakil-ahmed-sets-world-
record-in-indoor-rowing/articleshow/67527437.cms

104
http://www.edristi.in/
Short Notes

Personalities
Fame India Best Member of Parliament
Question:Who has been awarded the Fame India Best Member of Parliament 2019 title?
(a) Anurag Thakor (b) Dharmendra Yadav (c) Ranjeet Ranjan (d) Smriti Irani
Answer: (c)
Related facts:

 Congress MP Ranjeet Ranjan has been awarded the “Fame India Best Member of
Parliament 2019”.
 She represents Supaul Constituency of Bihar. The award was given by Union Minister
Giriraj Singh.
 The award was given in an event in Delhi.
 Last year this award was given to his husband Pappu Yadav who is the Member of
Parliament from Madhepura, Bihar from Jan Adhikar Party.

Links:
https://hindi.sakshi.com/national/2019/02/01/congress-mp-ranjeet-ranjan-conferred-fame-
india-best-parliament-award

New CBI Director


Question: Who is appointed as the new director of CBI?
(a) Rishi Kumar Shukla (b) Naresh Kumar Shukla (c) Sanjay Kumar Shukla
(d) Rajiv Mehrishi
Answer: (a)
Related facts:

 On 4 February 2019,Rishi Kumar Shukla took over as director


of Central Bureau of Investigation (CBI).
 He replaced M. Nageshwara Rao serving as an interim chief of CBI.
 Rishi Kumar Shukla is a Madhya Pradesh cadre 1983 batch IPS officer. He has also
served as DGP of Madhya Pradesh. His appointment for the post is for 2 years.
 A clash in CBI took place in last days after which central government removed director
Alok Verma and special director Sudhir Asthana and appointed M. Nageshwara Rao as
interim director.

Links:
http://www.newsonair.com/Main-News-Details.aspx?id=359190

The Hollywood Walk of Fame


Question: Who will be honored with a star on the Hollywood Walk of Fame?
(a) Pink (b) Salena Gomez (c) Shakira (d) Lady Gaga
Answer: (a)
Related facts:

105
http://www.edristi.in/
 American singer Alecia Beth Moore, professionally known as Pink, will be honored with a
star on the Hollywood Walk of Fame on February 5.
 The announcement was made the Hollywood Chamber of Commerce on Thursday.
 The singer is recognized in the recording category and will receive the 2,656th star on
Hollywood Boulevard.
 Pink is widely known for her popular songs “Get the Party Started”. “Don’t let me get me”,
and “Just like a pill” and many more.

Links:
https://indianexpress.com/article/entertainment/music/pink-singer-hollywood-walk-of-
fame-5554806/

Justice TBN Radhakrishnan takes oath as Chief Justice of the newly


formed Telangana High Court
Question: Who took oath as Chief Justice of the newly formed Telangana High Court
recently?
(a) Deepak Misra (b) Raghunandan Swarup Pathak (c) Kalyan Jyoti Sengupta
(d) TBN Radhakrishnan
Answer: (d)
Related facts:

 On 1st January 2019, Justice Thottathil Bhaskaran Nair Radhakrishnan sworn in as the
first Chief Justice of Telangana High Court.
 Justice Radhakrishnan was administered the oath of office
and secrecy by Governor of Telangana and Andhra Pradesh
ESL Narasimhan at a simple ceremony in Raj Bhawan.
 Later, Justice Radhakrishnan administered the oath of office to
12 justices becoming part of sitting judges of Telangana High
Court.
 Since July last year Justice Radhakrishnan had been serving
as the Chief Justice of the combined high court of the two Telugu states (Andhra Pradesh
& Telangana).
 On the same day,Justice Praveen Kumar sworn in as the acting chief justice of Andhra
Pradesh High Court.

Link :
http://www.newsonair.com/Main-News-Details.aspx?id=357375

Mrinal Sen passed away


Question: Name the veteran filmmaker & Phalke awardee who passed away recently?
(a) Kadar Khan (b) Mrinal Sen (c) Ritwik Ghatak (d) None of the above
Answer: (b)
Related facts:

 On 30th December 2018, Veteran filmmaker, Padma Bhushan


& Dadasaheb Phalke Awardee Mrinal Sen passed away at the
age of 95 years at his home in Bhawanipur, Kolkata.
 Born on May 14, 1923 in Faridpur, (currently in Bangladesh),
Mrinal Sen was suffering from old age related ailments for a
long time.

106
http://www.edristi.in/
 Contemporary of Satyajit Ray and Ritwik Ghatak; Mrinal Sen had done pioneering works
in parallel cinema like Bhuvan Shome, Mrigyaa, Ek Din Achanak and Padatik.
 In 1982 Bengali film “Kharij” had won the jury prize at the 1983 Cannes Film Festival.

Links:
https://www.business-standard.com/article/pti-stories/legendary-filmmaker-mrinal-sen-
cremated-119010100600_1.html

New Chief Justice of Pakistan


Question: Who has been appointed as Chief Justice of Pakistan?
(a) Mian Saqib Nisar (b) Anwar Zaheer Jamali (c) Justice Asif Saeed Khan Khosa (d)
Gulzar Ahmed
Answer: (c)
Related facts:

 President Arif Alvi appointed Justice Asif Saeed Khan Khosa as Chief
Justice of Pakistan (CJP). He succeded Justice Saqib Nisar.
 Justice Khosa is the 26th Chief Justice of Pakistan.
 He took charge on 18 January 2019. He will serve as CJP (Chief
Justice of Pakistan) till December 20, 2019
 The Chief Justice of Pakistan (CJP) is the head of the judiciary in Pakistan (the judicature
branch of government) and the chief judge of the Supreme Court of Pakistan.

Links:
https://www.thenews.com.pk/latest/414214-profile-of-justice-asif-saeed-khosa-pakistans-
new-chief-justice

President of the Global Solar Council


Question: Who took over as President of the Global Solar Council?
(a) Pranav R Mehta (b) Manjul Bhargava (c) Kal penn (d) Vivek Murthy
Answer: (a)
Related facts:

 Solar Man of India Pranav R Mehta took over as the President


of the Global Solar Council, headquartered in Washington DC
from 1 January 2019.
 He became the first Indian to head the Global Solar Council.He
is currently serving as the Chairman of National Solar Energy Federation of India.
 Global Solar Council is an international non-profit association of the national, regional and
international associations in solar energy and the world.
 The Global Solar Council (GSC) was launched on December 6, 2015, following the
historic United Nations Climate Change Conference (UN COP 21). It is an International
Coalition of more than 30 nations.
 National Solar Energy Federation of India (NSEFI) is an umbrella organization of all solar
energy stakeholders of India. This apex solar organization works in the area of policy
advocacy and is a national platform for addressing all issues connected with solar energy
growth in India.

Links:
http://nsefi.in/

107
http://www.edristi.in/
Dr. T.C.A Anant appointed as Member of UPSC
Question: Who administers oath to Member of UPSC?
(a) Chairman, UPSC (b) President of India (c) Chief Justice of India (d) None of these
Answer: (a)
Related facts:

 On 14 January 2019, Dr. T.C.A Anant was administered the


oath of office and secrecy of Member of UPSC by Chairman of
UPSC Arvind Saxena Dr. Anant is a former Chief Statistician of
India and Secretary, Ministry of Statistics and Programme
Implementation.
 A member of a Public Service Commission shall hold office for a term of six years from
the date on which he enters upon his office or until he attains, in the case of the Union
Commission, the age of sixty-five years, and in the case of a State Commission or a Joint
Commission, the age of sixty-two years, whichever is earlier
 As on 14 January 2019, the commission has strength of ten members including the
chairman.
 Other Members of UPSC are Prof. Pradeep Kumar Joshi, Sh. Bhim Sain Bassi, Air
Marshal A.S. Bhonsle (Retd.), Ms Sujata Mehta, Dr. Manoj Soni, Ms. Smita Nagraj, Ms.
M. Sathiyavathy, Sh. Bharat Bhushan Vyas.

Links:
http://www.pib.nic.in/Pressreleaseshare.aspx?PRID=1559815
https://upsc.gov.in/about-us/commission-

Chef de Mission for Tokyo Olympics


Question: Who is named as the Chef de Mission for the 2020 Tokyo Olympics?
(a) Birendra Prasad Baishya (b) Vijendra Singh (c) Narinder Dhrv
Batra (d) Rajeev Mehta
Answer: (a)
Related facts:

 Birendra Prasad Baishya, currently serving as President of


Indian Weightlifting Federation (IWF) was named Chef de Mission for 2020 Tokyo
Olympics.
 This marks history for being the first time that Weightlifting has been accorded with the
honour of Chef de Mission in the history of Olympics.
 Birendra Prasad Baishya is also one of the Indian Olympic Association Vice Presidents
besides holding several other positions like Vice President of the Asian Weightlifting
Federation and Vice President of the Commonwealth Weightlifting Federation.

Links:
http://www.newsonair.com/Main-News-Details.aspx?id=357862

Kumar Rajesh Chandra


Question: Kumar Rajesh Chandra has recently been appointed to the post of-
(a) DG ,SSB (b) Director, CBI (c) Director, DRDO (d) CEO, NITI
Aayog
Answer: (a)
Related facts:

108
http://www.edristi.in/
 Shri Kumar Rajesh Chandra, a 1985-batch IPS officer (Bihar Cadre) has been appointed
as the Director General (DG) of the Sashastra Seema Bal(SSB).
 He will succeed S.S. Deswal at this post.
 His tenure as DG, SSB will last from the date of joining of the post and upto 31.12.2021
i.e. date of his retirement on superannuation or until further orders, whichever is earlier.
 He is currently serving as DG, BCAS (Bureau of Civil Aviation Security) under the Union
Civil Aviation Ministry.

Links:
https://www.business-standard.com/article/news-ians/kumar-rajesh-chandra-appointed-ssb-dg-
119010800368_1.html

Alok Verma
Question: Director of which of the following agency of Union Government has recently
been reinstated by the Supreme Court?
(a) DRDO (b) CBI (c) TRAI (d) BIS
Answer: (b)
Related facts:

 On January 8, 2019; the Supreme Court reinstated Alok Verma as Director, Central
Bureau of Investigation.
 The decision of reinstatement of Mr Verma came after the quashing of unprecedented
order by the Union Government in which he was stripped of his power and sent on leave
along with his deputy after they traded corruption charges.
 The Apex court, however, subjugated the power of Mr Verma, saying he cannot take any
major policy decision till the high-powered committee, which selects the CBI chief,
decides on his continuation.
 Curtailment of powers of Mr. Verma has been done by the SC since the Central Vigilance
Commission is probing charges of corruption against him. Verma’s two-year tenure as
CBI director ends on January 31 this year.

Links:
https://www.indiatoday.in/india/story/alok-verma-supreme-court-judgment-cbi-1425928-2019-01-
08

World Bank President Steps Down


Question: Consider the following statements with reference to World Bank:
1) World Bank Group President Jim Yong Kim announced his resignation from the post
on January 7
2) He is the 14th President of World Bank
3) He was nominated in 2012 by US
4) World Bank CEO will assume the role of interim President
Correct option:
(a) 1, 3 and 4 (b) 1, 2 and 3 (c) 2, 3 and 4 (d) All of the above.
Answer: (a)
Related facts:

 World Bank President Jim Yong Kim on 7th January, 2019, announced his resignation
from the post. This announcement will take effect from 1st February, 2019, much before
his term which would have ended in 2022. World Bank CEO Kristalina Georgieva (World
Bank CEO) will be interim President from February 1.

109
http://www.edristi.in/
 Jim Yong Kim, a Korean-American anthropologist was elected as the 12th President of
World Bank for the first time in July 2012 and later got re-elected in September 2016 for
the second five year term. Jim Yong Kim succeeded Robert Zoellick. He was nominated
by the then US President Barak Obama which is the largest stakeholder of this
multilateral financial institution.
 Jim leaded the organization for setting two goals in 2012: to end extreme poverty by 2030
and to boost shared prosperity, focusing on the bottom 40% of the population in
developing countries. These goals works as guidelines in the works of institution.
 World Bank under his leadership is now supported strongly by stakeholders for its better
positioned to respond to the development needs as the Bank Group Fund for the Poorest
achieved two successive record replenishments. The Bank Group`s Governors also
approved $13 billion capital increase in IBRD and IFC allowing the bank to support
countries in reaching their development goals while responding to crisis.

World Bank:

 World Bank is the world`s largest development institution helping countries in their
developmental needs by offering loans and advice. It is headquartered in Washington,
US.
 It was founded in July 1944 as International Bank for Reconstruction and Development
soon called the World Bank. It was originally created to aid countries devastated in World
War II.

Links:
http://www.worldbank.org/en/news/press-release/2019/01/07/world-bank-group-president-kim-
to-step-down-february-1

IMF Chief Economist


Question: Who joined recently as the Chief Economist of International Monetary Fund?
(a) Maurice Obstfeld (b) Christine Legard (c) Raghuram Rajan (d)
Gita Gopinath
Answer: (d)
Related facts:

 On 8 January 2019, India born US citizen Gita Gopinath joined


as the 11th Chief Economist of International Monetary Fund
(IMF).
 Her appointment as the chief economist of IMF was made on 1 October 2018 by Christine
Legard, MD, IMF. She became the first women to be appointed at this post. She will
succeed Maurice (Maury) Obstfeld as Economic Counsellor and Director of IMF`s
Research Department who retired on 31 December 2018.
 Ms. Gopinath earlier served as the John Zwaanstra Professor of International Studies and
Economics at Harvard University. She is also co-editor of American Economic Review.
She is regarded as on the best economist with specialization on international trade &
finance.
 IMF (International Monetary Fund) also known as the Fund was conceived at Bretton
Woods Convention in 1944. This organization was created subsequently in 1945 and has
189 member countries working to foster global monetary cooperation, securing financial
stability, facilitating international trade, promoting high economic & sustainable growth
and reducing poverty around the world.Its headquarters is in Washington D.C.

Links:
http://www.newsonair.com/Main-News-Details.aspx?id=357728
110
http://www.edristi.in/
Nirupam Sen passed away
Question: Veteran politician Nirupam Sen belongs to which political party?
(a) Trinamool Congress (b) Communist Party of India-Marxist (CPI-M)
(c) Communist Party of India (CPI) (d) Communist Party of India- Marxist Leninist (CPI-ML)
Answer: (b)
Related facts:

 Veteran politician Nirupam Sen passed away on 25th December following a cardiac arrest
at a private hospital in Kolkata. He was a senior leader of Communist Party of India-
Marxist (CPI-M). He was 72 years old. Sen retired from active politics in 2013.
 He held the office of Ministry of Commerce and Industries
from 2001 to 2011 under the Chief Ministership of
Buddhadev Bhattacharya in West Bengal government. He is
widely known as architect of West Bengal`s industrial drive.
 The land acquisition movements in Singur and Nandigram
broke out during his tenure. He was also a politburo member
of the party.

Links:
https://indianexpress.com/article/india/veteran-cpim-leader-nirupam-sen-passes-away-5506872/

Sulagitti Narasamma
Question: Sulagitti Narasamma, who died recently was associated with which of the
following?
(a) Politics (b) Literature (c) Social work (d) Sports
Answer: (c)
Related facts:

 On 25 December 2018, renowned social worker Sulagitti Narasamma


died at a private hospital in Bengaluru. She was 98 years old and was
known for her service of midwife to the society.
 She was known as midwife of Krishnapura village in Pavagada taluk of
Tumkur district in Karnataka, she had conducted thousands of deliveries free of cost. She
helped the people of rural areas devoid of medical facilities by providing free services.
 She was awarded India`s fourth highest civilian award Padma Shri in 2018 for her
contribution to society. Prime Minister Modi expressed grief over her demise.

Links:
http://www.newsonair.com/Main-News-Details.aspx?id=357143

Indias Chief Boxing Coach

Question: Who is appointed as Indias chief boxing coach?


(a) Pravesh Punia (b) Vikas Krishnan (c) C A Kuttappa (d) Vijender Singh
Answer: (c)
Related facts:

 C A Kuttappa, a Dronacharya awardee took over as India`s chief


boxing coach post from veteran coach SR Singh.
 Kuttappa is known for coaching some of the most successful Indian
boxers such as Vijender Singh, Shiva Thapa, and M Suranjoy
111
http://www.edristi.in/
Singh and many others.
 Mysuru-based C.A. Kuttappa, serving as Subedar in the Army is a resident of Gokulam.
His first major assignment would be the second edition of the India Open scheduled in
January in Guwahati and the prestigious Strandja Memorial in Bulgaria.

Links:
http://www.newsonair.com/Main-News-Details.aspx?id=357174

Alenka Ermenc
Question: Alenka Ermenc has been appointed as the head of its army, she belongs to-
(a) Romania (b) Serbia (c) Croatia (d) Slovenia
Answer: (d)
Related facts:

 Slovenia became first NATO member state appointing a female officer Major General
Alenka Ermenc as the head of its army replacing Major Gen Alan Geder.
 Ermenc will now lead the country’s 7,500 troops, which also include reserves. She was
promoted to the rank of major general last week, marking the first woman to hold that
position.
 Ermenc, educated in the United Kingdom was also deployed in
Kosovo for six months as part of a peacekeeping mission in 2009.
She started her military career as a member of the Territorial
Forces that played a key role in winning Slovenia’s independence
from Yugoslavia in 1991.

About Slovenia:

 Slovenia, a small eastern European country of just over two million inhabitants is a
republic with a parliamentary democracy and a multi-party system. The president is the
head of state; he is elected by popular vote.Its capital is Ljubljana.
 After World War II, Slovenia became a republic of the renewed Yugoslavia, which though
communist, but distanced itself from Russian influence.
 Slovenia finally got independence from Yugoslavia in 1991. Since then, Slovenia has
successfully integrated into the international community and is becoming a full member of
the European Union and joined NATO.

Links:
https://www.euronews.com/2018/11/28/slovenia-is-first-nato-state-to-appoint-female-army-chief

Mister Supranational 2018


Question: Who won Mister Supranational, 2018?
(a) Rajiv Nautiyal (b) Prathamesh Maulingkar (c) Rajiv Arora (d) Parag Agarwal
Answer: (b)
Related facts:

 Prathamesh Maulingkar scripts history by becoming the first


Asian/Indian to win this year’s Mister Supranational title
 Contestants across the world represented their country for various
sub contests.
 Prathamesh Maulingkar is a Goa based model and an Indian footballer.
 The third edition of Mister Supranational 2018 pageant took place on December 9 in
Krynica-Zdroj, Poland.
112
http://www.edristi.in/
Awards & Honors
Government instituted Subhash Chandra Bose Aapda Prabandhan
Puraskar
Question: Government instituted Subhash Chandra Bose Aapda Prabandhan Puraskar
under which of the following Ministry?
(a) Ministry of Home Affairs (b) Finance Ministry (c) Ministry of
HRD
(d) Environment Ministry
Answer: (a)
Related facts:

 Government has instituted an annual award titled Subhash Chandra Bose Aapda
Prabandhan Puraskar on 23rd January, 2019 on the birth anniversary of Subhash
Chandra Bose.
 The aim of the award is to recognize the immense contribution and selfless service by
many organizations and individuals which goes unacknowledged.
 All Indian citizens and organizations who have excelled in areas of Disaster Management;
like Prevention, Mitigation, Preparedness, Rescue, Response, Relief, Rehabilitation,
Research/ Innovations or Early Warning are eligible for the Subhash Chandra Bose
Aapda Prabandhan Puraskar.

Subhash Chandra Bose Aapda Prabandhan Puraskar 2019:

 For the year 2019, 8th Battalion of National Disaster Response Force (NDRF) located at
Ghaziabad has been selected for the award with a certificate and cash price of Rs. 51
Lakh for its work in Disaster Management.
 Major disasters in which this NDRF battalion participated are Leh Cloud Burst (2011),
Kedarnath Floods (2013), Cyclone Hudhud (2014), Chennai Floods (2015) and Tripura
Floods (2018). The battalion had worked efficiently to neutralize the threat during
Radiation Incident at Mayapuri, Delhi during the year 2010.

Links:
http://pib.nic.in/newsite/PrintRelease.aspx?relid=187690

Maneka Sanjay Gandhi to confer national awards to Anganwadi


Workers under ICDS scheme
Question: In context of AWWs (Anganwadi Workers) under ICDS scheme, cash prize of
National level workers has been increased to?
(a) 10,000 (b) 20,000 (c) 30,000 (d) 50,000
Answer: (d)
Related facts:

 On 07th January 2018, Union Minister for Women & Child


Development, Maneka Sanjay Gandhi, in the presence of Minister
of State for MWCD, Dr. Virendra Kumar conferred National Awards to 97 Anganwadi
Workers (AWWs) for Exceptional Achievements for the year 2017-18.
 These awards are given annually by Government of India at the National and State Levels
since 2000-2001. The cash prize of AWW’s for National Level has been increased from
Rs. 25,000 to Rs. 50,000. State Level award comprises of a cash prize of Rs. 10,000.

113
http://www.edristi.in/
 There has been significant improvement in Anganwadi workers condition due to
improvements like increase in honorarium, better promotion avenues, health insurance
and better work environment among others.
 As per rules & procedures, National Level awards to AWWs are given to the awardees
nominated by States/UTs out of the State/UT Level awardees. The number of
nominations of AWWs at National Level depends on the size of the State/UT and
operational ICDS Projects.
 The award is given with the objective of motivating AWWs and recognises their exemplary
service in the field of child development and related areas under the ICDS Scheme.

Links:
http://pib.nic.in/newsite/PrintRelease.aspx?relid=187204

Padma Awards 2019


Question: Which of the following are conferred with Padma Vibhushan Award 2019?
1) Teejan Bai 2) Anilkumar Naik 3) Balwant Purandare 4) Bachendri Pal
Correct option:
(a) 1, 2 and 4 (b) 1, 3 and 4 (c) 1, 2 and 3 (d) All of the above.
Answer: (c)
Related facts:

 Government announced the list of Padma Awardees of 2019 on 25


January, 2019. The Padma awards comes in row of India`s highest civilian awards after
Bharat Ratna. A total of 112 people from various areas are announced to be awarded with
Padma Awards of 2019.
 The award has 21 women awardees, 11 Foreigner/NRI/PIO/OCI, three posthumous
awardees and first and only Transgender awardees.

Padma Vibhushan:

 Teejan Bai of Chattisgarh for Art-Vocal-Folk. She is a folk songer of Chhattisgarh.


 Foreigner Ismail Omar Guellah of Djibouti for Public Affairs. He is the president of Dijibouti
since 1999.
 Anilkumar Manibhai Naik of Maharashtra for Trade & Industry. He is CEO of construction
company Larsen & Tubro.
 Balwant Moreshwar Purandare of Maharashtra for Acting Theatre. He is well known
theatre personality of Maharashtra.

Padma Bhushan:

 14 people are announced to be awarded this award which famously includes Bachendri
Pal of Uttarakhand for Mountaineering. She is the first Indian women to climb Mt. Everest.
 BJP MP from Madhubani, Bihar and former Central Minister Hukumdev Narayan Yadav
for Public Affairs.
 Noted journalist and critical writer Kuldip Nayyar of Delhi for literature and journalism
posthumously.
 Former Vice Chairman of Rajya Sabha Kariya Munda of Jharkhand for Public Affairs.

Padma Shri:

 A total of 94 people are declared to be awarded with Padma Shri. Some famously include
Actor Manoj Bajpayee, Painter Jyoti Bhatt, Footballer Sunil Chhetri, vocalist Rajeshwar
Acharya.
114
http://www.edristi.in/
 Dancer Nartaki Natraj of Tamil Nadu becomes the first transgender to be awarded Padma
Shri.

About Padma Awards:

 Padma Awards were instituted in the year 1954 and announced every year on the
occasion of Republic Day. It was interrupted during the years 1978 and 1979 and 1993 to
1997.
 Padma Awards – one of the highest civilian Awards of the country, are conferred in three
categories, namely, Padma Vibhushan, Padma Bhushan and Padma Shri. The Awards
are given in various disciplines/ fields of activities, viz. – art, social work, public affairs,
science and engineering, trade and industry, medicine, literature and education, sports,
civil service, etc.
 ‘Padma Vibhushan’ is awarded for exceptional and distinguished service; ‘Padma
Bhushan’ for distinguished service of high order and ‘Padma Shri’ for distinguished
service in any field. The awards are announced on the occasion of Republic Day every
year.
 These awards are conferred by the President of India at ceremonial functions which are
held at Rashtrapati Bhawan usually around March/ April every year.

Links:
https://padmaawards.gov.in/PDFS/2019AwardeesList.pdf

Bharat Ratna 2019


Question: Which of the following personality is announced to be awarded with Bharat
Ratna Award?
(a) Pranab Mukherjee (b) Nanaji Deshmukh (c) Bhupen Hazarika (d) Major Dhyanchand
Correct option:
(a) 1, 2 and 3 (b) 1, 3 and 4 (c) 1, 2 and 4 (d) All of the above.
Answer: (a)
Related facts:

 Government announced the prestigious Bharat Ratna award,


India`s Highest Civilian Award. Former President Pranab
Mukherjee, Late Singer Bhupen Hazarika and Late Social Activist Nanaji Deshmukh will
be awarded the Bharat Ratna.

Pranab Mukherjee:

 Pranab Mukherjee was born in 1935 Bengal and has been a senior Congress leader. He
has held many key posts in Union Cabinet like that of Finance Minister, Defence Minister,
Minister of External Affairs as well as various other important offices.
 He then became the 13th President of Nation in 2012 and was succeeded by Ram Nath
Kovind in 2017. He was awarded with Padma Vibhushan in 2008.

Nanaji Deshmukh:

 Born in October 1916 in Hingoli district of Maharashtra, Nanaji Deshmukh was a


renowned social activist and RSS Veteran. He was the founding member of BJP and also
was MP from Balrampur, Uttar Pradesh.
 He did a lot of work in popularising social restructuring programme in over 500 villages of
UP and Madhya Pradesh. He passed away in 2010. He was awarded with Padma
Vibhushan in 1999.
115
http://www.edristi.in/
Bhupen Hazarika:

 Bhupen Hazarika (1926-2011) was a famous singer, filmmaker and lyricist. He is deeply
respected in North East and particularly in Assam for giving nationwide recognition to
Assamese songs by penning and singing songs on universal peace and communal
harmony.
 He is known as Bard of Brahmaputra. He was awarded cinema`s Dadasaheb Phalke
Award in 1992, Padma Bhushan in 2001 and Padma Vibhushan in 2012. He was also
associated to BJP and contested 2004 Lok Sabha pools from Guwahati constituency
which he lost.

Bharat Ratna Awards:

 ‘Bharat Ratna’, the highest civilian Award of the country, was instituted in the year 1954.
Any person without distinction of race, occupation, position or sex is eligible for these
awards.
 It is awarded in recognition of exceptional service/performance of the highest order in any
field of human endeavour. The recommendations for Bharat Ratna are made by the Prime
Minister himself to the President. No formal recommendations for this are necessary.
 The number of annual awards is restricted to a maximum of three in a particular year. On
conferment of the award, the recipient receives a Sanad (certificate) signed by the
President and a medallion. The Award does not carry any monetary grant.
 Chandrasekhara Venkat Raman, C. Rajgopalachari and Dr. S. Radhakrishnan were the
first recipients of the award in 1954.
 Only two foreign nationals are awarded Bharat Ratna namely Khan Abdul Gaffar Khan
(1987) and Nelson Mandela (1990). Lal Bahadur Shastri was the first to receive this
award posthumously in 1966.

Links:
https://mha.gov.in/sites/default/files/Scheme-BR_1.pdf

Exhibitor of the Year Award


Question: Which of the following organization got the ‘Exhibitor of the Year Award’ at
106th Indian Science Congress held recently?
(a) DRDO (b) ISRO (c) Indian Institute of Science (d) ICSR
Answer: (a)
Related facts:

 The Defence Research and Development Organisation


(DRDO) pavilion received ‘Exhibitor of the Year Award’ at
106th Indian Science Congress held at Lovely Professional University (LPU), Phagwara,
Jalandhar,Punjab.
 The pavilion narrated the saga of self-reliance & national pride with the ‘Make in India’
spirit was a big attraction among people, especially students visiting the pavilion and
getting opportunity to interact with DRDO scientists.
 The DRDO Pavillion at the mega science expo was inaugurated on January 03, 2019 by
Governor of Punjab Shri VP Singh Badnore and Union Science and Technology Minister
Dr Harsh Vardhan.
 DRDO’s outdoor exhibits included surface-to-air missile system – Akash, model of
BrahMos missile, remotely operated vehicle – Daksh, heavy weight torpedo – Varunastra,
Laser Ordnance Disposal System (LORDS) and Vehicle Mounted Dazzler etc.

116
http://www.edristi.in/
 Indoor exhibits of DRDO included models of various missile systems including Prithvi,
Astra, Nag, HELINA, LRSAM, Rustom UAV, MBT Arjun Mk 1A, PINAKA, SONAR
systems & many others.

Links:
http://pib.nic.in/PressReleaseIframePage.aspx?PRID=1559005

76th Golden Globe Awards


Question: Which movie is awarded as the Best Motion Picture in Drama category in the
2019 Golden Globe Awards?
(a) If Beale Street Could Talk (b) Cold War (c) Roma (d) Bohemian
Rhapsody
Answer: (d)
Related facts:

 Prestigious Golden Globes announced its 76th annual


awards on 6th January, 2019. Awards for various categories were given to the
contestants. The ceremony took place in Beverly Hills, California.

Awardees:

 Best Motion Picture- Bohemian Rhapsody


 Best Actor in Motion Picture Drama – Rami Malek for Bohemian Rhapsody
 Best Actress in Motion Picture Drama – Glenn Close for The Wife
 Best Motion Picture Musical or Comedy – Green Book
 Best Actor in Motion Picture Musical or Comedy – Christian Bale for The Vice
 Best Actress in Motion Picture Musical or Comedy – Olivia Colman for The Favorite
 Best Director, Motion Picture – Alfonso Cuaron for Roma
 Best Screenplay, Motion Picture – Green Book
 Best Original Song – Shallow from A Star is Born
 Best Television Series –Drama- The Americans

About The Golden Globe:

 The Hollywood Foreign Press Association (HFPA) is a non-profit organization, the


members of which are international journalists based in Southern California. The group
also is famous for starting and continuing to co-produce the annual Golden Globe
Awards, held in January each year.
 First award ceremony was conducted on January 20, 1944 and was granted by
Hollywood Foreign Correspondents Association to the cinema bests for their works.
Today Golden Globes has 25 categories recognitions; 14 in motion picture and 11 in
television.

Links:
https://www.goldenglobes.com/articles/all-winners-76th-golden-globes

Vanessa Ponce de Leon


Question: Vanessa Ponce de Leon who recently crowned Miss World 2018 belongs to
which of the following country?
(a) Colombia (b) Brazil (c) Argentina (d) Mexico

117
http://www.edristi.in/
Answer: (d)
Related facts:

 Miss Mexico Vanessa Ponce de Leon has been crowned as the Miss
World 2018.This was the 68th edition of the Miss World pageant, held at
the Sanya City Arena in Sanya, China
 She was crowned by 2017 winner, Manushi Chhillar of India. She is the
first Mexican to be crowned as Miss World.
 118 contestants from all around the world participated in the beauty
pageant.
 Vanessa Ponce de Leon holds a degree in international business,
volunteers for “Migrantes en el Camino” and is on the board of directors of a rehab centre
for girls.

Links:
https://www.indiatoday.in/lifestyle/what-s-hot/story/miss-world-2018-winner-is-miss-mexico-
vanessa-ponce-de-leon-1405215-2018-12-08

Malala Yousafzai conferred Gleitsman Award


Question: Recently Malala Yousafzai received the 2018 Gleitsman Award for promoting-
(a) Environmental Conservation (b) Siberian Cranes protection
(c) Handicapped People’s Education (d) Girl’s Education
Answer:(d)
Related facts:

 Malala Yousafzai received the Gleitsman Award-2018 from


Harvard’s Kennedy School. She was honored for her work
Promoting Girls’ Education.
 The Gleitsman Award provides $125,000 for activism that has improved quality of life
around the world. Yousafzai became the youngest person to win the Nobel Peace Prize in
2014.
 She won her Nobel for global work she had done for supporting the schooling for all
children. She founded the nonprofit Malala Fund to support her work
 Now 21, Yousafzai is a student at Oxford University in England. As a teen in Pakistan,
she survived an assassination attempt by the Taliban on 9 October 2012 in Pakistan’s
Swat Valley.

Links:
https://timesofindia.indiatimes.com/world/uk/malala-yousafzai-to-receive-harvard-award-for-
activism/articleshow/66964545.cms

Sunil Bharti Mittal receives ESCP Europe’s Doctor Honoris Causa


Question: Recently, which Indian has been conferred Doctor Honoris Causa, the highest
honor of ESCP Europe?
(a) Sunil Bharti Mittal (b) Mukesh Ambani (c) Ajit Premji (d) Gautam
Adani
Answer- (a)
Related facts

 On 7th December, 2018, Chairman of Bharti Enterprises, Sunil


Bharti Mittal and Carlos Moedes has been conferred Doctor Honoris Causa Degree from
ESCP Europe.
118
http://www.edristi.in/
 Sunil Bharti Mittal became the first Indian to be bestowed with this recognition in ESCP
Europe’s near 200 years history.
 With its campuses in Pari, Berlin, London, Madrid, Turin, and Warsaw, ESCP Europe is
the first business in the world. It was established in 1891.
 In 2007, he was awarded the Padma Bhushan. On June 15, 2016, he was elected as
Chairman of the International Chamber of Commerce.

Links:

https://www.escpeurope.eu/sites/default/files/2018-
12/ESCP%20Europe%20PR_Doctors%20Honoris%20Causa_dec_EN.pdf

Skoch Golden Jubilee Challenger Award


Question: Who has conferred the ‘Skoch Golden Jubilee Challenger Award’ at the 55th
Skoch Summit in New Delhi?
(a) Suresh Prabhu (b) Dharmendra Pradhan (c) Ram Vilas Paswan (d) None of the Above
Answer (a)
Related facts:

 Commerce and Industry Minister Suresh Prabhu has


conferred the ‘Skoch Golden Jubilee Challenger Award’ at
the 55th Skoch summit in New Delhi on 22ndDecember,
2018.
 The Skoch Challenger Awards are the highest independently instituted civilian honors.
 The ‘Commerce & Industry Minister was awarded for his contribution to reforms,
particularly in the power sector reforms.
 A book titled ‘India 2030’ was also released on the occasion.

Links:

http://www.newsonair.com/Main-News-Details.aspx?id=356985

Rafi Award conferred to late music composer Laxmikant and singer


Usha Timothy
Question: Who has been conferred Mohammed Rafi Lifetime Achievement Award, 2018?
(a) Laxmikant Shantaram Kudalkar (b) Poornima Shrestha (c) Vidya Balan
(d) Sushma Shrestha
Answer: (a)
Related facts:

 Late music composer Laxmikant Shantaram Kudalkar and playback


singer Usha Timothy has been conferred 2018 Mohammed Rafi
Award.
 The award was instituted by an NGO Spandan Arts headed by Mumbai BJP Chief Ashish
Shelar.
 Kudalkar, of the Laxmikant-Pyarelal duo, was given the Mohammed Rafi Lifetime
Achievement Award consisting of 1 lakh rupees and a trophy. Usha Timothy was
honoured with the award and 51,000 rupees at a programme held in suburban Bandra.
 As music composers, Kudalkar in partnership with Pyarelal Ramprasad Sharma have
belted out hit tracks like ‘Bindiya Chamkegi’, ‘Main Shayar Toh Nahin’ and ‘Hansta Hua
Noorani Chehra’ among others.

119
http://www.edristi.in/
 Timothy has sung 1,000 songs in multiple Indian languages such as Hindi, Malayalam,
Punjabi, Bhojpuri and Marathi, among others.

Links:

http://www.newsonair.com/Main-News-Details.aspx?id=357153

DIPP SWACHCH BHARAT GRAND CHALLENGE AWARDS


PRESENTED
Question: Swachch Bharat Grand Challenge awards was conferred by which of the
following?
(a) Department of Industrial Policy and Promotion (DIPP)
(b) Department of Public Enterprises (c) Department of Rural
Development
(d) Department of Social justice and Empowerment
Answer: (a)
Related facts:

 On 26th December, 2018, Department of Industrial Policy and Promotion (DIPP) has
conferred Swachch Bharat Grand Challenge awards in four sectors namely Air,
Sanitation, Waste and Water. The awards were given by Secretary DIPP, Ramesh
Abhishek in New Delhi.
 As part of the Swachhta Pakhwada held from 1st to 15th November, 2018, Department of
Industrial Policy and Promotion (DIPP) organized a Swachch Bharat Grand Challenge.
 This was organized to reward innovative solutions by DIPP recognized Start-ups in the
country. 165 applications were received from 70 districts in 22 states. Unique solutions
were received from start-ups who had also filed for intellectual property rights.
 The 1st Prize money for the Swachch Bharat Grand Challenge was Rs. 2 lakh and
2ndPrize, Rs. 1 lakh along with a certificate of appreciation.

Recipients:

 In the Air Sector first and second award was conferred to Maclec Technical Project
Laboratory Pvt. Ltd., Delhi and Small Spark Concepts Technologies Private Limited, Pune
(Maharashtra) respectively.
 In the Sanitation Sector first and second award was given to Altersoft Innovations India
Pvt. Ltd., Cochin (Kerala) and Naturesani Pvt. Ltd., Telangana respectively.
 In the Waste Sector first and second award was given to Sanshodhan An EWaste
Exchange Pvt. Ltd., Telangana and Flycatcher Technologies LLP, Mumbai (Maharashtra)
respectively.
 In the Water Sector first and second award was given REVY Environmental Solutions
Pvt. Ltd., Gujarat and EF Polymer Pvt. Ltd., Rajasthan respectively.

Swachhta Pakhwada:

 Swachhta Pakhwada observed by DIPP in November emphasized the cleanliness on the


lines of Swachch Bharat movement. Various activities were undertaken like cleaning of
corridors, office premises, quadrangles and parking areas of Udyog Bhawan, moping of
glass partitions and cleaning of sections/records/library.
 Along with the Horticulture Department of CPWD the cutting of hedges, pruning of tree
branches, mowing the lawn, display and coloring of plant pots, removal of weeds and
repairs and replacement of broken pots was undertaken.

120
http://www.edristi.in/
Links:

https://dipp.gov.in/sites/default/files/SWACHCHBHARAT_26122018.pdf

Arijit Singh, Shaan honoured with Sangeet Mahasamman award


Question: Sangeet Mahasamman award is conferred by which of the following state?
(a) Maharashtra (b) West Bengal (c) Rajasthan (d) Madhya Pradesh
Answer :(b)
Related facts:

 On 14th December, 2018, the famous singers Arijit Singh


and Shaan were conferred the ‘Sangeet Mahasamman
award ‘ at the inaugural session of the Bengal music and
folk culture festival in Kolkata.
 This award is constituted by the West Bengal Government.
 Apart from this, the renowned Bengali singers like Rupankar Bagchi, Raghab
Chattopadhyay, Manomoy Bhattacharya, Saikat Mitra, Pratik Chowdhury and music
director Debojyoti Mishra were also honoured.

Links:

https://m.dailyhunt.in/news/india/english/cinestaan-epaper-
cinestan/arijit+singh+shaan+honoured+with+sangeet+mahasamman+award+by+west+bengal+
government-newsid-104005386

Women Transforming India Awards


Question: The Third edition of the ‘Women Transforming India Awards’ was organized by

(a) NITI Aayog (b) National Commission For Women
(c) National human rights commission of India (d) None Of the
Above
Answer: (a)
Related facts:

 The third edition of the ‘Women Transforming India (WTI) Awards’ was organized by NITI
Aayog in partnership with MyGov and the United Nations, on 16th December, 2018.
 Upgraded portal of the ‘Women Entrepreneurship Platform’ (WEP portal 2.0) was
launched by Vice President of India, Shri M. Venkaiah Naidu.
 WTI award’s theme is “Women and Entrepreneurship” and fifteen winners have been
chosen through an extensive six-month long process to identify fascinating and inspiring
stories of women entrepreneurs.
 Awards ceremony recognizes the exceptional work of 12 women in critical areas such as
preventing violence against women; bridging the gender gap in education; providing
healthcare; building livelihoods and enterprises for women; and promoting inclusion for
persons with disabilities.
 WTI Awards 2018 received over 2300 nominations this year.

Links:
http://pib.nic.in/newsite/PrintRelease.aspx?relid=170334

121
http://www.edristi.in/
Mamata Banerjee
Question: Chief Minister of which of the following state got the Skoch Chief Minister of
the Year Award for 2017?
(a) West Bengal (b) Odisha (c) Tamilnadu (d) Telangana
Answer: (a)
Related facts:

 West Bengal Chief Minister Mamata Banerjee has been


conferred the Skoch Chief Minister of the Year Award for 2017.
 This award is given to for her contribution in the overall development of the state and
emerging as the best chief minister in the country.
 West Bengal emerges top performing State in Governance. The state has this year
bagged 31 Skoch Awards for its performance across different fields besides implementing
unique projects.
 She has been conferred with the prestigious award on the basis of the state’s
performance in the field of governance, culture, finance, urban and rural development, the
organization.
 While Chandrababu Naidu was awarded with the ‘SKOCH Sprinter of the Year’ and
Suresh Prabhu as the ‘Steady Reformer’.
 SKOCH Consultancy Services Pvt. Ltd. is founded in the year 1997; Sameer Kochhar is
the Chairman of the Group.

Links:
https://economictimes.indiatimes.com/news/politics-and-nation/mamata-banerjee-is-the-skoch-
chief-minister-of-the-year/articleshow/67178399.cms

Prime Minister Shram Awards


Question: How many workers were selected for the Prime Minister Shram Award for the
year of 2017?
(a) 45 (b) 43 (c) 40 (d) 47
Answer: (c)
Related facts:

 A total of 40 workers are selected for the Prime Minister


Shram Award for the year of 2017. The award is given under
Ministry of Labour & Employment.
 This award is given to the workers employed in Departmental Undertakings & Public
Sector Undertakings of the Central and State Governments and Private Sector Units
employing 500 or more workers in recognition of their distinguished performances,
innovative abilities, outstanding contribution in the field of productivity and exhibition of
exceptional courage and presence of mind.
 Nobody was selected for the Shram Rata Award. Shram Bhushan awards were conferred
to 3 nominations receiving a cash award of Rs. 1, 00,000 and a Sanad. Shram Veer/
Veerangana Award were given to 12 nominations who will receive a cash award of Rs.
60,000 and a Sanad. Shram Shree/ Devi Award were given to 16 nominations who will
receive a cash award of Rs. 40,000 and a Sanad. The awardee includes 3 women in the
list.

Links:
http://pib.nic.in/PressReleaseIframePage.aspx?PRID=1556844

122
http://www.edristi.in/
Handicrafts Export Awards
Question: Handicrafts export awards was presented by?
(a) Harsimrat – Kaur Badal (b) Smriti Irani (c) Nazma Heptullah (d) None of the Above
Answer: (b)
Related facts:

 Handicraft export awards were presented at India Expo


Centre & Mart, in Greater Noida on 18th December 2018, by
Union Minister of Textiles, Smriti Irani, for their outstanding
performance in exports in the year 2015-16 and 2016-17.
 The awards were given under different categories to 136 exporters. 70 awards were given
for the year 2015-16 and 66 awards were for the year 2016-17 during the ceremony.
 The handicrafts exports during the year 2015-16 was to the tune of Rs. 21,557.12 crores
which increased to Rs. 24392.39 crore in 2016-17 with an overall increase in exports of
13.15% in rupee terms and 10.52 % in dollar terms. The handicraft exports during 2018-
19 went ahead to the tune of Rs. 16, 825.75 crore during April-November, 2018-19.
 Export Promotion Council for Handicrafts (EPCH) was established under Companies Act
in the year 1986-87 and is a non-profit organization, with an object to promote, support,
protect, maintain and increase the export of handicrafts. it is an apex body of handicrafts
exporters for promotion of exports of Handicrafts

Links:
http://pib.nic.in/newsite/PrintRelease.aspx?relid=186596

Operations & Campaign


Saksham 2019
Question: Saksham 2019 is a campaign related to which of the following Ministry?
(a) Ministry of Petroleum and Natural Gas (b) Ministry of Agriculture
(c) Ministry of Communications and Information Technology
(d) Ministry of Commerce and Industry
Answer: (a)
Related facts:

 On January 16, 2019; ‘Saksham 2019’, an


annual high intensity one-month long people-
centric mega campaign of Petroleum Conservation Research Association (PCRA) was
launched.
 This campaign was organized under the aegis of Ministry of Petroleum and Natural Gas.
 During the month long ‘Saksham-2019’ various interactive programs and activities are
being planned by PCRA, Public Sector upstream/downstream Oil & Gas companies.
‘Saksham’ Cycle Day in 200 cities, Cyclothons, Short Film making contest on MyGov, will
be conducted, with a focus on reaching out to various segments of fuel users.
 Nationwide campaign will be done through Radio, TV, Digital Cinemas, Outdoor media
etc.
 Workshops for drivers of commercial vehicles and Seminars for housewives/cooks on
adopting simple fuel saving measure will be the part of this campaign.
 PCRA will utilize the social media platforms for various customized campaigns through
Facebook, twitter, MyGov platform.

123
http://www.edristi.in/
Links:
http://www.pib.nic.in/Pressreleaseshare.aspx?PRID=1560145

Commission & Committee


SEBI constitutes Research Advisory Committee
Question: SEBI constitutes research advisory committee is headed by?
(a) Dr. Sankar De (b) Dr. Ajit Rnade (c) Mr. Dhirendra Kumar (d) Madan Sabnavis
Answer (a)
Related facts:

 SEBI has constituted a Research Advisory Committee


headed by Dr. Sankar De and comprising of prominent
financial economists and market practitioners as Members
on 8th January, 2019.
 The objective of the committee is Research; which is
relevant for development and regulation of capital market in India and for SEBI keeping in
view the linkage of research to policy making.
 Effective Strategy for Effective execution and delivery of research.
 Promotion/Development/Maintenance of databases relevant for capital market regulation
research.
 Exploring Research Collaborations with external researchers, including other regulators
as well as academic institutions, both domestically and overseas, as appropriate.
 Promotion/development of research proposals in-house as well as by interested external
researchers.

Links:
https://www.sebi.gov.in/media/press-releases/jan-2019/sebi-constitutes-research-advisory-
committee-rac-_41576.html

Planning & Project


Promotion of North East Indian tribal culture
Question: Tribal Research Institutes (TRIs) has been set up recently in which of the
following state/s for promotion of North Eastern tribal culture?
(a) Sikkim (b) Mizoram (c) Arunachal Pradesh & Nagaland (d) All of the above
Answer: (d)
Related facts:

 On 3 January 2019, in written reply to a question in Rajya Sabha


Union Minister of State (Independent Charge) Development of
North-Eastern Region (DoNER), Dr Jitendra Singh notified that an
amount of Rs.23.50 crore has been approved for construction of a
Regional Convention Centre at North East Zone Cultural Centre (NEZCC) Complex,
Dimapur in Nagaland.
 Out of which an amount of Rs.13.00 crore released to NEZCC & Rs.5.72 crore has been
spent towards building of the Convention Centre.
 Government has set up Tribal Research Institutes (TRIs) in Sikkim, Nagaland, Arunachal
Pradesh and Mizoram during 2016-17 and 2018-19, which, inter alia, function as a body
of knowledge & research and preservation of tribal art and culture.

124
http://www.edristi.in/
 On line of development under Tagore Cultural Complex Scheme, from 2013 till date, an
amount of Rs.41.65 crore has been allocated and Rs.22.50 crore spent for construction of
Tagore cultural complexes at Chedema, Aboi, and Dimapur in Nagaland and at Raga and
Upper Subansiri District in Arunachal Pradesh.

Links:
http://pib.nic.in/PressReleseDetail.aspx?PRID=1558428

North Koel River project of Palamu Jharkhand


Question: Prime Minister Narendra Modi has laid the foundation stone of North Koel
(Mangal Dam) Project in which of the following state?
(a) Jharkhand (b) West Bengal (c) Karnataka (d) Odisha
Answer: (a)
Related facts:

 On 5 January 2019,Prime Minister Narendra Modi h laid the foundation stone for the
Mandal Dam Project on the North Koel River. The Mandal Dam Project would be project
of states of Bihar and Jharkhand.
 He laid the Foundation Stone for the revival of North Koel (Mandal Dam) project
(incomplete since 47 years), Kanhar Sone Pipeline Irrigation Scheme, and strengthening
of various irrigation systems and lining works. The cumulatively worth of these projects is
over 3500 crore.
 The Mandal Dam project will help around one lakh 11 thousand hectares of agricultural
land in Jharkhand and Bihar to get irrigation facilities and 24 MW of electricity will be
produced.
 At the event, he notified that 99 big irrigation projects, which have been stalled for
decades, are now being fast tracked, at a cost of almost 90,000 crore rupees.

Links:
http://www.newsonair.com/Main-News-Details.aspx?id=357580

Pradhan Mantri Jan Arogya Yojana


Question: Recently Cabinet approved restructuring of National Health Agency as
National Health Authority for better implementation of which of the following scheme?
(a) National Health Mission (b) Pradhan Mantri Jan Arogya
Yojana
(c) Rashtiya Swasthiya Bima Yojana (d) Janashree Bima Yojana
Answer: (b)
Related facts:

 On 2 January 2019, Union Cabinet approved the restructuring of existing National Health
Agency as “National Health Authority” for better implementation of the Pradhan Mantri Jan
Arogya Yojana (PMJAY).
 The existing multi-tier decision making structure will be replaced with a Governing Board
chaired by the Minister of Health & Family Welfare; this will enable the decision making at
a faster pace, which is required for smooth implementation of the scheme.
 Pradhan Mantri Jan Arogya Yojana (PM-JAY) will provide financial protection (Swasthya
Suraksha) to 10.74 crore poor, deprived rural families and identified occupational
categories of urban workers’ families as per the latest Socio-Economic Caste Census
(SECC).

125
http://www.edristi.in/
 PM-JAY will cover medical and hospitalization expenses for almost all secondary care
and most of tertiary care procedures. The scheme will be cashless & paperless at public
hospitals and empanelled private hospitals.

Links:
https://www.pmjay.gov.in/

Data connectivity services via SMS


Question: Which of the following Indian telecom company is planning to start data
connectivity via messages?
(a) Airtel (b) Jio (c) BSNL (d) Both 1 and 2
Answer: (c)
Related facts:

 BSNL has partnered with French company Be-Bound to offer data connectivity through
SMS in area where there is no internet connectivity or weak signal.
 Both the companies signed business partnership agreement on January 23rd to jointly
market the service in India. The technology will be embedded in mobile applications.
 The app will send command to Be-Bound server in case it does not get data connectivity
for SMS-based connectivity. Be-Bound servers will aid SMS connectivity after getting
command from the app.
 The app will also send SMS from user’s account because for connectivity we need to
SMS to and fro. There is a limit for 100 SMS per day per subscriber.
 Be-Bound has commercially launched the service in France and is running a pilot in
African countries as well.

Links:
https://timesofindia.indiatimes.com/business/india-business/bsnl-french-firm-join-hands-
to-offer-data-services-through-sms/articleshow/67658899.cms

Vision 2040: Aviation


Question: Consider the following statements regarding Vision 2040: Aviation:
1) India’s air passenger traffic is expected to grow six fold to around 1124 million by 2040
2) The Nabh Nirman fund (NNF) with a starting corpus of $2 billion to support low traffic
airports in their initial phases
3) The overall CAGR works out to around 9% in domestic and 7% in international traffic
during FY 2018-2040
Choose the correct options of the following:
(a) 1 only (b) 2 only (c) All of the above (d) 1 and 3 only
Answer: (c)
Related facts:

 On 15 January 2019, The Ministry of civil aviation revealed the Vision 2040 document,
which highlights the growth potential in different sub-sectors of Indian aviation.
 As per the document the total passenger traffic(to, from within India) in India is expected
to rise close to six-fold from 187 million in FY 2018 to around 1124 million in FY 2040.
 This includes around 821 million domestic passengers and around 303 million
international passengers.
 The overall CAGR works out to around 9% in domestic and 7% in international traffic
during FY 2018-2040.
 The government may consider establishing a Nabh Nirman fund (NNF) with starting
corpus of USD 2 billion to support low traffic airports in their initial phases.
126
http://www.edristi.in/
 This document talks about India having a robust 20-year plan that lays out the targets and
path to get there along with time lines and clear accountability.
 Technological developments like artificial intelligence, machine learning, Blockchain,
biometrics, composites, super-alloys, biofuels etc. are changing the face of aviation.
 Initiative like Nabh Nirman (for airport capacity augmentation), Digi Yatra (for paperless
travel) and Airsewa (for online passenger grievance redressal) are bringing in radical
changes.
 The tax structure of Aviation Turbine Fuel (ATF), maintenance, repair and overhaul
(MRO), and aircraft leasing may be gradually aligned with leading global jurisdictions.
 The concept of land pooling may be used to keep land acquisition costs low and to
provide landowners with high value developed plots in the vicinity of the airport.
 It also provided for DGCA may be converted into a fully independent Civil Aviation
Authority, with its own sources of funding and freedom to recruit professional at market-
linked salaries.
 Most transactions with DGCA will be automated with minimal human interface.

Links:
https://www.globalaviationsummit.in/documents/VISION-2040-FOR-THE-CIVIL-AVIATION-
INDUSTRY-IN-INDIA.pdf

MoU between Indian Army & PNB


Question: Recently Memorandum of Understanding (MoU) between Indian Army & PNB is
signed which aims to launch which of the following scheme?
(a) Rakshak Plus (b) Antyodaya (c) Jan Arogya Abhiyaan (d) Make in India
Answer: (a)
Related facts:

 On 8 January 2019,a Memorandum of Understanding


(MoU) was signed between the Indian Army and PNB
(Punjab National Bank) on the Defence Salary Package.
 Adjutant General, Lt. Gen. Ashwani Kumar and Naveen Kumar headed the army and
bank respectively.
 It aims to launch ‘RAKSHAK PLUS’ Scheme for serving peers and veterans, providing
cheque overdraft facility ranging from Rs. 75,000 to Rs. 3,00,000 multicity ‘At Par’
cheques, instant credit of outstation cheques , preferential allotment of lockers at
authorized AMC (Access Control and Perimeter Protection Systems for Military
Installations) & free SMS alerts.

Links:
http://pib.nic.in/newsite/PrintRelease.aspx?relid=187348

Direct Bus Service to Jammu & Kashmir


Question: Which two cities of Uttar Pradesh and Jammu & Kashmir are connected by
direct bus service?
(a) Lucknow and Katra (b) Ayodhya and Katra (c) Gorakhpur and Katra (d) Prayagraj and Katra
Answer: (d)
Related facts:

 A direct air conditioned bus service from Prayagraj to Katra in Jammu & Kashmir and
back started on January 7, 2019. The first bus was flagged off by Uttar Pradesh Chief
Minister Yogi Adityanath.

127
http://www.edristi.in/
 This will provide pilgrims coming to Kumbh in Prayagraj this
month a direct connectivity to visit Vaishno Devi shrine in
Katra, Jammu.
 The service started with an inter-state bus pact signed
between UP State Road Transport Corporation (UPSRTC) and
its J&K counterpart. A MoU with Himachal Pradesh was also
signed for similar arrangement.
 The distance of about 1,387 km between Prayagraj and Katra will be covered in almost
24 hours.

Links:

http://www.newsonair.com/Main-News-Details.aspx?id=357718

Integrated government online Training Programme


Question: Recently Integrated Government Online Training Programme was launched by
government which is developed by-
(a) Department of Personnel and Training (DoPT) (b) Department of Atomic Energy (DAE)
(c) Department of Space (d) Indian Space Research Organization (ISRO)
Answer: (a)
Related facts:

 On 20th December, 2018, Union Minister of State Dr Jitendra


Singh launched the iGOT (Integrated Government Online Training Programme)
developed by Department of Personnel and Training, Ministry of Personnel, Public
Grievances & Pensions.
 It will strengthen the existing training mechanism with online module-based training
approach coupled with certification.
 The National Training Policy-2012 mandates that all civil servants will be provided training
to prepare them with the aptitude for their current or future jobs.
 Integrated Government Online Training Programme (iGOT) launched by Minister will
further strengthen the existing training mechanism with online module-based training
coupled with certification. This will enhance training inputs to government servant on site
and on flexitime basis.
 The e-Learning mode provides relevant competence to grow & flourish (equal
opportunities) for training to a large number of civil servants spread all over the country.
 Latest technology & technological system provides huge resources of learning material
and online courses, enabling the individuals with enormous choices to flourish & explore
flexibility in learning.
 The training courses available under this programme will be accessible through DoPT’s
web portal. DoPT has worked together with JPAL, South Asia and MIT, USA to launch
MITx Micro Master Programmes for the government servants.

Links:
http://pib.nic.in/newsite/PrintRelease.aspx?relid=186683

MoU between India & France


Question: Recently in which area MoU between India & France is signed?
(a) Energy efficiency / Energy conservation (Science & technology)
(b) Health (c) Space (d) Telecommunication

128
http://www.edristi.in/
Answer: (a)
Related facts:

 On 6th of December, The Union Cabinet headed by Prime Minister Shri Narendra Modi
has been informed of Memorandum of Understanding (MoU) between India and France in
the field of energy efficiency / energy conservation.
 The MoU was signed on 17th October 2018.
 The MoU is a Science and Technology agreement, which includes knowledge sharing &
feedback in the nature of technical assistance only.
 It will generate consciousness on energy efficiency.
 Development of fundamental tools for collection, use and analysis of CO2 emissions and
green house gas emissions data for tracking global emissions for INDC.
 It will promote Research and Development, and demonstrate energy efficient technology
& technological systems.Development of viable mobility with primary focus on electrical
mode of transport. The objective of the MoU is the future collaboration regarding in a pilot
project to provide SECI an e-vehicle charging station with embedded batteries, powered
by solar panels and optimized connection to the grid in order to support the Indian
governments’ ambitious plan for the deployment of electrical vehicles by maximizing solar
mobility and minimizing its grid impact.

Links:
http://pib.nic.in/newsite/PrintRelease.aspx?relid=186185

Treaty & Agreements


MoRD-Maruti Suzuki Deal
Question: Which of the following is flagship placement linked skill-training programme
under the Ministry of Rural Development (MoRD)?
(a) Deen Dayal Upadhyay Grameen Kaushalya Yojna (DDU-GKY)
(b) National Apprenticeship Promotion Scheme
(c) Craftsmen Training Scheme (d) None of the above
Answer: (a)
Related facts:

 On 19 December 2018, Ministry of Rural Development (MoRD) signed a memorandum of


understanding with Maruti Suzuki India Ltd for training rural youth for skill development
under Deendayal Upadhyaya Grameen kaushalya Yojana (DDU-GKY).
 Deen Dayal Upadhyay Grameen Kaushalya Yojna (DDU-GKY) is the flagship placement
linked skill-training programme under the Ministry of Rural Development.
 This partnership between the Ministry and the leading industry from the automotive sector
will provide training to at least 5000 candidates in two years with assured placement to
the rural youth of India.One of the ways in which DDU-GKY aims to achieve this is
through the “Champion Employers” policy. The Champion Employers are the industry
leaders who have the potential to provide training and captive employment to the DDU-
GKY candidates.The policy seeks a strategic alignment of objectives of DDU-GKY with
the HR Strategy of organizations, which have a large potential to absorb trained
manpower. It was launched on 25 September 2014 by Union Ministers Nitin Gadkari and
Venkaiah Naidu on the occasion of 98th birth anniversary of Pandit Deendayal Upadhyay.

Links:
https://rural.nic.in/press-release/mord-signs-mou-maruti-suzuki-india-ltd-training-rural-youth-
skill-development-under

129
http://www.edristi.in/
Conference
Science Communicators’ Meet at the Indian Science Congress 2019
Question: Union Minister Ravi Shankar Prasad in an Inaugural meet ‘Science
Communicators’ at the Indian Science Congress, 2019 informed that India is at ……
number in patent filing ?
(a) 1 (b) 2 (c) 3 (d) 10
Answer: (d)
Related facts:

 As a part of Indian Science Congress 2019 organised at Lovely


Professional University in Jalandhar(Punjab), Science
Communicators’ Meet held from 5th to 6th January. It was inaugurated by the Union
Minister for Law & Justice and Electronics & Information Technology, Ravi Shankar
Prasad.
 Science Communicators’ Meet is held every year at the ISC for understanding and
promoting effective communication of science to diverse audiences with the objective of
disseminating science information to all stakeholders.
 Union Minister for Law & Justice and Electronics & Information Technology, Shri Ravi
Shankar Prasad notified that India is now at the 6th position in scientific publications and
10th in patent filing. India’s ranking in global innovation index has jumped from 81st in
2015 to 60th in 2017.
 Digital payments have surged to 8 folds in the last five years to 2070 crores. There has
been 2048% growth in Aadhar Enabled Payment System transactions while transactions
in UPI grew 1500 times in the last 24 months. Digital India has bridged the digital divide
between urban and rural India.

Links:
http://pib.nic.in/newsite/PrintRelease.aspx?relid=187207

Keynotes of MHRD at 106th Indian Science Congress


Question: In 2019, how much amount the Ministry of Human Resource & Development is
spending to promote research & development through innovations?
(a) Rs 33,000 Crore (b) Rs 29,000 Crore (c) Rs 30,000 Crore (d) Rs 31,000 Crore
Answer: (a)
Related facts:

 Addressing a valedictory session of the 8th Women Science


Congress during the ongoing the 106th Indian Science Congress
at the Lovely Professional University Jalandhar(Punjab), Union Minister of Human
Resource Development, Shri Prakash Javadekar notified that in 2019, Ministry of Human
Resource and Development is spending nearly Rs 33,000 Crore for the purpose of
promoting research& development through innovation.
 Already a fund has been set up under Higher Education Financing Agency (HEFA) to
provide assistance to the research laboratories.
 A fund of Rs 200 crore is allocated this year for the innovative solutions and around 1000
colleges across the country are being equipped with innovation hubs while to encourage
social science research Rs 250 crore has been allocated.
 Government has also launched SPARC mission to assist and encourage students to carry
on their ambitious research projects within India instead of going abroad. Ministry has
also planned to provide scholarship of Rs 1 Lakh per student per month for research and
innovation work for this.

130
http://www.edristi.in/
 With the motive of obtaining leadership in research & development through innovations
government has proposed to establish overall six research parks in different IIT’s of
country where students and teachers are doing their research work.

Links:
http://pib.nic.in/newsite/PrintRelease.aspx?relid=187213

International Civil Aviation Organisation Predicts 100% increase in


Global Air travel by 2030
Question: As per sources, International Civil Aviation Organisation Predicts 100%
increase in Global Air travel by year?
(a) 2021 (b) 2022 (c) 2024 (d) 2030
Answer: (d)
Related facts:

 The International Civil Aviation Organization has envisioned


that a 100 percent increase in global air travel by the year
2030, creates a need for extensive efforts to bring the entire aviation ecosystem to pace.
 Emerging economies like India are the biggest driver of demand in Aviation Industry,
which will need to cater for air travel for nearly 6 billion people. The Global Aviation
Summit provides an exclusive platform for collaborative efforts on a global magnitude.
 To ride this strong tailwind of growth and pave the way forward for the sector, a two day
Global Aviation Summit was organised by the Ministry of Civil Aviation (MoCA) in
collaboration with FICCI on 15-16 January 2019. The theme of summit was “Flying for all”
which took place in Mumbai, the financial capital of India.
 An official mobile application for Global Aviation Summit 2019 was launched earlier by
Shri Jayant Sinha, Minister of State for Civil Aviation, on 19th December, 2018.

Links:
http://www.pib.nic.in/PressReleaseIframePage.aspx?PRID=1558966

Vibrant Gujarat Global Summit 2019


Question: Consider the following statements in reference to the Vibrant Gujarat Global
Summit 2019 and choose the correct statement/s in the options given below:
(1) This was the 10th Edition of Vibrant Gujarat Global Summit.
(2) Prime Minister inaugurated the summit in Gandhinagar on 18th January, 2019.
(3) The summit witnessed participation of 5 head of states and over 42,000 delegates.
(4) It was started in 2003 by the then Chief Minister Narendra
Modi.
Choose the correct options:
(a) 1, 2 and 3 (b) 1, 3 and 4 (c) 2, 3 and 4 (d) All of the above
Answer: (c)
Related facts:

 Vibrant Gujarat Global Summit started in Gandhinagar, Gujarat with the inauguration by
Prime Minister Narendra Modi at Mahatma Mandir Exhibition cum Convention Centre on
18th January, 2019.
 This event was held between 18th and 20th January, 2019. This was the 9th edition of
this summit which started in 2003 under the guidance of Narendra Modi who then the
Chief Minister of Gujarat.
 This business summit held meetings, seminars, presentations for the growth of the state
with investment from key industrialists in the state as well as collaborations with various
131
http://www.edristi.in/
countries in trade and commerce. It has also grown as a platform to discuss and debate
for the major global socio-economic issues in the world.
 Five head of states from Uzbekistan, Rwanda, Denmark, Czech Republic and Malta along
with the captains of industry from various sectors & over 42,000 delegates from India and
abroad took part in the event.

Key highlights of the summit:

 15 partner countries of the summit are Australia, Canada, Czech Republic, Denmark,
France, Japan, Morocco, Norway, Poland, South Africa, South Korea, Thailand,
Netherlands, UAE and Uzbekistan as well as national partners CII and FICCI.
 A total of 28,360 Memorandum of Understanding (MoU) were signed during the summit of
which 21,889 were related to Micro Small & Medium Enterprises (MSME) sector. This is
expected to create over 21 lakh jobs in the state.
 Adani group led by Gautam Adani will invest Rs. 55,000 crore in next five years in Gujarat
which includes world`s largest solar plant, a copper plant, a cement unit and many other
projects.
 Reliance Industries Limited led by Mukesh Ambani will invest Rs. 3 lakh crore in next 10
years in projects ranging from energy and petrochemicals to digital business.
 Aditya Birla Group Chairman announced investment worth Rs. 15,000 crore in Gujarat in
next three years on capacity expansion and setting up of new units.
 Japanese Automaker Suzuki announced to commission its third plant in Gujarat by 2020
and will introduce hybrid vehicles in India with technical support from Toyota.
 Chromeni Steels to invest 3 billion USD in Dholera town of Gujarat for manufacturing
stainless steel and EV Batteries.
 Major events during the meet include a Round-table interaction with the heads of global
funds, ‘Africa Day’, MSME convention, Roundtable for Opportunities in Science &
Technology, Engineering and Mathematics (STEM) Education & Research.
 The closing ceremony was held in the presence of Vice President M Venkaiah Naidu.

Links:
https://indianexpress.com/article/india/vibrant-gujarat-summit-live-updates-narendra-
modi-inauguration-5544099/

Swachhagrahi Sammelan
Question: Which Union Minster addressed a specially convened Swachhagrahi
Sammelan in Prayagraj?
(a) Uma Bharti (b) Narendra Modi (c) Smriti Irani (d) Ram Vilas
Paswan
Answer: (a)
Related facts:

 Minster of Drinking Water and Sanitation, Sushree Uma Bharti reviewed the interventions
undertaken for better sanitation as well as awareness generation about Swachh Bharat
Abhiyan in Kumbh Mela premises at Prayagraj.
 Uma Bharti also addressed a specially convened Swachhagrahi Sammelan appreciating
the efforts made by Mela and district authorities for making the Mela clean and safe.
 Swacchagrahis are deployed in the Kumbh Mela to ensure cleanliness. They have been
assigned the work to monitor toilet cleanliness and awareness. The monitoring will be
done on an ICT app through which the cleanliness status will be recorded.
 Around 1,500 swacchagrahis are working actively in the Mela hailing from different
districts of Uttar Pradesh. 50 kits were distributed amongst the swacchagrahis and an
exhibition on the concept of Swacch Village was inaugurated in the Mela.
132
http://www.edristi.in/
 She also mentioned the good efforts of Uttar Pradesh government for making their state
ODF and increasing their sanitation coverage from 48% to 100%. State is now focusing
ODF- plus.
 1.22 lakh public toilets and 20,000 dustbins have been installed at the Kumbh Mela this
year.

Links:
http://pib.nic.in/newsite/PrintRelease.aspx?relid=187577

Regional Conference on Deendayal Disabled Rehabilitation Scheme


(DDRS)
Question: Regional conference on DDRS was organized by which of the following
Ministry?
(a) Ministry of Social Justice and Empowerment
(b) Ministry of HRD (c) Ministry of Home Affairs (d) Environment Ministry
Answer: (a)
Related Facts:

 A Regional conference for Deendayal Disabled Rehabilitation Scheme (DDRS) was


organized at Nehru Centre, Mumbai on 17th January, 2019, by Ministry of Social Justice
and Empowerment.
 Conference covered Programme Implementing Agencies (PIAs) from the Western Region
of the country. NGOs, State Government representatives of Gujarat, Maharashtra,
Madhya Pradesh and Goa were present at the conference.
 It was the second phase of a series of Regional conference to be held across the country
and will culminate with national conference in New Delhi.
 Aim of scheme is to ensure equal opportunities, equity, social Justice & empowerment
and to encourage voluntary action for ensuring effective implementation of the Right of
persons with Disabilities (RPwD) Act 2016- Deendayal Disabled Rehabilitation Scheme
(DDRS)
 DDRS is a central sector scheme of Government of India which is being implemented
since 1999 to provide financial assistance to NGOs working for education and
Rehabilitation of persons with disabilities.
 The scheme to promote voluntary action for persons with disabilities was revised and
renamed as Deendayal Disabled Rehabilitation Scheme (DDRS) w.e.f. 01.04.2003.
 The scheme was again revised in 2018, and since 1st April, 2018 it is being implemented
as revised.

Links:
http://pib.nic.in/newsite/PrintRelease.aspx?relid=187566

106th Indian Science Congress


Question: Which of the following statement/s is/are correct in reference to the Indian
Science Congress?
1) This is a 5 day 109th session of Indian Science Congress
2) It is organized in Jalandhar at Lovely Professional University
3) The theme for this edition is ‘Future India- Science and
Technology’
4) Indian Science Congress Association organizes this congress
Correct option is:
(a) 1, 2 and 3 (b) 2, 3 and 4 (c) 1, 3 and 4 (d) All of the above.
133
http://www.edristi.in/
Answer: (b)
Related facts:

 The 106th session of Indian Science Congress was held at Lovely Professional University
(LPU) in Jalandhar, Punjab from 3-7 January 2019. The theme of 2019 Congress is
Future India: Science & Technology.
 It was inaugurated by Prime Minister Narendra Modi. Many prominent dignitaries
including officials of DRDO, ISRO, AIIMS, UGC, Universities of UK, US and other
countries also took part in the event.
 This 5 day event witnessed over 100 conferences and event of scientific and
technological origin. The prominent guests include Nobel laureate in biochemistry
Thomas Sudoph, Nobel laureate in Chemistry and Physics Avram Hershko and Fredrick
Duncan Haldane respectively.
 Indian Science Congress Association (ISCA) is a premier scientific organization of India
with headquarters at Kolkata, West Bengal. The association started in the year 1914 in
Kolkata and it meets annually in the first week of January. It has a membership of more
than 30,000 scientists.
 PM Narendra Modi mentioned major achievements of Indian Science in 2018 which
included production of aviation grade biofuel; Divya Nayan – a machine for visually
impaired; inexpensive device for diagnosing cervical cancer,TB and dengue etc; a real
time landslide warning system in the Sikkim- Darjeeling region etc.
 He also made announcement for the approval of National Mission on Interdisciplinary
Cyber Physical System by Union Government worth Rs. 3,600 crore. He also notified the
preparation of sending three individual to space by Gaganyaan in 2022.

Links:
https://www.thehindu.com/sci-tech/pm-to-inaugurate-indian-science-congress-at-
jalandhar/article25896841.ece

Law & Justice


Lok Sabha passes Indian Medical Council (Amendment) Bill
Question: Recently Lok Sabha passed Indian Medical Council (Amendment) Bill 2018,
which will come into force from?
(a) 26 September 2018 (b) 27 June 2018 (c) 9 September 2018 (d) 9 May 2018
Answer: (a)
Related facts:

 On 14th December 2018, Mr. Jagat Prakash Nadda, Minister of


Health and Family Welfare introduced Indian Medical Council
(Amendment) Bill, 2018 in Lok Sabha. It shall be deemed to be effective from 26th
September 2018.
 Bill amends Indian Medical Council Act, 1956 and replaces it with the Indian Medical
Council (Amendment) Ordinance, 2018.
 The 1956 Act provides for replacement of the MCI (Medical Council of India) and its
reconstitution within a period of three years. The Bill amends this provision to provide for
the replacement of the MCI for a period of one year.
 The Act aims to provide for the Board of Governors which will be formed by seven
members (maximum). The members will be appointed by the Central Government and
they will be persons of prominence in the field of medical education.
 The Bill amends the plan so as to allow for efficient administrators.

134
http://www.edristi.in/
 Also the Bill provides for the Board of Governors to be assisted by a Secretary General
appointed by the government.

Links:
http://pib.nic.in/newsite/PrintRelease.aspx?relid=186227

Lok Sabha passes 124th Constitution Amendment Bill


Question: Which constitutional amendment bill passed quota of 10% for general
category?
(a) 123rd Constitutional Amendment Bill (b) 124th Constitutional Amendment Bill
(c) 125th Constitutional Amendment Bill (d) 126th Constitutional
Amendment Bill
Answer: (b)
Related facts:

 On 8th January 2019,Lok Sabha passed the Constitution (124th Amendment) Bill 2019
providing for ten per cent reservation to Economically Weaker Sections (EWS) in the
General category.
 The legislation namely the Constitution 124th Amendment Bill facilitates reservation for
EWS people in the upper caste in the direct recruitments and admission in higher
educational institutions.
 After an over four and half hours of debate, 323 Lok Sabha members voted in support of
the bill while only 3 voted against it.
 This is known to be as the 103rd Constitutional Amendment Act, 2018 enacting Economic
Reservation in India.

Links:
http://www.newsonair.com/Main-News-Details.aspx?id=357764

Witness Protection Scheme


Question: Which of the following statement/s is/are correct regarding Witness Protection
Scheme?
1) This scheme was formulated by Central Government
2) Supreme Court approved the draft of first Witness Protection Scheme
3) This scheme was finalized with special consultation of National Investigation Agency
(NIA)
4) This scheme is launched for protection of witness by maintaining secrecy over its
name
Correct option is:
(a) 1, 3 and 4 only. (b) 1, 2 and 4 only. (c) 2 and 4 only. (d) All of the above.
Answer: (b)
Related facts:

 The Supreme Court on 5th December, 2019, approved the draft of India’s first Witness
Protection Scheme. It was approved by the double bench consisting of Justices A.K. Sikri
and S. Abdul Nazeer asking the Center, States and Union Territories to implement it with
immediate effect.
 Supreme Court in its judgment said that it is a clear violation of Article 21(Protection of life
and personal liberty) of constitution if someone is pressurized or threatened to life for
testifying. The court said it shall be a law under Article 141(law declared by Supreme
Court is binding on all courts)/142(Enforcement of decrees and orders of Supreme Court)
of constitution until any parliamentary and/or state legislation is enacted on this subject.
135
http://www.edristi.in/
 It was drafted by centre with suggestions and inputs from 18 states/union territories,
National Legal Service Authority (NALSA), Bureau of Police Research and Development
(BPRD), open sources including civil society, three High Courts as well as police
personnel.
 The scheme classifies witness as per threat perception into 3 categories:
 Category A: It includes cases which has threat to life of witness or its family during trial,
investigation or later.
 Category B: It includes cases having threat to safety, reputation or property of witness
only during investigation or trial.
 Category C: It includes cases with moderate threat extending to harassment of the
witness or its family members during trial, investigation or later.
 It includes coverings of different kind of protections such as camera trial to avoid physical
confrontation of witness and accused, physical protection and anonymising of testimony.
Additional protective features regarding identity protection like giving a new identity, new
address as well as ‘parentage’ to witness is also enlisted.
 A Witness Protection Fund will also be established by states and union territories which
are to be funded by respective governments and by donations. Witness Deposition
Complexes are to be established where witness can depose without confronting accused.
 This issue has a long history; in 2003 Justice V Maliwat Committee on criminal justice
system recommended its enactment. This issue got fresh when court was hearing a PIL
(Public Interest Litigation) on protection of witness in cases against Asaram Bapu in 2018.
 Countries such as US, United Kingdom, China and Italy etc. have Witness Protection
Scheme.

Links:
https://indianexpress.com/article/what-is/witness-protection-scheme-supreme-court-5480930/

Lok Sabha clears amended Consumer Protection Bill


Question: Recently Consumer Protection Bill was passed in Lok Sabha. It Seeks –
(a) To replace Consumer Protection Act, 1986 (b) To replace Consumer Protection Act,1991
(c) To replace Consumer Protection Act,1982 (d) To replace Consumer
Protection Act,1981
Answer: (a)
elated facts:

 On 20 December 2018 Lok Sabha passed the Consumer Protection Bill, 2018 that seeks
to replace the Consumer Protection Act, 1986.
 The Bill enforces consumer rights, and provides a guided way for redressal of complaints
regarding defect in goods and deficiency in services.
 Consumer Disputes Redressal Commissions will be set up at the District, State and at
National levels for determining consumer complaints.
 District Commissions will redress complaints involving claims worth one crore rupees,
which was 20 lakh rupees earlier, at the State Commissions complaints involving claims
worth 15 crore rupees, which was 1 crore rupees earlier will be heard. Complaints over
and above that limit would be handled by the National Commission.
 The Bill will have the power to set up a Central Consumer Protection Authority to promote,
protect and enforce consumer rights as a class.
 The Bill also seeks to impose stringent punishment for food adulterations.

Links:
http://www.newsonair.com/Main-News-Details.aspx?id=356913

136
http://www.edristi.in/
Flight & Maritime Connectivity Rules, 2018
Question: What is the minimum height ascertained for IFMC( in-flight and maritime
connectivity) services in the notified Flight and Maritime Connectivity Rules,2018?
(a) 1500 metres (b) 2200 metres (c) 2800 metres (d) 3000 metres
Answer: (d)
Related facts:

 On 14th December 2018, the Central Government has notified


Flight and Maritime Connectivity Rules, 2018 in exercise of the
powers conferred by section 4 read with section 7 of the Indian
Telegraph Act, 1885.
 In these rules IFMC(in-flight and maritime connectivity) services has been defined
allowing phone calls and access to internet during air travel and ship voyage within the
Indian territory.
 IFMC services can be provided by a valid telecom licence holder in India through
domestic and foreign satellites having the permission of the Department of Space.
 The IFMC licenses will be granted against annual fee of Re 1 for a period of 10 years and
the permit holder will have to pay licence fees and spectrum charges based on revenue
earned from providing services.
 The IFMC service provider shall provide the operation of mobile communication services
in aircraft at minimum height of 3000 meters in Indian airspace to avoid interference with
terrestrial mobile networks.

Links:
http://dot.gov.in/sites/default/files/2018_12_17%20AS%20IFMC_0.pdf?download=1

Year, Day & Week


Republic Day Celebration 2019
Question: Which of the following award is conferred to martyr Lance Naik Nazir Wani on
Republic Day, 2019?
(a) Shaurya Chakra (b) Param Veer Chakra (c) Ashoka Chakra (d) Kirti Chakra
Answer: (c)
Related facts:

 India celebrated its 70th Republic Day on January 26, 2019. This momentous day
observed parade in Rajpath alongwith various other cultural programmes and facilitations.
 India hosted South African President Cyril Ramaphosa as the Chief Guest of the event.

Event Highlights:

 The celebration started with PM Modi paying tribute at Amar Jawan Jyoti after which
National Flag was unfurled by President Ram Nath Kovind and National Anthem was
played with 21 gun salute.
 Miltant turned army men Lance Naik Nazir Wani of Jammu & Kashmir was conferred
India`s highest peacetime gallantry Award Ashoka Chakra posthumously by President. It
was received by his Mother and Wife.
 The parade was commanded by Lieutenant General Asit Mistry, General Officer
Commanding, Headquarters Delhi Area.
 This year all women marching contingent of the Assam Rifles took part for the first time
which was led by Major Khusboo Kanwar displaying Nari Shakti.
137
http://www.edristi.in/
 The contingent of Gwalior Lancers 61 Cavalry led by Major Apurva Dabhade also
paraded. It is the only active serving horse Cavalry Regiment in the world which was
raised on 01 August 1953 with the amalgamation of six State Forces’ Cavalry Units.
 Tanks and missiles of Army also paraded in the event. This includes T- 90 Bhishma,
Ballway Machine Pikate (BMP-II/IIK), Surface Mine Clearing System (SMCS), K-9 Vajra-
T, M 777 A2 Ultra Light Howitzer (ULH), Transportable Satellite Terminal (TST), Akash
weapon system.
 Defence Research and Development Organisation (DRDO)
displayed Medium range Surface to Air Missile (MRSAM).
 Twenty-two tableaux, comprising of 16 from different States
& Union Territories and six Ministries, Departments and
other institutions displaying life, times and ideals of Mahatma
Gandhi were displayed. This year is his 150th birth
anniversary.
 Twenty-six children including six girls and 20 boys Awardees of Pradhan Mantri Rashtriya
Bal Puruskar 2019 for their exceptional achievement also graced the occasion.

Awards:

 Various awards for Gallantry and other distinguished works were given to brave soldiers
and personnel.
 Param Vishisht Seva Medal was given to 19 personnel including Army Chief Gen. Bipin
Rawat.
 Kirti Chakra was awarded to 2, Uttam Yudh Seva Medal to 3 and Ati Vishisht Seva Medal
to 32 personnel. Various other awards were also given in different category.
 Tripura bagged the best Tableau Award. The tableau had theme ‘Empowering Rural
Economy the Gandhian Way’. Jammu & Kashmir and Punjab came second and third.
 The Central Industrial Security Force (CISF) won the Best Tableau Trophy of Republic
Day Parade.

Links:
http://pib.nic.in/newsite/PrintRelease.aspx?relid=187793

PM remembers Swami Vivekanand on his Jayanti


Question: National Youth day celebrated on?
(a) 12th January (b) 12th February (c) 12th March (d) 23rd May
Answer: (a)
Related facts:

 National Youth Day is observed every year on 12th January to mark the birth anniversary
of social reformer, philosopher, thinker and youth icon of India, Swami Vivekananda.
 The Prime Minister, Shri Narendra Modi paid homage to Swami Vivekananda, on his
Jayanti.
 Swami Vivekananda emphasized on the ideals of service and renunciation. His belief in
Yuva Shakti was unwavering.
 Swami Vivekananda was born as Narendranath Dutta on 12th January, 1863, and died on
4 July 1902.

Links:
http://pib.nic.in/newsite/PrintRelease.aspx?relid=187443

138
http://www.edristi.in/
Vijay Diwas
Question: Vijay Diwas is observed on-
(a) 21st December (b) 19th December (c) 17th December (d) 16th December
Answer- (d)
Related facts:

 On 16th December, 2018, 47th Vijay Diwas was celebrated


throughout country commemorating India’s victory over Pakistan in 1971 war.
 On this day, the martyrs who sacrificed their lives in this war were paid homage.
 In this war, General of Pakistan Army General Amir Abdullah Khan Niazi surrendered
unconditionally to the allied forces consisting of Indian Army led by Lieutenant General
Jagjit Singh Aurora in Dhaka with about 93 thousand soldiers.
 After this war, East Pakistan became independent, now known as Bangladesh.

Links:

http:// http://www.newsonair.com/Main-News-Details.aspx?id=356746

National Mathematics Day


Question: National Mathematics Day is observed on-
(a) 22nd December (b) 04th December (c) 12th November (d) 18th December
Answer: (a)
Related facts:

 National Mathematics Day is observed on22 December every year to


honor the birth anniversary of the famous mathematician Sir
Srinivasa Ramanujan.
 This day recognizes the contribution of mathematician Sir Srinivasa
Ramanujan to the analytical theory of numbers and worked on elliptic
functions, continued fractions, and infinite series. Various
programmes seminar, workshop, symposium, lecture are organized
at schools, colleges and universities.
 The main objective is to provide a forum to the students to pursue their natural curiosity
and stimulate their interest in mathematics, developing in students the habit and skills to
use mathematics, to remove maths phobia’ from the minds of the students and to improve
teaching methodology of mathematics at schools.
 The Union Government of India by then Prime Minister Manmohan Singh on 26thFebruary
2012 at Madras University, declared to celebrate 22nd December, as the National
Mathematics Day.

Links:

https://en.wikipedia.org/wiki/National_Mathematics_Day_(India)

International Human Solidarity Day


Question: International Human Solidarity Day is observed on-
(a) 20th December (b) 21st December (c) 12th December (d) 28th December
Answer: (a)
Related facts:

139
http://www.edristi.in/
 International Human Solidarity Day is observed on 20th December every year to celebrate
unity in diversity and to raise public awareness of the importance of solidarity.
 Its objective is to eradicate poverty and promote human and social development in
developing countries, in particular among the poorest segments of their populations.
 This day was formally established by UN General Assembly on 22nd December 2005.
 Solidarity is identified in the Millennium Declaration as one of the fundamental values of
international relations in the 21st Century. The new Sustainable Development
Goals (SDGs) agenda is centred on people & planet, underpinned by human rights and
supported by a global partnership determined to lift people out of poverty, hunger and
disease.

Links:

http://www.un.org/en/events/humansolidarityday/background.shtml

Books
Book ‘Early Indians:the story of our Ancestors & where we came from’
Question: “Early Indians: the story of our Ancestors & where we came from” is released
recently. Who is the author of this book?
(a) Tony Joseph (b) Charles C Royce (c) Clay McCauley (d) Franz Boas
Answer: (a)
Related facts:

 A book titled “Early Indians: The Story of Our Ancestors and Where We Came from” over
the ancestry of Indians authored by Tony Joseph is released. This book looks for the
footprints of first modern humans in India.
 The book mentions that Indian population is result of four major migrations in history
including the Aryan’s migration in prehistory times. It is claimed in the book that first
modern human arrived in India as a part of Out of Africa migration around 65,000 years
ago which holds 50-65 percent of the Indian ancestry today.
 The second Major migration took place in 9000 to 5000 years ago with movement of
agriculturists from Iran to India leading to the foundation of Harappa Civilization. The third
and fourth migration took place around 2000 BCE and between 2000 and 1000 BCE
respectively.
 The book also suggests that caste system did not begin with the arrival of Aryans and
began nearly 1,000 years later in around 100 CE.

Links:
https://economictimes.indiatimes.com/news/politics-and-nation/4-prehistoric-migrations-
shaped-indias-population-book/articleshow/67299719.cms

Miscellaneous
Three Banks Merger
Question: The Cabinet recently approved the merger of Bank of Baroda with which two
banks?
(a) Dena Bank and Vijaya Bank (b) Vijaya Bank and RBL Bank
(c) RBL Bank and IDBI Bank (d) IDBI Bank and Dena Bank
Answer: (a)

140
http://www.edristi.in/
Related facts:

 On 2nd January, 2019, Union Cabinet, chaired by Prime Minister Narendra Modi
approved the ‘Scheme of Amalgamation’ for merger of Bank of Baroda, Vijaya Bank and
Dena Bank.
 The amalgamation will be the first-ever three-way consolidation of banks in India. The
scheme shall come into force on 1.4.2019.
 The merger of Bank of Baroda, Dena Bank and Vijaya Bank was proposed by the Union
Finance Ministry on September 17, 2018.

Links:
http://pib.nic.in/newsite/PrintRelease.aspx?relid=187080

Project Reweave
Question: Which of the following software giant have recently launched an e-platform
named ‘project reweave’?
(a) Google (b) Facebook (c) Microsoft (d) IBM
Answer: (c)
Related facts:

 Microsoft India has launched a new e-commerce platform for handloom weavers in
Telangana, under its Project Reweave, as a part of its philanthropic initiative.
 The e-commerce platform would help connect artisans to buyers directly enabling them to
expand to newer customers and markets.
 Microsoft in association with the National Institute of Fashion Technology (NIFT), has also
curated a special curriculum in ‘CAD and colour for handloom weaving’ to provide digital
training in handloom design.
 The first batch of hundred handloom weavers was also awarded certificates for successful
design course completion.
 Indian handloom industry is the second largest employment provider for the rural
population in India, with almost 43 lakh weavers.
 Project Reweave also aims to help weavers with working capital support through non-
profit organizations.

Links:
https://www.business-standard.com/article/companies/microsoft-launches-e-commerce-
portal-for-telangana-s-handloom-weavers-119011900236_1.html
https://www.re-weave.in/our-story

Most valued IT service brand


Question: Which of the following Indian software company has been 3rd most valued IT
service brand?
(a) Infosys (b) TCS (c) Wipro (d) HCL
Answer: (b)
Related facts:

 According to a report by Brand Finance, India’s Tata Consultancy Services (TCS) has
been ranked 3rd most-valued IT services brand globally in 2018-19 after Accenture and
IBM. Four Indian IT firms- TCS, Infosys, HCL and Wipro – grabbed spots in the top -10
global tally.

141
http://www.edristi.in/
 Accenture (Valued at $26.3 billion) took the title of world’s most valued IT services brand.
Last year, IBM was at the top, Now it slipped to second place this year (Brand Value at
US$20.4 billion).
 TCS remains in third place, (Value holding up 23 per cent to US$ 12.8 billion), with a
value aided by the brand’s disciplined focus on the market’s increased demand for digital
services.
 TCS is also the first Indian IT services brand to achieve success in the Japanese
market.TCS has positioned itself as a leader in providing a superior all round customer
experience, leveraging artificial intelligence and robotic automation across its
transformation programmes.

Facts about TCS:

 TCS is an IT services, digital and business solutions company.


 It provides end-to-end technology related services to global enterprises. The company
operate through five segments: Banking, Financial Services & Insurance Manufacturing,
Retail & Consumer Packaged goods, telecom, media & entertainment and others such as
energy, resources and utilities, high tech, life sciences and healthcare, travel
transportation and hospitality, products etc.’

Links:
https://www.indiatoday.in/india-today-at-davos-2019/story/tcs-3rd-most-valued-it-services-
brand-globally-brand-finance-at-davos-1438273-2019-01-24

Slogan Jai Jawan Jai Kisan Jai Vigyan Jai Anusandhan


Question: Recently who has given slogan of New India: Jai Jawan, Jai Kisan, Jai Vigyan,
Jai Anusandhan?
(a) Lal Bahadur Shastri (b) Atal Bihari Vajpayee (c) Narendra Modi
(d) None of the above
Answer: (c)
Related facts:

 In 106th Session of Indian Science Congress, 2019 which held from 3rd to 7th January,
2019 at Lovely Professional University Jalandhar(Punjab), PM Narendra Modi has given
slogan of New India: Jai Jawan, Jai Kisan, Jai Vigyan, Jai Anusandhan.
 Prime Minister Narendra Modi recalled the great Indian scientists of the past, including
Acharya J.C. Bose, C.V. Raman, Meghnad Saha, and S.N. Bose and praised them for
serving the people through “minimum resources” and “maximum struggle”.
 A strong research ecosystem needs to be developed in the State Universities and
Colleges which can be a fusion of Arts and Humanities, Social Science, Science and
Technology. India is willing to grow itself with a vision to be amongst top ten in the world
in the field of Science and Technology.
 At Lovely Professional University the events included Women’s Science Congress,
Children Science Congress, and Science Exhibition exhibiting various scientific
innovations. Overall 20,000 delegates from nearly 60 countries participated in this mega
science congress.

Links:
http://pib.nic.in/newsite/PrintRelease.aspx?relid=187126

142
http://www.edristi.in/
Mansukh Mandaviya undertakes 150 km Padyatra
Question:Union Minister Mansukh Mandaviya started 150 km Padyatra regarding which
of the following occasion?
(a) 150th Birth Anniversary of Mahatma Gandhi (b) 150th Birth
Anniversary of B.R. Ambedkar
(c) Both (a) & (b) (d) None of the Above
Answer:(a)
Related facts:

 Union Minister of State for Road Transport & Highways, Shipping, and Chemical &
Fertilizers Mansukh Mandaviya started 150 km Padyatra on 16th January 2019 to mark
the 150th Birth Anniversary of Mahatma Gandhi.
 The Padyatra began from Gram Daxinamurti Institute, Manar in the Talaja Taluk of
Bhavnagar district of Gujarat with tree plantation and explained the relation among
Gandhian thoughts and nature.
 Aim of Padyatra was to spread Gandhian values among peoples so as to promote
sustainable development.
 The seven day Padyatra came to an end at Sanosara, Taluk – Sihar Bhavnagar in
Gujarat on 22nd January 2019.

Links:

http://pib.nic.in/newsite/PrintRelease.aspx?relid=187573

Series for Budget on Twitter


Question: Name the title of the series started by Finance Ministry about Budget on
Twitter?
(a) About Budget (b) Aapka Budget (c) Budget Kitna Hai (d) Know Your Budget
Answer: (d)
Related facts:

 Union Finance Minister Arun Jaitley on 15th January 2019 started a series on Twitter
titled ‘Know Your Budget’.
 This series will educate the general public about budgetary process and will provide
definitions of various terms present in Budget.
 This series will continue for about a fortnight explaining the importance of Union Budget.
 The budget session of the parliament will start early due to upcoming general elections.
The interim budget will be brought to parliament on February 1 and final budget would be
presented by the new government after elections.

Links:
https://economictimes.indiatimes.com/news/economy/policy/know-your-budget-better-with-
finance-ministrys-twitter-series/articleshow/67543165.cms

UTTAM serves as an important tool for Leveraging Technology in Coal


Quality Monitoring
Question: Consider the following statement regarding UTTAM:
1) It is launched by Ministry of Railway.
2) UTTAM stands for Unlocking Transparency by Third Party Assessment of Mined Coal.

143
http://www.edristi.in/
Of the above correct statement/s is/are:
(a) Only 1 (b) Only 2 (c) Both 1 and 2 (d) None of the above
Answer (b)
Related facts:

 In its year-end review by Ministry of Coal


emphasizing its year-long achievements,
UTTAM (Unlocking Transparency by Third Party
Assessment of Mined Coal) (portal site-uttam.coalindia.in) proves to an important
milestone for coal quality monitoring.
 At present,coal supply under all FSAs and e-auction for both power utilities and non-
power consumers have been covered under Third Party Sampling.As of now, consumers
are paying the coal value for the quality of coal supplied to them based on analyzed grade
given by independent third party agencies.
 UTTAM app was launched on 5 April 2018 by Shri Piyush Goyal, Union Minister for
Railways and Coal.
 The App has ensured accountability, transparency, effectiveness and efficiency in coal
ecosystem and brought coal governance closer to people.
 The App has been jointly developed by Ministry of Coal and Coal India Limited (CIL).
 The App provides coverage of Third Party Sampling of coal across CIL subsidiaries which
include information on production, dispatch and quantity sampled of coal.

Links:
http://www.pib.nic.in/PressReleseDetail.aspx?PRID=1557888

Harbin International Ice & Snow Sculpture Festival


Question: The famous Harbin Ice Festival is organized in which country?
(a) Switzerland (b) Denmark (c) Russia (d) China
Answer: (d)
Related facts:

 The 35th edition of famous Harbin International Ice & Snow


Sculpture Festival in China started on January 5th, 2019.The
theme of the event is “20-year Grand Ceremony & Review”.
 This is a month long festival featuring over 100 activities featuring nearly 100 programs,
events & activities for ice and snow tourism, culture, fashion, trade and sports. This
festival is said to be the largest ice and snow festival in the world. This festival began in
1963.
 There are various snow sculptures erected in an area of 10 hectare on the frozen
Songhua River.
 Harbin is the capital of Heilongjiang province, and is the largest city in the northeastern
region of the People’s Republic of China. Harbin receives heavy snowfall as well as
severe winter and it is heralded as the Ice City for its well-known winter tourism and
recreations.

Links:
http://www.harbinice.com/fact-v43-the-35th-harbin-ice-and-snow-festival-2019.html

Soldierathon 2018
Question: Soldierathon Marathon 2018 was organized in which of the following city?
(a) New Delhi (b) Hyderabad (c) Lucknow (d) Bhubaneswar

144
http://www.edristi.in/
Answer: (a)
Related facts:

 On16th December, 2018, Soldierathon Marathon was organized at Jawaharlal Nehru


Stadium, New Delhi, on the occasion of Vijay Diwas.
 Union Sports Minister Col Rajyavardhan Singh Rathore flagged off the event.
 Soldierathon 1st edition 2018 was a unique marathon organized to honour the bravest of
the brave our country’s soldiers.
 Around 12,467 people participated in the 1st edition of Soldierathon.

Links:

http://www.soldierathon.com/event.html

Rise in Canadian Citizenship for Indians


Question: Rise in Canadian citizenship for Indians during 2018 is-

(a) 50% (b) 60% (c) 70% (d) 80%


Answer:(a)
Related facts:

 According to recently issued IRCC(Immigration, Refugees and


Citizenship Canada) estimate, 1.53 lakh people would have
obtained Canadian citizenship by October 2018 – being a year from the date the new
relaxed norms for citizenship came into effect.
 As per IRCC, there is a rise of 50% in Canadian citizenship for Indians by October 2018.
 Immigration, Refugees and Citizenship Canada (IRCC) is the federal department which
facilitates arrival of immigrants to Canada.
 As country of birth, India figured in the second highest number of applications from those
claiming Canadian citizenship.
 Philippines topped this list by a slender lead: 15,600 – odd Filipinos became citizens of
Canada during this ten-month period, but this was a marginal increase of 11% compared
to the traffic form Philippines for the entire 12 months of 2017.
 During the 10-month period ending October 2018 nearly 15,000-odd Indians obtained
citizenship. If compared with 2017, it is a steep rise of 50%.

Links:

https://timesofindia.indiatimes.com/india/50-rise-in-canadian-citizenship-for-indians-this-
year/articleshow/67248609.cms

Fadanavis launches Vajpayee International Schools in Mumbai


Question: Recently, Atal Bihari Vajpayee International Schools inaugurated by
Maharashtra CM Devendra Fadnavis is located in which of the following city?
(a) Mumbai (b) Delhi (c) Chennai (d) Madhya Pradesh
Answer: (a)
Related facts:

 On 25th December, 2018, Maharashtra Chief Minister Devendra Fadnavis


launched Bharat Ratna Atal Bihari Vajpayee International Schools in Mumbai on the
occasion of 94th birth anniversary of the late Prime Minister.

145
http://www.edristi.in/
 The schools are affiliated to the Maharashtra International Education Board, which the
state government had set up with an aim to achieve higher education standards of global
standards in the state.
 Addressing the occasion, CM Fadnavis notified that initially, 13 zilla parishad (ZP) schools
would be part of this international board and the curriculum will be in the mother tongue.

Links:

http://www.newsonair.com/Main-News-Details.aspx?id=357154

A Commemorative Coin in Honour of Atal Bihari Vajpayee


Question: In which year former Prime Minister Shri Atal Bihari Vajpayee was conferred
India’s highest civilian honour, the ‘Bharat Ratna’?
(a) 2015 (b) 2014 (c) 2016 (d) None of the Above
Answer: (a)
Related facts:

 The Prime Minister Shri Narendra Modi released a


commemorative 100-rupee coin in honour of former Prime Minister, Bharat Ratna Shri
Atal Bihari Vajpayee on 24thDecember, 2018.
 Vajpayee was born on 25 December 1924 in Gwalior. He was a stalwart loved and
respected across all sections of society.
 Parliamentarian for over four decades, he was elected nine times to the Lok Sabha, twice
to the Rajya Sabha and served as the Prime Minister of India for three terms, 1996, 1998,
1999-2004. He was Minister of external affairs during 1977-79 and was awarded the “Best
Parliamentarian” in 1994. Bharat Ratna award was originally confined to the arts,
literature, science, and public services, as per the 1954 regulations. In December 2011,
the rules were changed to include “any field of human endeavor”.
 Former Prime Minister Lal Bahadur Shashtri became the first individual to honoured
posthumously. In 2014, cricketer Sachin Tendulkar, then aged 40, became the youngest
recipient. While social reformer Dhondo Keshav Karve was awarded on his
100th Birthday.

Links:

http://pib.nic.in/newsite/PrintRelease.aspx?relid=186773

Tri Services Cycling & Trekking Expedition


Question: Recently, Tri Services Cycling & Trekking Expedition was conducted in which
of the following state?
(a)Sikkim (b) Himachal Pradesh (c) Uttarakhand (d) Jammu & Kashmir
Answer: (a)
Related facts:

 To enhance the spirit of adventure ingrained in the Indian Armed Forces, demonstrate
high standards of physical fitness, competitive spirit, mental robustness and camaraderie
amongst the three Services- a Tri Services adventure expedition is being conducted from
December 17, 2018 to January 05, 2019.
 The adventure is –Cycling & Trekking expedition. Tri
Services team will be represented by of three Officers, three
warrant rank officers and thirteen soldiers from the Army,
Navy and Air Force.
146
http://www.edristi.in/
 The expedition is in three phases which includes cycling-cum-trekking, para- gliding and
sea diving. Expedition team would cycle the treacherous and challenging mountains of
North Sikkim under the aegis of the Striking Lion Division for the next five days reaching
altitudes of 15,000 feet in the Lachen & Lachung Valleys.
 Apart from cycling, the team would also trek through the mountain ranges along the
Teesta River. During course of the expedition, the team will spread the message of Green
Sikkim, Clean Sikkim.

Links:
http://pib.nic.in/newsite/PrintRelease.aspx?relid=186462

Fastest Asian to Cycle


Question: Who became the fastest Asian to cycle around the
globe?
(a) Vedangi Kulkarni (b) Jenny Graham (c) Jyong Kix (d) Mustafa Ali
Answer: (a)
Related facts:

 Vedangi Kulkarni from India became the fastest Asian to cycle the globe. This was
achieved when she completed the 29,000 kms cycling required to qualify as bicycling
across the globe and landed in the early hours in Kolkata on 23rd December, 2018.
 Hailing from Pune, the 20 year old Vedangi spent 159 days cycling up to 300 kms per day
across 14 countries. She started it from Perth, Australia in July this year. Vedangi studies
sports management in University of Bornemouth, United Kingdom.
 She will now be flying back to the Australian city to complete the record by cycling a 15
kilometer distance to reach the same place from where she started.
 Thirty eight year old British Adventurer Jenny Graham became the fastest women to cycle
the globe. She achieved this feat in 124 days earlier this year.

Links:
http://www.newsonair.com/Main-News-Details.aspx?id=357047#

55th Anniversary of SSB


Question: Which of the following Armed Force had its 55th Anniversary on 24th
December, 2018?
(a) Assam Rifles (b) ITBP (Indo Tibetan Border Police)
(c) Border Security Forces (d) Sashastra Seema Bal
Answer: (d)
Related facts:

 The 55th Anniversary of armed force Sashastra Seema Bal (SSB) was organized in New
Delhi on 24th December, 2018. The chief guest at the ceremony was Minister of State for
Home Affairs, Kiren Rijiju. Delegates from Nepal and Bhutan were also present during the
Raising Day parade. The event also had parade of 52 international medalists from SSB,
martial arts and mountaineering skills demonstration, performance by dare devil team, a
motorcycle squad of SSB in the conclusion.
 This force was conceived in November 1962 following Sino-India war and created in
March 1963 with the objective of ‘Total security preparedness’ in the remote border areas.
As of 2017, it has 76,337 active personnel in 67 battalions.

Links:
http://www.newsonair.com/Main-News-Details.aspx?id=357091
147
http://www.edristi.in/

You might also like